Sie sind auf Seite 1von 107

: Please use with discretion; digest includes cases assigned for CaseZ; doctrines only for cases not

assigned but in the syllabus. Hope this helps 😊

I. CONCEPT AND ATTRIBUTES OF A CORPORATION

1. MAGSAYSAY-LABRADOR v. CA and RODRIGUEZ-MAGSAYSAY


G.R. No. 58168
December 19, 1989

FACTS
- On February 9, 1979, private respondent, Adelaida Rodriguez-Magsaysay, widow of late Senator Genaro Magsaysay brought an
action against Artemio Panganiban, Subic Land Corp., Filipinas Manufacturer’s Bank (FILMANBANK), and Register of Deeds of
Zambales alleging that:
- In 1958, she and her husband, acquired a parcel of land known as Pequena Island using conjugal funds;
- That she discovered at the back of the Title an annotation that it was acquired by her husband from his separate capital;
- That the late Senator purportedly executed a Deed of Assignment in favor of Subic in 1976 covering the same;
- And Subic executed a Deed of Mortgage in 1977 in favor of Filmanbank covering the same;
- That the said Deeds were void considering that the land is conjugal and her consent was not obtained in the annotation in the Title.
- She therefore alleged that the assignment in favor of Subic is consequently null and void.

- Petitioners, sisters of the late Senator filed a motion for intervention on the ground that:
- In 1976, their brother conveyed half of his shareholdings in Subic, a total of 416,566.6 shares and 41% of total outstanding
shares of such stocks
- And that they have substantial and legal interest in the subject matter of litigation
- Trial court denied motion for intervention, ruling that they have no legal interest in the matter in litigation, that being assignees of
such shares does not entitle them to intervene because Subic has a personality separate and distinct from its stockholders.
- CA affirmed

ISSUE: Whether or not the petitioners have substantial and legal interest in the property, and hence can intervene in the case

HELD:
- No. They do not have legal interest in the subject matter as to entitle them to intervene in the proceedings.
- To allow intervention:
a) it must be shown that the movant has legal interest in the matter in litigation, or otherwise qualified; and
b) consideration must be given as to whether the adjudication of the rights of the original parties may be delayed or prejudiced,
or whether the intervenor’s rights may be protected in a separate proceeding or not.
Note that both must concur.
- The words, “an interest in the subject” mean a direct interest in the cause of action as pleaded, which would put the intervenor in
a legal position to litigate a fact alleged in the complaint.

- While a share of stock represents a proportionate or aliquot interest in the property of the corporation, it does not vest the owner
thereof with any legal right or title to any of the property, his interest in the corporate property being equitable or beneficial in nature.
- At the very least, their interest is purely inchoate, or in sheer expectancy of a right in the management of the corporation and to
share in the profits thereof and in the properties and assets thereof on dissolution, after payment of the corporate debts and obligations.
- Shareholders are in no legal sense the owners of corporate property, which is owned by the corporation as a distinct legal person.
CASE DIGEST | Corporation Law (2019) Atty. Villanueva-Castro ib3h

2. SULO NG BAYAN, INC. v. ARANETA


G.R. No. L-31061
August 17, 1976

FACTS
- On April 26, 1966, petitioner filed an accion de reivindicacion with Bulacan CFI, against defendant (respondents) to recover the
ownership and possession of a large tract of land in San Jose del Monte, Bulacan, containing an area of 27,982,250 sqm registered
under the Torrens system.
- It alleged that the members of the petitioner corporation through themselves and their predecessors-in-interest, pioneered the
clearing of the tract of land, cultivated the same since the Spanish period, and continuously possessed the property (OCENPO) under
the concept of ownership adverse against the whole world;
- and that Gregorio Araneta, in 1958, ejected the members from their possession through force and intimidation.
- They allege that the land in question was fraudulently or erroneously included in the Certificate of Title No. 466 which they only
discovered in 1961, and therefore subsequent titles derived therefrom are void.
- Trial court dismissed the complaint on ground of lack of cause of action and prescription.
- CA certified the case to the Supreme Court upon finding that no question of fact was involved but only questions of law and
jurisdiction.

ISSUE: Whether or not the plaintiff corporation (non-stock) may institute an action in behalf of its individual members for the
recovery of certain parcels of land allegedly owned by said members; for the nullification of the transfer certificates issued in favor
of defendants covering the said land, among others

HELD:
- No. The trial court correctly dismissed the amended complaint.
- A corporation is a distinct legal entity to be considered as separate and apart from the individual stockholders or members who
compose it, and is not affected by the personal rights, obligations and transactions of its stockholders or members.
- Conversely, a corporation ordinarily has no interest in the individual property of its stockholders unless transferred to the
corporation, even in the case of a one-man corporation.
- The only exception where the personality of the corporation may be disregarded, in cases where it is used as a cloak or
cover for fraud or illegality, or to work an injustice, or where necessary to achieve equity. (Aforementioned cases has no
application in the case at bar, therefore cannot be applied)
- It has not been claimed that the members have assigned or transferred whatever rights they may have on the land in question to the
petitioner corporation.
- Therefore, absent showing of interest, the corporation has no personality to bring an action for and in behalf of its stockho lders or
members for recovering property which belong to the stockholders/members in their personal capacities.
- The corporation do not have any interest in the subject matter of the case which is material and direct so as to entitle it to file the
suit as a real party in interest.

2
CASE DIGEST | Corporation Law (2019) Atty. Villanueva-Castro ib3h

3. BATAAN SHIPYARD & ENGINEERING, CO., INC. (BASECO) v. PCGG


G.R. No. 75885
May 27, 1987

FACTS
- Challenged in this case are EOs 1 and 2, signed by then President Aquino on February 28, 1986 and March 12, 1986; and the
sequestration, takeover, and other orders issued by the PCGG in accordance with the said EOs affecting the petitioner corporation.
- The Basic sequestration order was issued April 14, 1986 addressed to the agents of PCGG, sequestering BASECO, among others
- On the strength of the sequestration order, Jose Balde, acting for the PCGG addressed a letter dated April 18, 1986 to the President
and other officers of the firm for the request for the production of certain documents including: 1) Stock Transfer Book; 2) Legal
documents such as articles of incorporation, by-laws, minutes of the meeting; 3) Yearly list of stockholders; 4) Financial statements;
5) inventory listings; 6) list of depository banks, among others.
- The letters were enclosed with a warning that if the documents were not submitted within 5 days, the officers would be
cited for contempt in pursuance with EOs No. 1 and 2.
- By the letter dated July 14, 1986, Commissioner Ramon Diaz decreed the provisional takeover by the PCGG of BASECO, invoking
Sec. 3(c) of EO 1, empowering the PCGG to “provisionally takeover in the public interest or prevent its disposal x x x business
enterprises and properties taken over by the government of the Marcos administration or entities or persons close to former President
Marcos x x x”
- Petitioner BASECO asks the Supreme Court:
a) declare unconstitutional and void EOs 1 and 2
b) annul the sequestration order and all other orders subsequently issued and acts done on the basis thereof, inclusive of the
takeover.
- Particularly, it argues that the order to produce corporate records from 1973 to 1986 was issued without court authority and infringed
its constitutional right against self-incrimination, and unreasonable search and seizure.

ISSUE: Whether or not the order to produce corporate records violates the right of BASECO against self-incrimination and
unreasonable search and seizure.

HELD:
- It is elementary that the right against self-incrimination has no application to juridical persons; and the constitutional right against
unreasonable search and seizure finds no application to the case at bar.
- While an individual may lawfully refuse to answer incriminating questions unless protected by an immunity statute, it does not
follow that a corporation, vested with special privileges and franchises, may refuse to show its hand when charged with an abuse of
such privileges.
- It is also settled that an officer of the company cannot refuse to produce its records in its possession, upon the plea that they will
either incriminate him or may incriminate it.
- It would be a strange anomaly to hold that a state, having chartered a corporation to make use of certain franchises could not, in the
exercise of sovereignty, inquire how these franchises had been employed, and whether they had been abused, and demand the
production of the corporate books and papers for that purpose.

3
CASE DIGEST | Corporation Law (2019) Atty. Villanueva-Castro ib3h

4. LUXURIA HOMES, INC., v. CA


G.R. No. 125986
January 28, 1999

FACTS
- Petitioner Aida Posadas and her 2 minor children co-owned a 1.6 hectare property in Sucat, Muntinlupa occupied by squatters.
- She entered into negotiations with private respondent Bravo regarding development of the said property into a residential
subdivision, and also authorized to negotiate with the squatters to leave the property on May 1989.
- On December 11, 1989, petitioner assigned, through a Deed of Assignment the same property to Luxuria Homes, Inc. purportedly
for organizational and tax avoidance purposes, Bravo as one of the witnesses.
- In 1992, their relationship turned sour because Posadas could not accept the management contracts Bravo was proposing
- In retaliation Bravo demanded payment for services rendered in connection with the development of the land in the amount of
P1,708,489.
- Posadas refused to pay the amount hence Bravo and James Builder Construction instituted a complaint for specific performance
before the trial court.
- The trial court held Posadas and Luxuria Homes liable in solidum to pay the private respondents.
- CA affirmed with modification of deletion of moral damages.
- In January 15, 1997, this Court denied due course to the petition, showing no reversible error on the part of CA. Filed a motion for
MR, and found merit.

ISSUE: Whether or not Luxuria Homes was a party to the transactions entered into by Posadas and hence can be held jointly and
severally with Posadas

HELD:
- No. The Court held that CA committed a reversible error when it upheld that petitioner’s liability was aggravated by the fact that
Luxuria Homes was formed by petitioner Posadas after demand for payment was made.
- Factual records provide that the Deed of Assignment and the Articles of Incorporation of Luxura Homes, issued on 11 December
1989, and 26 January 1990, respectively, signed by Bravo as witness, and the relationship of the two were very pleasant at the time.

- Besides, Posadas is not a majority stockholder of the corporation, owning only 33% of the capital stock. It cannot be considered
then that Posadas is an alter ego of the petitioner corporation.

- To disregard the separate juridical personality of a corporation, the wrongdoing must be clearly and convincingly established, that
the corporation is used as a cloak or cover for fraud or illegality, or to work injustice, or where necessary for the protection of the
creditors.
- In this case, the private respondents failed to show proof that Posadas acted in bad faith. Consequently, the respondents failed to
show that Luxuria Homes, Inc., was a party to any of the transactions, and since it was petitioner Posadas who contracted with Bravo
to render the services, it is only she that is liable to pay the amounts adjudged.

4
CASE DIGEST | Corporation Law (2019) Atty. Villanueva-Castro ib3h

5. CONCEPT BUILDERS, INC., v. NLRC


G.R. NO. 108734
May 29, 1996

FACTS
- Petitioner corporation is a domestic corporation with principal office at 355 Maysan Rd., Valenzuela, and engaged in construction business.
- Private respondents then filed a complaint for illegal dismissal, unfair labor practice, and non-payment of holiday pay, overtime pay and thirteenth-
month pay against petitioner corporation for terminating the private respondents when the project they were hired had not yet been finished and
complete. .
- The Labor Arbiter rendered judgement against the petitioner ordering them to reinstate the workers and pay them back wages equivalent to 300
days.
- After dismissing the MR by the NLRC, the LA issued a writ of execution and was partially satisfied. Another Alias Writ of Execution was issued
for the balance, but the service was refused on the ground that petitioner no longer occupied the premises.
- It was claimed that Hydro Pipes Philippines (HPPI) already occupied the premises at 355 Maysan Rd., Valenzuela.

- Thereafter, Dennis Cuyegkeng, claiming to be the Vice-President of HPPI filed a third-party claim with the LA alleging that the properties sought
to be levied upon were owned by HPPI; and that it is a separate and distinct from the petitioner.
- private respondents then filed a Motion for Issuance of a Break-Open alleging that HPPI and petitioner corporation were owned by the same
incorporator/stockholders, as evinced by the General information sheet submitted by petitioner and HPPI with the SEC.
- LA denied the Motion; NLRC reversed LA.
- Before this court, the petitioner averred grave abuse of discretion on the part of NLRC when it ordered the execution for, among others, that the
doctrine of piercing of the corporate veil should not be applied in the absence of showing that it created HPPI in order to evade its liability.

ISSUE: Whether or not the NLRC committed grave abuse of discretion when it issued a “break-open order” to the sheriff to be
enforced against personal property found in the premises of petitioner’s sister company.

HELD:
- No. The petitioner’s contention is unmeritorious.
- When the notion of separate juridical personality is used to defeat public convenience, justify wrong, protect fraud, or defend crime, or is used as
a device to defeat the labor laws, this separate personality of the corporation may be disregarded or the veil of corporate fiction pierced. This is true
when the corporation is merely an adjunct, a business conduit, or an alter ego of another corporation.
- There are no hard and fast rule in determining justification of the application of the doctrine of the piercing of corporate veil, and vary according
to the peculiar facts and circumstances of each case. Certainly though, there are some probative factors of identity that will justify the application
of the doctrine, to wit:
1. Stock ownership by one or common ownership of both corporations;
2. Identity of directors and officers
3. The manner of keeping corporate books and records
4. Methods of conducting the business.
- Moreover, the SEC en banc explained the instrumentality rule which the courts applied in disregarding the separate juridical personality of
corporations as follows:
- Where one corporation is so organized and controlled and its affairs are conducted so that it is, in fact, a mere instrumentality or adjunct
of another, the fiction of the corporate entity of the instrumentality may be disregarded.
- The control necessary to invoke the rule is not majority or even complete stock control but such domination of finances, policies, and practices
that the controlled corporation has, so to speak, no separate mind, will or existence of its own, and is but a conduit for its principal.
- The question then of whether the corporation is a mere alter ego, is purely a fact.
- The NLRC noted that the petitioner filed an Information Sheet with SEC on May 15, 1987 stating the address as the same as aforementioned, and
on the other hand, the HPPI submitted on the same day, the similar information sheet stating that the office is at the address the same as the
petitioner.
- Clearly, HPPI is obviously a business conduit of petitioner corporation and its emergence was skillfully orchestrated to avoid financial liability
that already attached to petitioner corporation.

5
CASE DIGEST | Corporation Law (2019) Atty. Villanueva-Castro ib3h

6. FRANCISCO MOTORS CORPORATION v. CA


G.R. NO. 100812
June 25, 1999

FACTS
- On January 23, 1985, petitioner filed a complaint against private respondents Gregorio and Librada Manuel to recover the amount
of balance of jeep body purchased by the Manuels, unpaid balance on the cost of the repair of the vehicle, from the petitioner, with
cost of suit and attorney’s fees.
- Private respondents filed a counterclaim for unpaid legal services by Gregorio Manuel in the amount of P50,000 not paid by the
incorporators, officers, and directors of the petitioner.
- Trial court ruled in favor of the petitioners but also allowed the counter-claim, upheld by the CA.
- Private respondent alleged that while he was petitioner’s Assistant Legal Officer, he represented members of the Francisco family
in the intestate estate proceedings of the late Benita Trinidad. However, after the termination of the proceedings, he was not paid.
The said family members were also incorporators, directors, and officers of the petitioner.
- Petitioners, averred that there was no cause of action by the respondent against petitioner, that the personal concerns of the heirs
should be distinguished from those involving corporate affairs. Therefore, the CA should not have resorted to piercing the veil of
corporate fiction for it does not fall among the instances wherein the courts may look beyond the distinct personality of a corporation.

ISSUE: Whether or not the petitioner could be held liable to the private respondent

HELD:
- Under the doctrine of piercing the veil of corporate entity, the corporation’s separate juridical personality may be disregard, for
example, when the corporate identity is used to:
- defeat public convenience,
- justify wrong,
- protect fraud, or
- defend crime.
- Given the facts of this case, the doctrine has no relevant application here. The estate proceedings did not involve any business of
the petitioner.
- The offsetting the unpaid balance of the purchase and repair of a jeep body, is a misapprehension that the corporate assets could be
used to answer for the liabilities of its individual directors, officers, and incorporators and could easily prejudice the corporation, its
own creditors, and even other stockholders.
- When directors and officers of a corporation are unable to compensate a party for a personal obligation, it is far-fetched to allege
that the corporation is perpetuating fraud or promoting injustice, and be thereby be held liable therefor by piercing its corporate veil.

6
CASE DIGEST | Corporation Law (2019) Atty. Villanueva-Castro ib3h

7. TIMES TRANSPORTATION COMPANY v. SOTELO ET AL.


G.R. NO. 163786
February 16, 2005

FACTS
- Petitioner is a corporation engaged in the business of land transportation.
- Times Employees Union (TEU) held a strike against Times. Thereafter, in a certification election, TEU was certified as the sole
and exclusive collective bargaining agent of Times.
- In August 1997, TEU filed a notice of strike, then another on October 1997 on grounds of unfair labor practices on the part of
Times.
- In the meantime, Mencorp Transport Systems, Inc. had acquired ownership of Time’s Certificate of Public Convenience and number
of bus units by virtue of deeds of sale. Mencorp is controlled and operated by Virgina Mendoza, daughter of Santiago Rondaris, the
majority stockholder of Times.
- After the closure of the petitioner company in 1997, the respondents filed cases for illegal dismissal, money claims and and unfair
labor practices before the Regional Arbitration Branch in San Fernando La Union. However, the arbitration branch archived the case
pending resolution of another case pending before the Supreme Court.
- Respondents then withdrew the complaint and filed a new set of case before the NCR Arbitration Branch impleading
Mencorp and Spouses Mendoza
- The Labor Arbiter both held Times and Mencorp liable to pay jointly and severally.
- Times, Mencorp, and Spouses Mendoza appealed to NLRC, which the NLRC denied for lack of merit.
- In the MR, NLRC vacated the decision appealed, and “remanded to the Arbitration Branch of origin for disposition”.
- Respondents appealed to CA attributing grave abuse of discretion on the NLRC for, among others, not ruling that Times and
Mencorp are one and the same entity.
- CA reinstated the decision of the LA, vacating that of NLRC.

ISSUE: Whether or not the piercing the corporate veil is warranted

HELD
- Yes. Piercing of the corporate veil is warranted only in cases when the separate legal entity is used to defeat public convenience,
justify wrong, protect fraud, or defend crime, such that in case of two corporations, the law will regard the corporations as merged
into one.
- It may be allowed only if the following elements concur:
a) control - not mere stock control, but complete domination - not only of finances, but of policy and business practice in
respect to the transaction attacked;
b) such control must have been used to commit a fraud or a wrong to perpetuate the violation of a statutory or other positive
legal duty, or a dishonest and an unjust act in contravention of a legal right; and
c) said control and breach of duty must have proximately caused the injury or unjust loss complained of.
- The factual findings corroborate the fact:
a) Sale was transferred to a corporation by V. Mendoza, daughter of Rondaris of TImes where she is also a director;
b) All of the stockholders/incorporators of Mencorp are all relatives of Rondaris;
c) Timing of the sale to negatethe employees’ right to organization
d) The buses of TImes Transportation already being run/operated by Mencorp.
- It is therefore evident that the transaction was made in order to remove Times’ remaining assets from the reach of any judgment
that may be rendered in the unfair labor practice cases filed against it.

7
CASE DIGEST | Corporation Law (2019) Atty. Villanueva-Castro ib3h

8. YAO, SR., v. PEOPLE


G.R. No. 168306
June 19, 2007

FACTS
- Petitioners are incorporators and officers of Masagana Gas Corporation; private respondents Petron is the registered owner of the
trademarks Gasul and Gasul cylinders, and Pilipinas Shell as the owner of trademarks of Shellane and Shell device.
- On April 3, 2003, NBI filed applications for search warrant against petitioner Yao for alleged violation of Sec. 155, in relation to
Sec. 170 of RA 8293, also known as the Intellectual Property Code of the Philippine, which was issued by Judge Sadang of RTC
Cavite City. The articles mentioned in the search warrant were subsequently seized.
- Petitioners filed with the RTC motion to quash on both search warrants (2), while Masagana as third party claimant, filed wit h the
RTC a Motion for Return of Motor Compressor and LPG Refilling Machine
- Masagana argued that these were used in the operation of legitimate business, and its seizure will jeopardize its business interests.
- Petition of Masagana was dismissed because evidence disclosed that petitioners are stockholders of Masagana and they conduct
their business through the same juridical entity. Also stated that the veil of corporate fiction cannot be used as a refuge from liability.
- CA affirmed RTC.

ISSUE:Whether or not the Masagana has the right to intervene and move for the return of the seized items

HELD
- The law will not recognize the separate corporate existence if the corporation is being used pursuant to x x x unlawful objectives.
- Where the separate corporate entity is disregarded, the corporation will be treated merely as an association of persons and the
stockholders or members will be considered as the corporation, that is, liability will attach personally and directly to the officers and
stockholders.
- The petitioners, as directors/officers of Masagana, are utilizing the corporation in violating the intellectual property rights of Petron
and Pilipinas Shell.
- Thus, petitioners collectively and Masagana should be considered as one and the same person for liability purposes.
- Masagana’s third party claim then serves no refuge for petitioners.

8
CASE DIGEST | Corporation Law (2019) Atty. Villanueva-Castro ib3h

9. SEVENTH DAY ADVENTIST CONFERENCE CHURCH OF SOUTHERN PHILIPPINES, INC.


v. NORTHEASTERN MINDANAO MISSION OF SEVENTH DAY ADVENTIST, INC.
G.R. No. 150416
July 21, 2006

FACTS
- The case involves a 1069 sqm lot covered by TCT No. 4468 in Bayugan, Agusan del Sur owned by Felix Cosio and his wife, Felisa
Cuysona.
- Spouses Cosio donated the land to the South Philippine Union Mission of Seventh Day Adventist Church of Bayugan Esperanza,
Agusan (SPUM-SDA Bayugan) on 1959, accepted by Liberato Rayos, an elder of the church.
- In 1980, the same parcel of land was sold by the spouses to Seventh Day Adventist Church of Northeastern Mindanao Mission
(SDA-NEMM).
- Petitioners asserted ownership over the property, opposed by the respondents who argued that at the time of the donation, SPUM-
SDA Bayugan could not legally be a donee because, not having incorporated yet, it had no juridical personality.
- In the civil case filed by the petitioners, the trial court upheld the sale in favor of the respondents.
- CA affirmed

ISSUE: Whether SDA-NEMM’s ownership of the lot be upheld

HELD:
- Yes. Consequently, the alleged donation to petitioners was void.
- Donation is an act of liberality whereby a person disposes gratuitously of a thing or right in favor of another person who accepts
it.
- The donation could not have been made in favor of an entity yet inexistent at the time it was made. SPUM-SDA Bayugan had
neither juridical personality nor capacity to accept such gift.
- There are stringent requirements before one can qualify as a de facto corporation:
a) existence of a valid law under which it may be incorporated;
b) attempt in good faith to incorporate;
c) assumption of corporate powers.
- There is no proof that there was an attempt to incorporate at that time.
- The filing of articles of incorporation and the issuance of the certificate of incorporation are essential for the existence of a de facto
corporation. An organization not registered with the SEC cannot be considered a corporation in any concept, not even as a corporation
de facto.

- Corporate existence begins only from the moment a certificate of incorporation is issued. When no such certificate was issued to
petitioners or their supposed predecessor-in-interest at the time of donation, petitioners obviously could not have claimed succession
to an entity that never came to exist.

9
CASE DIGEST | Corporation Law (2019) Atty. Villanueva-Castro ib3h

10. LIM TONG LIM v. PHILIPPINE FISHING GEAR INDUSTRIES


G.R. NO. 136448
November 3, 1999

FACTS
- Antonio Chua and Peter Yao entered into a contract in behalf of “Ocean Quest Fishing Corporation” in February 1999 for the
purchase of fishing nets and floats from the petitioner PFGI.
- They claimed they were engaged in a business venture with petitioner Lim Tong Lim, who is not a signatory to the agreement.
- The buyers however failed to pay the fishing nets amounting to P532,045 and floats amounting to P68,000 and so private
respondents filed a collection suit against Chua, Yao and Lim with writ of preliminary attachment.
- The suit was brought against them in their capacity as general partners on the allegation that “Ocean Quest Fishing Corporation”
was a nonexistent corporation.
- Chua admitted his liability, Yao failed to appear in subsequent hearings, while petitioner Lim filed an Answer with Crossclaim and
Counterclaim and moved for the lifting of the Writ of Attachment.
- The trial court held that Chua, Yao, and Lim were jointly liable as general partners to pay respondent, and that a partnership between
them existed.
- CA affirmed RTC.

ISSUE: Whether or not petitioner may be held liable for the fishing nets and floats purchased from respondent.

HELD:
- Yes. While it was clear that Chua, Yao, and Lim is engaged in the partnership engaged in the fishing business, the Court disagrees
that only Chua and Yao is only liable under the doctrine of corporation by estoppel.
- An unincorporated corporation has no personality and would be incompetent to act and appropriate for itself the power and attributes
of a corporation as provided by law; it cannot create agents or confer authority on another to act in its behalf; thus, those who act or
purport to act as its representatives or agents do so without authority and at their own risk.
- A person acting or purporting to act on behalf of a corporation which has no valid existence assumes such privileges and obligations
and becomes personally liable for contracts entered into or for other acts performed as such agent.
- On the other hand, a third party who, knowing an association to be unincorporated, nonetheless treated it as a corporation and
received benefits from it, may be barred from denying its corporate existence in a suit brought against the alleged corporation.

- Insofar as the fact the name of Lim does not appear on any of the contracts and since he never directly transacted with the respondent
corporation, the Court held that the petitioner benefitted from the use of the nets inside the boat that has been proven to be an asset
of the partnership; that the attachment of the nets effectively stopped his use of the fishing vessel.

- Therefore, though did not directly act on behalf of the corporation, having reaped the benefits of the contract entered into by persons
with whom he previously had an existing relationship, he is deemed to be part of said association and is covered by the scope of the
doctrine of corporation by estoppel.

10
CASE DIGEST | Corporation Law (2019) Atty. Villanueva-Castro ib3h

11. INTERNATIONAL EXPRESS TRAVEL & TOUR SERVICES, INC. v. CA, HENRI KAHN
G.R. No. 119020
October 19, 2000

FACTS
- Petitioner IETTS wrote a letter on June 1989 to the Philippine Football Federation (PFF) through its president, Henri Kahn, wherein
the former offered services as a travel agency to the latter, which was accepted.
- Petitioner secured airline tickets for the trips of the athletes and officials in SEA Games in Kuala Lumpur, as well as trips in China
PR, and Brisbane (Australia). The tickets amounted to P449,654.83, and the PFF made two partial payments of P176,467.50.
- The tickets weren’t fully paid, which prompted the petitioner to file a civil case before the Manila RTC, suing Henri Kahn in his
personal capacity as the President of the Federation and the PFF as an alternative defendant.

- Henri Kahn averred that petitioner has no cause of action against him because he merely acted as an agent of the Federation which
has a separate and distinct juridical personality.
- RTC ruled in favor of petitioner, CA reversed the trial court, recognizing the juridical existence of PFF that is separate and distinct
from its officers, citing RA 3135, the Revised Charter of the Philippine Amateur Athletic Federation, and PD 604.

ISSUE: Whether or not PFF exists as a juridical person.

HELD:
- No, While RA 3135 and PD 604 recognized the juridical existence of national sports associations, it is basic postulate that before a
corporation may acquire juridical personality, the State must give its consent either in a form of a special law or a general enabling
act.
- It was held that PFF did not came into existence upon the passage of these laws; nor it can be found in the provisions creating the
PFF. These laws merely recognized the existence of national sports associations and provided the manner by which these entities
may acquire juridical personality.
- Clearly, before an entity may be considered as a national sports association, it must be recognized by the accrediting organization,
the Philippine Amateur Athletic Federation (RA 3135), and the Department of Youth and Sports Development (PD604), which Kahn
failed to substantiate the fact of recognition.

- Any person acting or purporting to act on behalf of a corporation which has no valid existence assumes such privileges and becomes
personally liable for contract entered into or for other acts performed as such agent.
- Even assuming that PFF was defectively incorporated, the petitioner cannot deny the existence of the PFF because it had contracted
and dealt with the PFF in such a manner as to recognize and in effect admit its existence.
- Therefore, private respondent should be held liable for the unpaid obligations of the unincorporated PFF.

11
CASE DIGEST | Corporation Law (2019) Atty. Villanueva-Castro ib3h

12. FILIPINAS BROADCASTING NETWORK, INC., v. AGO MEDICAL AND EDUCATIONAL


CENTER (AMEC-BCCM)
G.R. NO. 141994
January 17, 2005

FACTS
- Expose is a radio documentary hosted by Carmelo Rima and Hermogenes Alegre over DZRC-AM owned by the petitioner (FBNI)
and hear all over Legazpi, Albay and other Bicol areas.
- On December 14 and 15, 1989, they exposed various alleged complaints against respondents and its administrators.
- Claiming that the broadcasts were defamatory, AMEC and Ago, as Dean of AMEC’s College of Medicine, filed a complaint for
damages against FBNI, Rima, and Alegre.
- The complaint alleged that AMEC is a reputable learning institution, and that the exposes destroyed AMEC’s reputation.
- The trial court found FBNI and Alegre liable for libel, but not Rima.
- CA made Rima solidarily liable with them.
- Before this Court, petitioner contends that AMEC is not entitled to moral damages because it is a corporation.

ISSUE: Whether or not AMEC is entitled to damages.

HELD:
- Yes.
- A juridical person generally is not entitled to moral damages because, unlike a natural person, it cannot experience physical suffering
or such sentiments as wounded feelings, serious anxiety, mental anguish, or moral shock.
- However, it is an obiter dictum that a corporation may have good reputation which, if besmirched, may also be a ground for the
award of moral damages.
- AMEC’s claim for moral damages falls under Art. 2219 (7) of the Civil Code, which expressly authorizes claim for moral damages
in cases of libel, slander, or any other form of defamation. The provision does not qualify whether the plaintiff is a natural or juridical
person.
- Therefore, a juridical person such as a corporation can validly complain for libel or any other form of defamation and claim for
moral damages.
- Here, where the broadcast is libelous per se, the law implies damages.

12
CASE DIGEST | Corporation Law (2019) Atty. Villanueva-Castro ib3h

13. COASTAL PACIFIC TRADING, INC. v. SOUTHERN ROLLING MILLS, CO, INC., (VISCO)
ET. AL.
G.R. NO. 118692
July 28, 2006

FACTS
- Respondent Southern Rolling Mills, Co., Inc., was organized in 1959, then renamed into Visayan Integrated Steel Corporation
(Visco).
- It obtained a loan of P836,000 in 1961 from DBP secured by a Real Estate Mortgage over its 3 parcels of land.
- Then in 1963, it entered into a Loan Agreement with the respondent banks (Consortium) for the amount of P21,745,707.36 to
finance the importation of raw materials, secured by a second mortgage on the land.
- When VISCO defaulted in their payment, negotiations were conducted between them (Visco and Consortium) for the conversion
of the unpaid loan into equity in the corporation. In time, the creditor banks agreed to group themselves into a creditors’ consortium
and as a result, it acquired more than 90% of its equity, but Visco remain indebted in the amount of P16,123,918.02

- Meanwhile, VISCO entered into a processing agreement with the petitioner wherein the latter delivered 3,000 metric tons of hot
rolled steel coils to VISCO for processing. However, VISCO was only able to deliver only 1,600 metric tons.

- In 1974, Jose Fernandez, Jr., Visco chairman and FEBTC President, expressed his apprehension that either DBP or the government
would soon pursue extra-judicial foreclosure against Visco. He also informed that two corporations were interested with purchasing
VISCO’s generator sets.
- The sale to Filmag was approved and took place, the proceeds deposited in a special account held in trust for the Consortium.

- Meanwhile, petitioner filed with the Pasig RTC Complaint for Recovery of Property and Damages alleging that VISCO fraudulently
misapplied or converted the finished steel sheets entrusted to it.
- While petitioner’s case was pending, VISCO vice-president requested from FEBTC a cash advance for the full settlement of
VISCO’s account with DBP (P1,342,656.88)
- Upon payment, the Deed signified that all of DBP’s rights under the mortgage agreement with VISCO were being transferred and
conveyed to the Consortium.
- Consortium then filed a Petition for Extrajudicial Foreclosure with which the auction sale took place by which Consortium emerged
as the highest bidder. VISCO assigned its right of redemption to NSC, the Consortium selling it to the Consortium was fraudulent
- Trial court ruled in favor of Coastal, and ordered VISCO to pay P851,316.9 and attorney’s fees.
- CA upheld the Consortium’s mortgage, foreclosure, and assignments.

ISSUE: Whether or not petitioner is entitled to moral damages

HELD:
- No. As a rule, a corporation is not entitled to moral damages because, not being a natural person, it cannot experience physical
suffering or sentiments like wounded feelings, serious anxiety, mental anguish, and moral shock. The only exception to this rule is
when the corporation has a good corporation that is debased, resulting in its humiliation in the business realm.
- The records do not show that the name and reputation of petitioner has been sullied as a result of the Consortium’s acts.
- As shown, Consortium took over the management and control of VISCO by acquiring 90% of the latter’s equity. As directors of
VISCO, the officials were in a position of trust; and owed it a duty of loyalty. Because they were deemed trustees of the creditors in
those instances, they should have managed the corporation’s assets with strict regard for (other) creditor’s interests. They should
have not permitted themselves to secure any undue advantage over other creditors.
- The Consortium failed miserably to observe its duty of fidelity towards VISCO and its creditors.
- At this point, an exemplary damages was granted as a way of example or correction for the public good.

13
CASE DIGEST | Corporation Law (2019) Atty. Villanueva-Castro ib3h

II. FORMATION AND ORGANIZATION OF A PRIVATE CORPORATION

14. LYCEUM OF THE PHILIPPINES, INC. v. CA


G.R. NO. 101897
March 5, 1993

FACTS
● Petitioner registered with the SEC using the corporate name Lyceum of the Philippines, Inc. on September 21, 1950, and has used
that name ever since.
● On February 24, 1984, petitioner instituted before the SEC to compel private respondents, also educational institutions, to delete
the word “Lyceum” from their corporate names and permanently enjoin them from using it as part of their respective names.
● Previously, in G.R. No. L-46595, the petitioners obtained a favorable decision in this Court against Lyceum of Baguio, Inc. which
the former used to advise the private respondents to discontinue the word “Lyceum”
● When this recourse failed, it instituted the petition before the SEC to which the SEC hearing officer ruled relying in the Lyceum of
Baguio case and held by that the word “Lyceum” was capable of appropriation and that the petitioners had acquired an enforceable
exclusive right to use the word.
● SEC en banc set aside the decision, holding that 1) the word “Lyceum” has not become so identified with petitioner as to render
use thereof by other institutions to have caused confusion to the mind of the general public; and 2) the attaching of geographical
names to the word “Lyceum” served sufficiently to distinguish the schools from one another, and that the petitioner and respondent
schools were physically remote to each other.
● CA affirmed the decision of SEC en banc.

ISSUE: Whether the names of the respondents are confusingly similar with the petitioner; whether the use by petitioner of “Lyceum”
in its corporate name has been for such length of time and with such exclusivity as to have become associated or identified with the
petitioner in the mind of the general public. (application of doctrine of secondary meaning)

HELD:
● No. Sec. 18 of the Corporation Code states that no corporate name may be allowed by the SEC if the proposed name is identical
or deceptively or confusingly similar to that of any existing corporation x x x or is patently deceptive, confusing, or contrary to
existing laws.
○ This is for the avoidance of fraud upon the public which would have occasion to deal with the entity concerned, the evasion
of legal obligations and duties, and the reduction of difficulties of administration and supervision over corporations.
● In this case, while they both carry the word “Lyceum”, the confusion and deception are effectively precluded by the appending of
geographic names to the word “Lyceum.”
● Today, “Lyceum” generally refers to a school or institution of learning, and therefore it is not unnatural to use this word to designate
an entity which is organized as an educational institution.
● The doctrine of secondary meaning is elaborated as that when “a word or phrase originally incapable of exclusive appropriation
with reference to an article on the market, might nevertheless have been used so long and so exclusively by one producer that x x
x the word or phrase has come to mean that the article was his product.”
● The circumstance has been referred to as the distinctiveness into which the name or phrase has evolved through the substantial and
exclusive use of the same for a considerable period of time.
● The CA was correct in holding that no evidence was presented in the hearing which sufficiently proved that the word “Lyceum”
has indeed acquired secondary meaning in favor of the appellant.
● The number alone of the private respondents in the case at bar suggests strongly that the petitioner’s use of the word has not been
attended with the exclusivity essential for applicability of the doctrine of secondary meaning.
● In fact, Western Pangasinan Lyceum, Inc. used the term 17 years BEFORE the petitioner registered its own corporate name.
Therefore, the petitioner’s use of the word was not exclusive, but rather shared with Western Pangasinan Lyceum and later with
other private respondent institutions.

14
CASE DIGEST | Corporation Law (2019) Atty. Villanueva-Castro ib3h
● Lastly, one must evaluate corporate names in their entirety to determine whether it is “identical” or “confusingly similar” with each
other.

15. ANG MGA KAANIB SA IGLESIA NG DIOS KAY KRISTO HESUS, H.S.K. SA BANSANG
PILIPINAS, INC. v. IGLESIA NG DIOS KAY CRISTO HESUS, HALIGI AT SUHAY NG
KATOTOHANAN
G.R. NO. 137592
December 12, 2001

FACTS
● Respondent is a non-stock religious society or corporation registered in 1936.
● Sometime in 1976, Eliseo Soriano and other members disassociated themselves from the respondent and registered in 1977 a non-
stock religious society or corporation, named Iglesia ng Dios kay Kristo Hesus, Haligi at Saligan ng Katotohanan.
● Respondent then filed with the SEC petition to compel the other to change its corporate name, which the SEC ruled in favor of
respondent, and no appeal was made because it appears that the petitioners caused the registration of the petitioner (name in the
title as a whole), the H.S.K. stands for “Haligi at Sandigan ng Katotohanan.”
● Respondent filed before the SEC petition to compel the change its corporate name and bar the same, to which the SEC rendered
decision ordering petitioner to change its corporate name.
● SEC en banc affirmed, and also CA.

ISSUE: Whether or not the names are confusingly similar.

HELD
● Yes. It is the duty of the SEC to prevent confusion in the use of corporate names not only for the protection of the corporations
involved but for the protection of the public.
● Corollary to Sec. 18 of the Corporation Code, SEC Guidelines on Corporate Names states that if the proposed name contains a
word similar to a word already used as part of the firm name of a registered company, the proposed name must contain two other
words different from the name of the company already registered.
● Parties organizing a corporation must choose a name at their peril; and the use of a name similar to one adopted by another
corporation, x x x if misleading x x x, regardless of intent, may be prevented by the corporation having a prior right, by a suit for
injunction against a new corporation to prevent the use of the name.
● The words “Ang Mga Kaanib” and “Sa Bansang Pilipinas” are merely descriptive and only referring to the members of petitioner
residing in the Philippines; hardly serve as an effective medium necessary to avoid confusion or difficulty in distinguishing
petitioner from respondent. It is true most especially both corporations are using the same acronym HSK and are espousing religious
beliefs.
● The difference between the corporate names, saligan and suhay, are synonymous - both referring to “foundation”, or “support”.
● The wholesale appropriation by petitioner of respondent’s corporate name cannot find justification under the generic word rule

15
CASE DIGEST | Corporation Law (2019) Atty. Villanueva-Castro ib3h

16. YOUNG AUTO SUPPLY CO. v. CA


G.R. NO. 104715
June 25, 1993

FACTS
● On October 28, 1987, Young Auto Supply, Co. (YASCO), represented by Nemesio Garcia, its president sold all of their shares of
stock in Consolidated Marketing & Development Corporation (CMDC) to private respondent George Chiong Roxas. Roxas then
took over the four markets of CMDC.
● The purchase price was P8 Million, a down payment of P4 Million and balance payable in postdated checks of P1 Million each.
● There remains a balance of P3.4 Million, which prompted the petitioners to file a complaint against Roxas in Cebu City RTC
praying that Roxas be ordered to pay the remaining sum or that full control of the three markets turned over to YASCO.
● Roxas moved to dismiss after being declared in default on the ground of, among others, that the venue was improperly laid.
● Trial court dismissed the motion to dismiss; CA ordered dismissal on the ground of improper venue.
● Note that the residence of both petitioners and plaintiffs are in CEBU CITY, while it is averred that YASCO’s address is in PASAY
CITY.

ISSUE: Whether the venue was improperly laid

HELD:
● No. In RTC, all personal actions are commenced and tried in the province or city where the defendant or any of the defendants
resides, or where the plaintiff or plaintiffs resides, at the election of the plaintiff. (Sec. 2(b), Rule 4)
● While the Deed of Sale of the shares provide that the address of YASCO is “No. 1708 Dominga Street, Pasay City”, the plaintiffs,
natural and a domestic corporation, aver in the complaints that they are residents of Cebu City, the latter as stated in the Articles of
Incorporation.
● A corporation has no residence in the same sense in which the term is applied to a natural person. However, practically, a
corporation is in a metaphysical sense, a resident of the place where its principal office is located, as stated in the articles of
incorporation.
● The Corporation Code requires each corporation to specify in its articles of incorporation the place where the principal office of
the corporation is to be located which must be within the Philippines (Sec. 14[3]), to fix the residence of a corporation in a definite
place.
● Actions cannot be filed against the corporation in any place where the corporation maintains its branch offices; and the corporation
on the other hand, the corporation cannot be allowed to file personal actions in a place other than its principal place of business.
● The residence of YASCO for purposes of venue is Cebu City.

16
CASE DIGEST | Corporation Law (2019) Atty. Villanueva-Castro ib3h

17. GAMBOA v. TEVES


G.R. NO. 176579
June 28, 2011

FACTS
● Petitioner Wilson Gamboa is a stockholder of PLDT, who filed an instant petition for prohibition, injunction, declaratory relief,
and declaration of nullity of sale of 111,415 shares. In 1969, General Telephone and Electronics Corporation (GTE), an American
company, sold 26% of its outstanding shares of PLDT to PTIC.
● In 1977, Prime Holdings, Inc. (PHI) became the owner of 111,415 shares of stock of PTIC executed by PTIC stockholders Ramon
Cojuangco and Luis Tirso Rivilla.
● These 111,415 shares were sequestered by the PCGG, which represents about 46.125% of outstanding capital stock of PTIC, which
was later declared to be owned by RP.
● The Philippine Government sold these shares with which Parallax Venture Funt XXVII won the bid for P25.6 billion.
● In 1999, First Pacific, a Bermuda-registered, Hong Kong-based investment firm, acquired the remaining 54% of the outstanding
capital stock of PTIC. They announced that they would exercise its right of first refusal as stockholder and buy the 111,415 PTIC
shares with which it bought the same through its subsidiary, MPAH for P25,217,556,000.
● Since PTIC is a stockholder of PLDT, the sale of 46.125% shares of PTIC is an indirect sale of 12 M shares or about 6.3% of the
outstanding common shares of PLDT. With the sale, First Pacific’s common shareholdings in PLDT increased to 37, increasing
the common shareholdings of foreigners in PLDT to 81.47%, which violates the Constitutional limit in Sec. 11, Art. XII of the
1987 Constitution.
● Petitioner claims, that, combined with Japanese NTT DoCoMo’s common shareholdings with PLDT, the total foreign
shareholdings in PLDT is 51.56%, well above the 40% constitutional limit.

ISSUE: Whether the term “capital” in Sec. 11, Art. XII of the Constituion refers to common shares only or to the total outstanding
capital stock of PLDT, a public utility.

HELD:
● Sec. 11, Art. XII of the Constitution mandates the Filipinization of public utilities:
○ “No franchise, certificate, or any other form of authorization for the operation of a public utility shall be granted except to
citizens of the Philippines or to corporations or associations organized under the laws of the Philippines, at least sixty per
centum of whose capital is owned by such citizens x x x
● The “capital” in the said provision refers only to shares of stock entitled to vote in the election of directors, and thus the present
case only to common shares, and not to the total outstanding shares, which comprise of both common and non-voting preferred
shares.
● In the reading of Sec. 6 of the corporation code, it is said that no share may be deprived of voting rights except those classified and
issued as “preferred” or “redeemable” shares, unless otherwise provided in this Code x x x. In the absence of provisions in the
Articles of incorporation denying voting rights to preferred shares, they have the same voting rights as common shares. However,
preferred shareholders are often excluded from any control, that is deprived the right to vote.
● Note that the rights of a stockholder to participate in the control or management of the corporation is exercised through his vote in
the election of directors.
● Therefore, considering that common shares have voting rights which translates into control, then it refers only to common shares.
However, if the preferred shares also have the right to vote in the election of directors, then the term “capital” shall include such
shares.
● Thus, the 60% “capital” assumes, or should result in, “controlling interest” of the corporation. Sec. 3 of the Foreign Investments
Act of 1991 states that:
○ A “Philippine national” is x x x a corporation organized under the laws of the Philippines of which at least 60% of the
capital stock outstanding AND entitled to vote is owned and held by citizens of the Philippines.
● Mere legal title is insufficient to meet the 60% Filipino-owned “capital” required in the Constitution. Full beneficial ownership of
60% of the outstanding capital stock, coupled with 60% of the voting rights is required.

17
CASE DIGEST | Corporation Law (2019) Atty. Villanueva-Castro ib3h

Gamboa v. Teves
In the case at bar,
● common shares: 64.27% foreigners; 35.73% Filipinos
● preferred shares (non-voting): 99.94% Filipino; .56% foreigners
● authorized capital stock: preferred shares - 77.85% preferred; 22.15% common
● Common shares have way more market value than preferred shares.

18. HEIRS GAMBOA v. TEVES


G.R. NO. 176579
October 9, 2012

MOTION FOR RECONSIDERATION

● This is a motion for reconsideration filed on the previous decision (Gamboa v. Teves, June 28, 2011) interpreting the word “capital”
under Sec. 11, Art. XII of the 1987 Constitution

HELD
● There is no redefinition of the term “capital” in the Constitution.
● The Filipinization of public utilities is pursuant to the State policy of development of a national economy effectively controlled by
Filipinos.
● The 60-40 ownership requirement in favor of FIlipino citizens in the Constitution to engage in certain economic activities applies
not only to voting control of the corporation, but also to the beneficial ownership of the corporation. Sec. 11, Art. XII of the
Constitution provides that to own and operate a public utility a corporation’s capital must at least be 60% owned by Philippine
nationals.
● Pursuant to the express mandate of Sec. 11, Art. XII of the 1987 Constitution is Sec. 3, RA 7042 or the Foreign Investments Act
of 1991 which provides that:
○ A “Philippine national” shall mean x x x a corporation organized under the laws of the Philippines of which at least
60% of the capital stock outstanding AND entitled to vote is owned and held by citizens of the Philippines.
● RA 7042 or the Foreign Investments Act of 1991, like its predecessor statutes, is explicit that a domestic corporation is a Philippine
national if at least 60% of the capital stock outstanding and entitled to vote is owned by Filipino citizens.
● Since the 60-percent requirement by the Constitution applies to beneficial ownership of the corporation aside from voting control,
if a corporation, engaged in a partially nationalized industry, issues a mixture of common and preferred non-voting shares, at least
60-percent of the common shares and at least 60-percent of the preferred non-voting shares must be owned by the Filipinos.
● In other words, the 60-40 ownership requirement must apply separately to each class of shares, whether common, preferred non-
voting, preferred voting or any other class of shares.
● In the constitutional mandate that partly nationalized industry must be effectively owned by Filipinos, voting control

18
CASE DIGEST | Corporation Law (2019) Atty. Villanueva-Castro ib3h

19. NARRA NICKEL MINING AND DEVELOPMENT CORP. v. REDMONT CONSOLIDATED


MINES CORP.
G.R. NO. 195580
January 28, 2015

FACTS
● This is a motion for reconsideration filed by petitioner filed on the previous April 21, 2014 decision, which held affirming the CA’s
decision ruling that petitioners, being foreign corporations, are not entitled to Mineral Production Sharing Agreements (MPSAs).
● The Court upheld the CA finding that there was a doubt on petitioner’s nationality since a 100% Canadian-owned firm, MBMI
Resources, Inc. effectively owns 60% of the common stocks of the petitioners by owning equity interes of petitioners’ other
majority corporate shareholders.
● Petitioners claim that the Court’s application of the Grandfather Rule to determine their nationality is erroneous and allegedly
without basis in the Constitution, the Foreign Investments Act of 1991, the Philippine Mining Act of 1995, and the Rules issued
by the SEC

HELD
● The Court used the “Grandfather Rule” as a supplement to the Control test so that the intent underlying the averted Sec. 2, Art. XII
of the Constitution be given effect.
● In the earlier decision the Court says that the control test is still the prevailing mode of determining whether or not a corporation is
a Filipino corporation, and when in the mind of the Court, there is doubt, based on the facts and circumstances in the 60-40 Filipino
equity ownership in the corporation, then it may apply the “grandfather rule.”
● To reiterate, Sec. 2, Art. XII of the Constitution reserves the exploration, development, and utilization of natural resources to
Filipino citizens and “corporations or associations at least sixty per centum of whose capital is owned by such citizens.” And
Philippine Mining Act of 1995 considers a corporation registered in accordance with law at least sixty per cent of the capital of
which is owned by citizens of the Philippines as qualified to undertake a mining corporation.
● Consistent with this objective, the Grandfather Rule was originally conceived to look into the citizenship of the individuals who
ultimately own and control the shares of stock of a corporation for purposes of determining compliance with the constitutional
requirement of Filipino ownership.
● If there are layers of intervening corporation investing in a mining joint venture, the citizenship of the individual stockholders of
each corporation must be delved into.
● The method employed in the Grandfather Rule of attributing the shareholdings of a given corporate shareholder to the second or
even subsequent tier of ownership is due with the rule that the beneficial ownership of corporations engaged in nationalized
activities must reside in the hands of Filipino citizens.
● Hence, the Grandfather Rule must be applied to accurately determine the actual participation, both direct and indirect, of foreigners
in a corporation engaged in a nationalized activity or business.
● The control test, then, can be applied jointly with the Grandfather Rule, and if appropriate, can be used cumulatively in the
determination of the ownership and control of corporations engaged in fully or partly nationalized activities.
● The Grandfather rule, standing alone, should not be used to determine the Filipino ownership and control in a corporation, as it
could result in an otherwise foreign corporation rendered qualified to perform nationalized or partly nationalized activities.
● Hence, it is only when the Control Test is first complied with that the Grandfather Rule may be applied. If Filipino equity falls
below 6%5, then it is considered foreign owned, and Grandfather rule disappears, as it is inapplicable in the first place.

When applied:
● In Tesoro: foreign participation is 59.99%; 40.01% Filipino;
● In McArthur: foreign participation is 59.99%; 40.01% Filipino;
● In Narra: Foreign participation 60.36%; 39.64% Filipino.

Note that petitioners only issued common shares, as it does not indicate it issued any other class of shares besides the 10,000 common
shares.
19
CASE DIGEST | Corporation Law (2019) Atty. Villanueva-Castro ib3h

Narra Nickel v. Redmont


Application of the Grandfather Rule

Tesoro
- Sarah Marie Mining (59.97%)
- Olympic (Filipino) 66.63% (not paid)
- MBMI (Canadian) 33.37% (99% paid)
- MBMI (39.98%)

McArthur
-Madrilejos (59.97%)
- Olympic
- MBMI (same composition as Sarah Marie)
- MBMI (39.98%)

Narra
- Patricia Louise (59.97%)
- PASRDC (Filipino) (0%)
- MBMI (99.75%)
- MBMI - (39.98%)

20
CASE DIGEST | Corporation Law (2019) Atty. Villanueva-Castro ib3h

III. CONTROL AND MANAGEMENT OF A CORPORATION

20. GRACE CHRISTIAN HIGH SCHOOL v. CA


G.R. NO. 108905
October 23, 1997

FACTS
● Petitioner, an educational institution offering preparatory, elementary and secondary courses at the Grace Village in Quezon City
has been recognized as “permanent director” from 1975 to 1989 of the private respondent Grace Village Association, Inc.
● The by-laws of the association adopted in 1968 provides that the elections of Board of Directors, composed by 11 members, shall
take place at the annual meeting of the members by plurality vote and by secret balloting, which shall serve for one year until their
successors are duly elected and qualified.
● An draft of an amendment to the by-laws was prepared by the committee in 1975 which reads that the first fourteen (14) highest
number of votes shall be declared x x x elected and GRACE CHRISTIAN HIGH SCHOOL representative is a permanent Director
of the Association.
● The draft was never presented to the general membership for approval, but from 1975, until 1990, it was given a permanent seat to
the board of directors.
● However, on February 13, 1990, petitioner received notice from the commission on election of the association that they are
“reexamining” the right of the petitioners representative to continue as an unelected member of the board.
● Petitioner therefore brought an action for mandamus before the Home Insurance and Guaranty Corporation (HIGC).
● Respondent association cited the SEC opinion to the effect that the practice of unelected members in the board was contrary to the
existing by-laws of the association and to Sec. 92 of the Corporation Code; that the by-laws registered with the SEC on 1969
remains the prevailing by-laws since the proposed bylaws has not yet been approved by competent authority nor registered with
the SEC.
● HIGC dismissed the petition, citing the SEC opinion; CA affirmed HIGC decision.
● Petitioner claims that it has acquired a vested right to a permanent seat in the board.

ISSUE: Whether the school's representative has the right to sit in the board of directors of Grace Village Association, Inc. as a
permanent member thereof

HELD:
- No. Sec. 28 and 29 of the old Corporation Law, now Sec. 23 of the Corporation Code leave no doubt that the board of directors
must be elected from among the stockholders or members.
- While there may be corporations in which there are unelected members in the board, which were cited by the petitioner, they sit as
ex-officio members, sitting by virtue of and as long as they hold a particular office.
- There is no reason at all for its representative to be given a sear in the board, nor claim a right to such seat by virtue of an office
held.
- Nor can petitioner claim a vested right to sit in the board on the basis of practice. Practice, no matter how long continued, cannot
give rise to any vested right if it is contrary to law.
- Neither can it attain validity through acquiescence because, if it is contrary to law, it is beyond the members of the association to
waive its invalidity; if they would have formally adopted the provision in question, their action would be of no avail because no
provision of tha by-laws can be adopted if it is contrary to law.
- It should be noted that they did not actually implement the provision in question except perhaps insofar as it increased the number
of directors from 11 to 15, but certainly not the allowance of petitioner's representative as an unelected member of the board of
directors. It is more accurate to say that the members merely tolerated petitioner's representative and tolerance cannot be considered
ratification.

21
CASE DIGEST | Corporation Law (2019) Atty. Villanueva-Castro ib3h

21. GOKONGWEI, JR. v. SEC and SAN MIGUEL CORPORATION


G.R. NO. L-45911
April 11, 1979

FACTS
● Petitioner, as stockholder of respondent SMC, filed with the SEC a petition for declaration of nullity of amended by-laws, among
others, against the majority of the members of the Board of Directors and SMC as an unwilling petitioner.
● As a first cause of action, Gokongwei alleged that n 1976, private individual respondents amended the by-laws of the corporation:
○ basing their authority on a resolution of the stockholders adopted in 1961 when the outstanding capital of the
corporation was only P70,139,740, divided into 5,513,974 common shares and 150,000 preferred shares;
○ At the time of amendment (1976), the outstanding shares and paid-up shares totalled 30,127,043, with total par value
of P301,270,430
He contends that the power to amend, modify, repeal, or adopt new by-laws may be delegated to the Board by vote of stockholders
representing ⅔ of subscribed and paid-up capital stock of the corporation; the ⅔ should be computed at the time of the amendment.
The said amendment was based on a 1961 authorization, therefore acted without authority.
● Other cause of action includes, among others, that petitioner alleges that he has all the qualifications to be a director and that as a
substantial stockholder, he had acquired inherent rights in such ownership, such as rights to vote and be voted in the election of
directors.
○ The amendment, he alleges, that in amending the by-laws, respondents purposely disqualified petitioner and
deprived him of the aforementioned vested right.
● Additional cause of action alleges that corporations have no inherent power to disqualify stockholder from being elected as a
director, and, therefore the questioned act is ultra vires and void.

● Respondents denied the allegations, proffering affirmative defenses that the action taken by the Board resulting in the amendments
is valid and legal because the power to “amend, modify, repeal, or adopt new by-laws delegated to the Board from, March 13, 1961
and long prior thereto has never been revoked, withdrawn or otherwise nullified by the stockholders of SMC; that the vote
requirement is determined in relation to the total subscribed capital at the time the delegation of power is made.
● That petitioner is estopped from questioning the amendments on the ground of lack of authority of the board since he failed to
object to other amendments made on the basis of the 1961 authorization
● That the respondent corporation should not be precluded from adopting protective measures to minimize or eliminate situations
where its directors might be tempted to put their personal interests over that of corporation; that the question amendments is a
matter of internal policy and the judgment of the board should not be interfered with.
● That the petitioner was rejected by stockholders in his bid to secure a seat in the Board on the basic issue that petitioner was engaged
in a competitive business and his securing a seat would have subjected respondent corporation to grave disadvantages.

● In 1977, the amendments were ratified.

ISSUE: Whether or not the amended by-laws of SMC disqualifying a competitor from nomination or election to the Board of
Directors of SMC valid and reasonable.

HELD:
- Yes.
- The validity or reasonableness of a by-law is purely a question of law; subject to the limitation that where the reasonableness of a
by-law is a mere matter of judgment, and one upon which reasonable minds must necessarily differ, a court would not be warranted
in substituting its judgment instead of judgment of those who are authorized to make by-laws and who have exercised their authority.

22
CASE DIGEST | Corporation Law (2019) Atty. Villanueva-Castro ib3h
Re: Authority of the corporation to prescribe qualifications of directors expressly conferred by law
- It is recognized that every corporation has the inherent power to adopt by-laws for its internal government, and to regulate the
conduct and prescribe the rights and duties of its members towards itself and among themselves in reference to the management of
its affairs.
- Under Sec. 21 of the (old) Corporation Law (now Sec. 47, Corporation Code), the corporation may prescribe in its by-laws the
qualifications, duties, and compensation of directors, officers, and employees, which necessarily refer to a qualification in addition
to that specified by Sec. 30 of (old) Corporation Law (now Sec. 23, Corporation Code).

Re: Right of the stockholder to be elected director


- There is no vested right of the stockholder to be elected director, in the face of the fact that the law at the time such right as
stockholder was acquired contained the prescription that the corporate charter and the by-laws shall be subject to amendment,
alteration, and modification.
- Any person "who buys stock in a corporation does so with the knowledge that its affairs are dominated by a majority of the
stockholders and that he impliedly contracts that the will of the majority shall govern in all matters within the limits of the act of
incorporation and lawfully enacted by-laws and not forbidden by law."
- The only right of a dissenting minority, in pursuant to amendment of articles of incorporation in Sec. 18 of the (old) Corporation
Law (now Sec. 16, Corporation Code) is to object in writing and demand payment for his share.

Re: Amendment of corporate by-law rendering a stockholder ineligible to be director if he be also a director in a corporation whose
business is in competition with that of the other corporation
- Held as valid; based upon a settled state law in the United States upon the principle that where the director is so employed in the
service of a rival company, he cannot serve both, but must betray one or another.
- This ruling is in pursuant to the allegations that petitioner has control over SMC personally and through two corporations, to wit:
a) John Gokongwei, Jr. - 6,325 shares; b) Universal Robina Corporation - 738,647 shares; and c) CFC Corporation - 658,313 shares;
that Gokongwei is the president and substantial stockholder of URC and CFC, and are engaged in business directly and substantially
competing with allied businesses of SMC.
- The doctrine of corporate opportunity is precisely a recognition by the courts that fiduciary standards could not be upheld where
the fiduciary was acting for two entities with competing interests.
- The questioned amendment of the by-laws is obviously to prevent the creation of an opportunity for an officer or director of SMC,
who is also the officer or owner of a competing corporation, from taking advantage of the information which he acquires as director
to promote his individual or corporate interests to the prejudice of SMC and its stockholders.
- Indeed, access by a competitor to confidential information regarding marketing strategies and pricing policies of SMC would subject
the latter to a competitive disadvantage and unjustly enrich the competitor, which would frustrate the Constitutional mandate and
laws prohibiting combinations in restraint of trade or unfair competition.

- In sum, in the absence of any legal prohibition or overriding public policy, wide latitude may be accorded to the corporation
in adopting measures to protect legitimate corporate interests.
- Where the reasonableness of a by-law is a mere matter of judgment, and upon which reasonable minds must necessarily
differ, a court would not be warranted in substituting its judgment instead of the judgment of those who are authorized to
make by-laws and who have expressed their authority.

23
CASE DIGEST | Corporation Law (2019) Atty. Villanueva-Castro ib3h

22. PEOPLE’S AIRCARGO AND WAREHOUSING v. CA and SANO


G.R. NO. 117847
October 7, 1998

FACTS
● This is a collection case filed by private respondent Stefani Sano against petitioner, a domestic corporation organized to operate a
customs bonded warehouse in Pasay City.
● To obtain a license for the corporation from Bureau of Customs, the corporation president, Antonio Punsalan, Jr., solicited a
proposal from private respondent for the preparation of a feasibility study. The services amounted to P350,000.
● A majority stockholder of petitioner, Cheng Yong, objected to the offer, as another company priced a similar proposal at only
P15,000.
● Nonetheless, the petitioner confirmed their agreement through a letter signed by Punsalan on October 17, 1986 (First Letter),
preferring the private respondent because the respondent is a member of the task force supervising the transition of Customs in
the Aquino administration.
● On December 4, 1986, upon Punsalan’s request, private respondent sent petitioner another letter-proposal with which the total
amount of the services is at P400,000. (Second Letter)
● On January 10, 1987, Andy Villaceren, petitioner’s vice president, received the operations manual prepared by the private
respondent which was submitted to the Bureau of Customs, the result of which the Bureau issued to it a license to operate. a three-
day seminar for the letter’s employees was also conducted by private respondent.
● Private respondent then filed a collection suit against the petitioner because despite demand, it refused to pay him for his services.
Petitioner, in its answer, alleged that the letter-agreement was signed by Punsalan without authority.
● Trial court declared Second Contract unenforceable; CA ruled for its validity, ruling that when the president and corporation
entered into the First Contract, similar to the Second Contract, the petitioner clothed him with apparent authority to enter into the
disputed agreement.

ISSUE: Whether or not the president of petitioner-corporation had apparent authority to bind petitioner to the Second Contract

HELD:
- Yes. The general rule is that, in the absence of authority from the board of directors, no person, not even its officers, can validly bind a corporation.
- Sec. 23 of the corporation code provide that the Board has the power and responsibility to decide whether the corporation should enter into a
contract that will bind the corporation is lodged in the board, subject to the articles of incorporation, bylaws, or relevant provisions of law.
- However, just as natural person may authorize another to do certain acts for and on his behalf, the board of directors may validly delegate some
of its functions and powers to officers, committees, or agents, derived from: 1) law; 2) by-laws; 3) or authorization from the board either expressly
or impliedly by habit, custom, or acquiescence in the general course of business.
- Apparent authority is derived not merely from practice; its existence may be ascertained through:
a) the general manner in which the corporation holds out an officer or agent as having the power to act or, in other words, the apparent
authority to act in general; or
b) acquiescence in his acts of a particular nature, with actual or constructive knowledge thereof, whether within or beyond the scope of his
ordinary powers.
- In the case at bar, Punsalan entered into the First Contract without first securing board approval but the corporation did not object to or repudiate
said contract, thus clothing the president with the power to bind the corporation.
- Therefore, the private respondent cannot be faulted for believing the binding effect of Punsalan’s conformity with the contract to the corporatio.
It is a familiar doctrine that if a corporation knowingly permits one of its officers, or any other agent, to act within the scope of an apparent
authority, it holds him out to the public as possessing the power to do those acts; and thus, the corporation will, as against anyone who has in good
faith dealt with it through such agent be estopped from denying the agent’s authority.
- Lastly, in the absence of a charter or bylaw provision to the contrary, the president is presumed to have the authority to act within the domain of
the general objectives of its business and within the scope of his or her usual duties.
- The corporation also benefited from the contract to hold them in estoppel

24
CASE DIGEST | Corporation Law (2019) Atty. Villanueva-Castro ib3h

23. MARC II MARKETING v. JOSON


G.R. NO. 171993
December 12, 2011

FACTS
● Petitioner Marc II Marketing, Inc. is engaged in buying, marketing, selling, and distributing in retail or wholesale for export or
import household appliances and products and other items, who took over the business operations of Marc Marketing, Inc.
● Petitioner Lucila Joson is the President and majority stockholder of the corporation. On the other hand, respondent Alfredo Joson
was the General Manager, incorporator, director, and stockholder of petitioner corporation.
● Before petitioner corporation was officially incorporated, respondent has been engaged by Lucila to work as General Manager of
the petitioner corporation under the letterhead of Marc Marketing, formalized through a Management Contract. After Marc
Marketing has dissolved and the petitioner corporation was officially incorporated, he continued his duties as General Manager.
● Pursuant to Sec. 1, Article IV of the petitioner’s bylaws, its corporate officers are: 1) Chairman, 2) President, 3) one or more Vice-
President(s), 4) Treasurer, and 5) Secretary. Its Board of Directors, may, from time to time, appoint such other officers as it may
determine to be necessary or proper.
● On June 1997, petitioner corporation decided to stop and cease its operations, and the respondent was apprised of the termination
of his services as General Manager since his services are no longer necessary for the winding up.
● Feeling aggrieved, respondent filed before the NLRC complaint for reinstatement and money claim.
● Notwithstanding the motion by petitioner to dismiss on the ground of lack of jurisdiction as that the case involves an intra-corporate
controversy, the LA held that the position of General Manager is not a corporate officer, and that the dismissal is illegal.
● The NLRC ruled in favor of petitioners, dismissing the complaint for want of jurisdiction, holding that the position is a corporate
officer.
● CA reinstated the decision of the Labor Arbiter.

ISSUE: Whether or not the respondent as General Manager of petitioner corporation is a corporate officer or a mere employee.

HELD:
- Corporate officers are those officers of a corporation who are given that character either by the Corporation Code or by
corporation’s by-laws.
- Sec. 25 of the Corporation Code specifically enumerates the corporate officers, to wit: 1) president; 2) secretary; 3) treasurer, and
4) such other officers as may be provided for in the by-laws.
- Conforming thereto, a position must be expressly mentioned in the bylaws in order to be considered as corporate office. An enabling
provision is not enough to make a position a corporate office; otherwise they could be considered only as employees or subordinate
officials.
- Therefore, corporate officers are exclusively those who are given that character either by the Corporation Code or by the
corporation’s bylaws.

- The Board has no power to create other Offices (other than those enumerated in the bylaws) without amending first the corporate
bylaws.
- A careful perusal of the petitioner’s bylaws show that the position was not among those enumerated; mere board resolution to that
effect cannot make such position a corporate office.
- Unless and until petitioner corporation’s bylaws is amended for the inclusion of General Manager in the list of its corporate officers,
such position cannot be considered as a corporate office within the realm of Sec. 25 of the Corporation Code.
- Though occupying a high-ranking and vital position in petitioner corporation, he can only be regarded as its employee or subordinate
official.
- Since he is merely an employee, NLRC has jurisdiction over his dismissal, not SEC.
- This is to note that Sec. 25 safeguards to constitutionally enshrined right of every employee to security of tenure.

25
CASE DIGEST | Corporation Law (2019) Atty. Villanueva-Castro ib3h

24. SPOUSES DAVID and CGI v. CIAC and SPOUSES QUIAMBAO


G.R. NO. 159795
July 30, 2004

FACTS
● Petitioner Coordinated Group, Inc. (CGI) is a corporation engaged in the construction business, with petitioner-spouses as its
President and Treasurer respectively.
● Respondent spouses engaged the services of the petitioner to design and construct a five-storey concrete office/residential building
on their land in Tondo, Manila.
● Petitioners failed to follow the specifications and plans, hence the respondents rescinded the contract after paying 74.84% of the
cost of construction for failure to follow the specifications and plans, and repeated failure to act on the corrections.
● The new contractor found that petitioners revised and deviated from the structural plan of the building without notice or approval
by the respondents.
● Respondents then filed for breach of contract before the RTC, which was trasnferred to Construction Industry Arbitration
Commission (CIAC) after agreement by the parties.
● CIAC held petitioners jointly and severally liable for the net award amounting to P4,073,229.94.
● Petitioners aver that there is error in finding that they are jointly and severally liable with the corporation, in clear vio lation of the
doctrine of separate juridical personality.

ISSUE: Whether or not petitioner-spouses as corporate officers were grossly negligent in ordering the the revisions on the
construction plan without the knowledge and consent of the respondent-spouses

HELD:
Yes.
- As a general rule, the officers of a corpoation are not personally liable for their official acts unless it is shown that they have
exceeded their authority.
- However, the personal liability of the corporate director, trustee, or officer along with the corporation may so validly attach when
he assents to a patently unlawful act of the corporation or for bad faith or gross negligence in directing its affairs.
- Legal basis: Sec. 31, Corporation Code
- In finding the ruling of CIAC correct when it attributed bad faith on the petitioner, it was admitted that the revision of the plans
was in pursuance of the request of Engr. David to revise the structural plans to provide for a significant reduction of the cost of
construction. Such justification was confirmed by Engr. David, that he wanted to reduce the cost of construction.
- The Court did not find any reason to disprove this fact cited by the CIAC.

26
CASE DIGEST | Corporation Law (2019) Atty. Villanueva-Castro ib3h

25. INTER-ASIA INVESTMENT INDUSTRIES, INC. v. CA


G.R. NO. 125778
June 10, 2003

FACTS
● On September 1978, petitioner corporation sold to private respondent Asia Industries, Inc. for a consideration of P19.5 M all its
right, title, and interest in and to all the outstanding shares of FARMACOR, Inc.
○ The seller made warranties that the Minimum Guaranteed Net Worth of FARMACOR is P12 Million;
○ The agreement further states that the private respondent may retain the P7.5 M; may deduct any shortfall on the
Minimum Guaranteed Net Worth of P12 M, and if the amount retained is not sufficient to make up for the
deficiency in the Minimum Guaranteed Net Worth, then the respondent shall pay the difference.
● The shortfall in the guaranteed net worth amounted to P13.224 M, and therefore the adjusted contract price amounts to P6,225,775.
The respondent then was entitled to a refund of P5,744,225, from the P12 M amount already paid by them.
● Petitioner thereafter, through its president, proposed through a letter on January 24, 1980 to reduce the respondent’s claim for
refund to P4,093,993, and to pay the cost of P759,570 of NOCOSII superstructures which the latter agreed. Petitioner, however,
reneged on the promise.
● The respondent then filed a complaint against petitioner in Makati RTC for recovery of P4,853,503 plus interest.
● The Makati RTC ruled in favor of private respondents; CA affirmed.
● Petitioner argues that the January 24, 1980 letter-proposal signed by its president has no legal force and effect as it was not
authorized by its board of directors.

ISSUE:

HELD:
- The Court in holding that the January 24, 1980 is valid, cited People’s Aircargo v. CA.
- The general rule is that, in the absence of authority from the board of directors, no person, not even its officers, can validly
bind a corporation. (Basis: Sec. 23, Corporation Code)
- However, just as natural person may authorize another to do certain acts for and on his behalf, the board of directors may
validly delegate some of its functions and powers to officers, committees, or agents, derived from: 1) law; 2) by-laws; 3) or
authorization from the board either expressly or impliedly by habit, custom, or acquiescence in the general course of
business.
- A corporate officer or agent may represent and bind the corporation in transactions with third persons to the extent that
the authority to do so has been conferred upon him, and this includes powers as,:
i) in the usual course of the particular business;
ii) incidental to or may be implied from the powers intentionally conferred;
iii) powers added by custom and usage as usually pertaining to the particular officer or agent;
iv) and such apparent powers as the corporation has caused person dealing with the officer or agent.
- Apparent authority is derived not merely from practice; its existence may be ascertained through:
i) the general manner in which the corporation holds ut an officer or agent as having the power to act or, in other
words, the apparent authority to act in general; or
ii) acquiescence in his acts of a particular nature, with actual or constructive knowledge thereof, whether within or
beyond the scope of his ordinary powers.
- In the case at bar, by allowing the president to sign the Agreement on its behalf, petitioner clothed him with apparent capacity to
perform all acts which are expressly, impliedly, and inherently stated therein.

27
CASE DIGEST | Corporation Law (2019) Atty. Villanueva-Castro ib3h

26. MEGAN SUGAR CORPORATION v. ILOILO RTC BR. 68, NEW FRONTIER SUGAR
CORPORATION, EPCIB
G.R. NO. 170352
June 1, 2011

FACTS
● On July 23, 1993, respondent New Frontier Sugar Corporation (NFSC) obtained a loan from Equitable PCI Bank (EPCIB),
mortgaging a 92 hectare land in Passi, Iloilo, and chattel mortgage on the former’s sugar mill.
● In 2000, because of liquidity problems, a MOA was entered, with Central Iloilo Milling Corporation (CIMICO) the latter taking
over the operation of NFSC.
○ NFSC filed a complaint for specific performance and collection against CIMICO for failure to pay its obligations.
● EPCIB instituted an extrajudicial foreclosure against NFSC, where the fomer is the sole bidder. However, CIMICO succeeded in
obtaining a TRO hence continued operations.
● On the other hand, petitioner Megan Sugar Corporation entered with CIMCO in a MOA whereby Megan assumed CIMICO’s
rights, interests, and obligations over the property. Megan then started operating the sugar mill.
● On the other hand, Passi Iloilo Sugar Central, Inc. filed with RTC Motion to Intervention claiming to be the vendee of EPCIB.
● In the hearing of the said Motion, Atty. Reuben Mikhail Sabig appeared before the RTC as counsel for Megan, manifesting that
his statements would bind Megan.
● The RTC thereafter ordered Megan to deposit in escrow within 5 days the sugar quedans representing the miller’s share to the
Court pursuant to the MOA entered by CIMICO and NFSC. Sabig filed an MR.
● Petitioner appealed, arguing that RTC had no jurisdiction over Megan. To which the CA ruled that since Atty. Sabig had actively
participated before the RTC, they are estopped from assailing the RTC’s jurisdiction.

ISSUE: Whether the petitioner is estopped from questioning the assailed orders because of the acts of Atty. Sabig

HELD:
- Yes. A corporation may be held in estoppel from denying as against third persons the authority of its officers or agents who have
been clothed by it with ostensible or apparent authority.
- Apparent authority, or what is sometimes referred to as holding out doctrine, or doctrine of ostensible agency, imposes liability,
not as the result of the reality of the contractual relationship, but rather because of the actions of a principal in somehow misleading
the public into believing that the relationship or the authority exists.
-Megan had all the opportunity to repudiate the authority of Atty. Sabig since all motions, pleadings, and court orders were sent to
Megan’s office but they did not question the acts of Atty. Sabig; moreso, did not assail the authority of Atty. Sabig within the
reasonable time from the moment when the first adverse order was issued.
- While it is true that Atty. Sabig said that he appeared in court only for the hearing for motion for intervention and not on the case
itself, his subsequent acts, coupled with Megan’s inaction and negligence to repudiate his authority, bar Megan from assailing the
validity of RTC proceedings under the principle of estoppel.

28
CASE DIGEST | Corporation Law (2019) Atty. Villanueva-Castro ib3h

IV. CONTROL AND MANAGEMENT OF A CORPORATION

27. NIELSON & COMPANY, INC. v. LEPANTO


G.R. NO. L-21601
December 28, 1968

FACTS
- On January 30, 1937, the parties have entered into an operating agreement wherein Nielson & Co. would operate and manage the
mining properties owned by Lepanto Consolidated Mining Co. for a period of five years.
- Before the lapse of the five year period, the parties have renewed the contract for another five years with modifications made by
Lepanto on the management fee.
- On its modified contract, Nielson will receive 10% of the dividends declared and paid, when and as paid during the period of the
contract and at the end of each year, among others.
- In January, 1942 operation of the mining properties was disrupted on account of the war, and only liberated in 1945.
- After the mines were liberated from the Japanese invaders in 1945, Lepanto embarked a rehabilitation and reconstruction of the
mine, which was not finished until 1948.
- Shortly after the mines were liberated from the Japanese invaders in 1945, a disagreement arose between NIELSON and LEPANTO
over the status of the operating contract in question which as renewed expired in 1947.
- Under the terms thereof, the management contract shall remain in suspense in case fortuitous event or force majeure, such as war
or civil commotion, adversely affects the work of mining and milling.
- Nielson brought an action against LEPANTO before the Court of First Instance of Manila to recover certain sums of money
representing damages allegedly suffered by the former in view of the refusal of the latter to comply with the terms of a management
contract entered into between them on 30 January 1937, including attorney's fees and costs.
- The Supreme Court ruled, in its Decision rendered on December 17, 1966, ordering Lepanto to issue and deliver shares of stock
together with fruits thereof equivalent to 10% of dividends declared, together with cash and stock dividends from 1949 to 1950
- Lepanto seeks reconsideration on the said ruling, among others.

ISSUE: Whether or not Nielson is entitled to its share in the stock dividends.

HELD:
- No. Instead of shares of stock worth P300,000 based on stock dividends, the Court ordered Lepanto to pay Nielson P300,000 in
cash, representing 10% of the money value of the stock dividends declared from 1949 to 1950.
- A reading of Section 16 of the (old) Corporation Law led the Court to rule that stock dividends cannot be issued to a person not a
stockholder in payment of services rendered.
- Sec. 16 actually provides that the consideration for which shares of stock may be issued are:
1) cash; 2) property; and 3) undistributed profits.
- Shares of stock are called “stock dividends” only if they are issued in lieu of undistributed profits. If these are issued in exchange
of cash, or property, then these would not fall under the category of “stock dividends”.
- A corporation may legally issue shares of stock in consideration of services rendered to it by a person not a stockholder, equivalent
to a stock in exchange of property, because services is equivalent to property.
- But a share of stock thus issued should be part of the original capital stock of the corporation, in other words, stocks that are
originally issued by the corporation and forming part of the capital; original shares of stock unsold or unsubscribed.
- This kind of shares may be issued to a person not a stockholder for services rendered, but those share of stock issued
coming from stock dividends declared cannot be issued to one not a stockholder of a corporation
- Thus, stock dividends are issued only to stockholders; stock dividends are civil fruits of the original investment, and to the owner
of the shares belong the civil fruits.
- Nielson, therefore, (not a stockholder), cannot be paid in shares of stock which form part of stock dividends of Lepanto for services
rendered under the management contract.

29
CASE DIGEST | Corporation Law (2019) Atty. Villanueva-Castro ib3h

28. ISLAMIC DIRECTORATE OF THE PHILIPPINES v. CA


G.R. NO. 117897
May 14, 1997

FACTS
● Petitioner IDP-Tamano Group alleges that in 1971, the Islamic leaders of all Muslim tribal groups organized and incorporated the
petitioner corporation, the primary purpose of which is to establish an Islamic Center in Quezon City, for the establishment of a
Mosque, Madrasah, and other religious infrastructures.
● The Libyan government donated money to IDP to purchase a land at Culiat, Tandang Sora, Quezon City.
● However, in 1972, most of the 1971 Board of Trustees flew to the Middle East to escape political persecution at the onset of the
declaration of Martial Law.
● Thereafter, two groups sprung claiming to be the legitimate IDP: the Carpizo Group, and the Abbas Group. In a suit before the
SEC between these contending groups (SEC Case No. 2687), it ruled on October 3, 1986 declaring as null and void the election of
both. No valid election was ever held.
● Thereafter, in April 20, 1989, the Carpizo Group caused to be signed an alleged Board Resolution of the IDP authorizing the sale
of the subject two parcels of land to the private respondent Iglesia ni Cristo for a consideration of P22,343,000.
● Petitioner IDP-Tamano, headed by former Senator Mamintal Tamano, filed a petition before the SEC seeking to declare null and
void the Deed on ground that the Capizo group was not the legitimate Board of Trustees. (SEC Case No. 4012)
● INC, on the other hand filed an action for Specific Performance with Damages against the vendor to compel complete delivery and
physical possession of the property.
● In this case, IDP-Tamano sought to intervene, alleging that they are the duly constituted representatives of IDP, to which the RTC
denied the motion on grounds of lack of juridical personality.
● RTC ruled INC as the rightful owner of the properties; while the SEC, in SEC Case No. 4012 ruled that the sale as null and vo id,
in view that the election of Board of Directors from 1986 to 1991 is void.
● INC appealed the SEC decision, to which the CA set aside the SEC decision.

ISSUE: Whether or not CA committed a reversible error in setting aside the SEC decision declaring the sale of land to private
respondent INC null and void

HELD:
- Yes.
- SEC made an unequivocal finding that the IDP-Carpizo Group is a bogus Board of Trustees, consequently, they are bereft of any
authority whatsoever to bind IDP in any kind of transaction, including the sale or disposition of IDP property.
- Note that if SEC can declare who is the legitimate Board, then it can declare who is not the legitimate Board.

- Furthermore, the sale is further deemed null and void ab initio for failure to comply with Sec. 40 of the Corporation on sale of all
or substantially all of corporate property which requires:
a) Majority vote of its board of directors and trustees; and
b) authorized by vote of the stockholders representing ⅔ of outstanding capital stock or ⅔ of members.
- The said provision applies because the property sought to be sold constitutes the only property of IDP.
- The twin requirement were not met as the Carpizo Group constitutes a fake Board of Trustees; and those whose names and signatures
affixed with the sham Board Resolution were not bona fide members as they were made to appear to be.

30
CASE DIGEST | Corporation Law (2019) Atty. Villanueva-Castro ib3h

29. LOPEZ DEE v. SEC


G.R. NO. 60502
July 16, 1991

FACTS
● Private respondent Naga Telephone Company, Inc. was organized in 1954, which saw its increase of its authorized capital stock
from P100,000 to P3,000,000.
● BOC approved the said increase in January 1975 with the condition that no further issues will be made after a year from approval
without previous authority from the Board.
● In 1977, Natelco entered a contract with Communication Services, Inc. (CSI) for manufacture, supply, delivery, and installation of
telephone equipment. In accordance to this contract, Natelco issued 24,000 common stocks to CSI; in 1979, another 12,000 shares
were issued to CSI. No authorization from the BOC was sought pursuant to the imposed condition by the BOC (now NTC)
● In the 1979 elections for Board of Directors of Natelco, petitioner Pedro Lopez Dee was unseated as the chairman but remained as
a director; CSI gained control of Natelco when its legal counsel, Atty. Luciano Maggay won a seat in the Board, and later on
became president.
● During the tenure of the Maggay Board (June 22, 1979 to March 10, 1980), it entered into a contract with CSI for supply and
installation of additional equipment but also issued a total of 113,800 common shares (including the previously issued 36,000
shares)
● Petitioner filed before the SEC (SEC Case No. 1748) questioning the validity of the elections upon ground that no valid list of
stockholders through which the right to vote could be determined.
○ The SEC held in this case that CSI is a stockholder and entitled to vote; the unexplained shares are not entitled to
vote; and 82 stockholders shall be allowed to vote. It ordered holding of special stockholder’s meeting based on
this findings.
○ The issue of the 113,800 shares (excluding the first 36,000) was made by the Maggay Board during the pendency
of SEC Case No. 1748.
● In another case, (G.R. No. 63922) petitioner Antonio Villasenor filed a case before the Naga CFI claiming that he was an assignee
of an option to repurchase 36,000 common stocks of Natelco under Deed of Assignment; in which the defendants, principally the
Maggay group, allegedly refused to allow the repurchase of said stocks by tendering payment of its price.

ISSUES:
WN stockholders have the right to preemption to the 113,800 shares in question

HELD:
- No. The Court held that there is no pre-emptive right of Natelco violated upon issuance of the 113,800 shares.
- The Court, in arriving this decision, cited Benito v. SEC which it held that:
- The general rule is that pre-emptive right is recognized only with respect to new issues of shares, and not with respect to
additional issues of originally authorized shares. This is based on the theory that when the corporation, at its inception,
offers its first shares, it is presumed to have offered all of those which it is authorized to issue. And when the shares left
unsubscribed are later reoffered, an original subscriber cannot claim dilution of interest.
- The power to issue shares of stocks in a corporation is lodged in the board of directors and no stockholders meeting is required to
consider it because additional issuance of shares of stocks does not need approval of the stockholders. Therefore, even if it was made
without notice to stockholders, it remains valid.

31
CASE DIGEST | Corporation Law (2019) Atty. Villanueva-Castro ib3h

30. JIAO, ET. AL. v. NLRC


G.R. NO. 182331
April 18, 2012

FACTS
● Petitioners were regular employees of Philippine Banking Corporation, each with at least ten year of service in the company.
● Philbank, in 1970, established a Gratuity Pay Plan, providing that the gratuity pay of an employee shall be an amount equivalent
to one month salary for every year of credited service, and an employee with credited service of 10 years or more shall be ent itled
to a gratuity of full amount of the gratuity pay but in no case shall exceed the equivalent of 24 months salary.
● In 1991, Philbank implemented a new Gratuity Pay Plan which says that an employee separated by reason of any cause not due to
his own fault, including redundancy, shall be entitled to either 100% of his accrued gratuity benefit or the actual benefit due him
under the Plan, whichever is higher.
● In 2000, Philbank merged with Global Business Bank, Inc. in which the former becomes the surviving corporation but under the
name of the latter.
● As a result of the merger, the complainants’ positions became redundant, and a special separation program (SSP) wherein they
were granted separation package equivalent to 150% of monthly pay for every year of service based on their current salary. They
were asked to sign an Acceptance Letter and a Quitclaim to avail the package.
● Private respondent Metrobank acquired Globalbank’s assets and liabilities in 2002.
● Subsequently, petitioners filed separate complaints for non-payment of separation pay with damages before NLRC.
● Metrobank denied any liability, arguing that its acquisition of its assets and liabilities of Globalbank did not include the latter’s
obligation to its employees. Therefore, there is an absence of employment relationship with the petitioners.
● LA absolved Metrobank from liability, and dismissed the complaints; NLRC affirmed the LA’s decision. CA also dismissed the
appeal

ISSUE: Whether or not Metrobank be held liable for the petitioners’ claim

HELD
- No
- As a rule, corporation that purchases the assets of another will not be liable for the debts of the selling corporation, provided the
former acted in good faith and paid adequate consideration for such assets, except:
1) where the purchaser expressly or impliedly agrees to assume the debts;
2) where the transaction amounts to a consolidation or merger of the corporations;
3) where the purchasing corporation is merely a continuation of the selling corporation; and
4) where the selling corporation fraudulently enters into the transaction to escape liability for those debts.
- Under the Deed of Assignments of Assets and Assumption of Liabilities, the liabilities assumed by Metrobank are characterized as
those pertaining to Globalbank’s banking operations, and does not include liabilities to pay separation pay to its former employees.
- Moreover, it is understood that the liabilities ended when they were paid the amounts embodied in their respective acceptance letters
and quitclaims, hence, it could have not been said to have passed to Metrobank.
- At this point, Globalbank has a separate and distinct juridical personality and the case at bar would not warrant the piercing of
corporate fiction.
- Note: No merger between Metrobank and Globalbank

32
CASE DIGEST | Corporation Law (2019) Atty. Villanueva-Castro ib3h

31. LOYOLA GRAND VILLAS HOMEOWNERS (SOUTH) ASSOCIATION, INC. v. CA


G.R. NO. 117188
August 7, 1997

FACTS
● Loyola Grand Villas Homeowners Association (LGVHAI) was organized in February 8, 1983 as the sole association of
homeowners and residents of the Loyola Grand Villas, registered with the HIGC. For unknown reasons, however, LGVHAI did
not file its corporate by-laws.
● Sometime in 1988, the officers of LGVHAI tried to register its by-laws, but failed to do so.
● Thereafter, the officers discovered that there were two other organizations within the subdivision: the North and South Association.
The North Association was registered with the HIGC on February 13, 1989, submitted its by-laws on December 20, 1988.
● In July 1989, then LGVHAI president Victorio Soliven, upon inquiry to HIGC, was informed that LGVHAI was automatically
dissolved because it did not submit its by-laws, among others. This information led to the registration of the South association with
the HIGC on July 27, 1989, its by-laws submitted on July 26, 1989.
● This prompted the LGVHAI officers to lodge a complaint, questioning the revocation of registration without notice and
concomitantly praying for the cancellation of registration of both North and South associations. The HIGC Hearing Officer ruled
in favor of LGVHAI, and consequently revoking the registration of both North and South Associations.
● Petitioner LGVHAI South appealed to the Appeal Board of HIGC, which dismissed the same; CA affirmed.
● Before the Supreme Court, petitioner contends that Sec. 46 uses the word “must” with respect to the filing of by-laws,
noncompliance thereof would result in “self-extinction”; further asserts that Sec. 22 mandates the corporation to formally organize
within two years from transaction otherwise, they are deemed dissolved.
○ NB: First sentence of Sec. 46 states that “Every corporation formed under this Code must, within one month after receipt of
official notice of the issuance of certificate of incorporation by the Securities and Exchange Commission, adopt a code of
by-laws for its government not inconsistent with this Code.

ISSUE: Whether or not LGVHAI’s failure to file its by-laws within the period prescribed had the effect of automatically dissolving
the said corporation.

HELD:
- No. The automatic corporate dissolution for failure to file the by-laws on time was never the intention of the legislature. Sec. 46
reveals the intent, therefore, to attach a directory, and not mandatory meaning for the word “must” in the first sentence thereof.
Because there may be instances where the by-laws are submitted prior to incorporation. Failure to file by-laws within the period does
not imply the “demise” of the corporation.
- By-laws may be necessary for the “government” of the corporation but these are subordinate to the articles of incorporation as well
as to the Corporation Code and related statutes.
- By-laws are indispensable to corporations in this jurisdiction, as it is the “rules and regulations or private laws enacted by the
corporation to regulate, govern and control its own actions, affairs, and concerns and its stockholders or members and directors and
officers with relation thereto and among themselves in their relation to it,” that though not essential to corporate birth, but certainly,
required by law for an orderly governance and management of corporations.
- Nonetheless, failure to file them within the period required by law by no means tolls the automatic dissolution of a corporation.
- This is so notwithstanding the provision of Sec. 22 of the Corporation Code providing that the powers of corporation would cease
if it did not formally organize and commence the transaction of its business or the continuation of its works within two years from
date of inception.
- While Sec. 6(l)(5), PD 902-A grants the SEC the power to revoke or suspend certificate of registration on grounds of failure to file
by-laws within the required period, there can be no automatic corporate dissolution thereto; it can only do so after the compliance
of twin requirements of due process, proper notice and hearing.
- Therefore, it is clear that failure to file bylaws within the required period is only a ground for suspension or revocation of the
certificate of registration.

33
CASE DIGEST | Corporation Law (2019) Atty. Villanueva-Castro ib3h

32. CHINA BANKING CORPORATION v. CA


G.R. NO. 117604
March 26, 1997

FACTS
● Galicano Calapatia, a stockholder of private respondent Valley Golf & Country Club, Inc. (VGCCI), pledged his stock certificate
to petitioner CBC.
● In virtue thereof, petitioner wrote to VGCCI to record the pledge to the latter’s books, to which it duly noted it.
● Thereafter, Calapatia further loaned P20,000 from petitioner secured with the same pledged stock certificate.
● Due to failure of Calapatia to pay, petitioner filed for extrajudicial foreclosure. Thereafter, petitioner wrote a letter to VGCCI
informing of the foreclosure proceedings and requested that the pledged stock be transferred to its name and recorded in the books.
VGCCI expressed its inability to accede to the request in view of Calapatia’s unsettled accounts with the club. Nonetheless, the
public auction ensued in favor of petitioners.
● For failure of Calapatia to pay his overdue account with the VGCCI, VGCCI caused to be published notice of auction sale of his
stock certificate, among others.
● Petitioner then advised VGCCI of the fact that it is the new owner of the stock certificate and requested a new certificate to be
issued in its name. Nonetheless, the sale ensued and it was sold at the public auction.
● Petitioner protested the sale and hence filed with the Makati RTC for nullification of the auction.
● RTC dismissed the complaint for lack of jurisdiction as it held to be involving intra-corporate affairs.
● SEC Hearing officer ruled in favor of VGGCI; SEC en banc reversed the decision. CA disregarded both the ruling of SEC hearing
holding that the controversy involves an intra-corporate dispute.
● VGCCI anchors its right over the share on Sec. 3, Art. VIII of its by-laws which provides that “after a member shall have been
posted as delinquent, the Board may order his/her/its share sold to satisfy the claims of the Club…”p

ISSUE: Whether or not CBC is bound by the bylaws of VGCCI.

HELD:
- No. The general rule is that third persons are not bound by the by-laws of a corporation. The exception is that when third persons
have actual or constructive knowledge of the same.
- In order to be bound, the third party must have acquired knowledge of the pertinent by-laws at the time the transaction between the
third party and the shareholder was entered into.
- VGCCI could have informed petitioner of its by-laws when it sent the notice recognizing the petitioner as a pledgee; belated notice
will not suffice.

The ruling of SEC en banc is particularly instructive:


- By-laws signifies the rules and regulations or private laws enacted by the corporation to regulate, govern, and control its own
actions, affairs, and concerns and its stockholders or members and directors and officers with relation thereto and among themselves
in their relation to it.
- They are self-imposed and, although pursuant to statutory authority, have no status as a public law.
- Knowledge must of the provisions of the VGCCI by-laws must be acquired at the time the pledge agreement was contracted.
- Neither the postulate that the petitioner is duty-bound to know the by-laws is part of diligence of a good father of a family fails to
convince.

34
CASE DIGEST | Corporation Law (2019) Atty. Villanueva-Castro ib3h

33. ASSOCIATED BANK v. CA


G.R. NO. 123793
June 29, 1998

FACTS
● On September 16, 1975, Associated Banking Corporation and Citizens Bank and Trust Company (CBTC) merged to form
Associated Citizens Bank, which would be renamed to Associated Bank.
● On September 7, 1977 the defendant Lorenzo Sarmiento, Jr executed in favor of Associated Bank a promissory note whereby he
undertook to pay the latter the sum of P2,500,000 payable on March 6, 1978. However, the defendant still owes P2,250,000
exclusive of interest and other damages, which he failed to pay.
● The trial court ordered to pay the bank the remaining balance plus interests;
● CA held that Associated Bank has no cause of action against Sarmiento because they is not privy to the transaction entered by
Sarmiento and CBTC. Further held that the merger could not have vested Associated Bank with any interest arising from the
promissory note executed in favor of CBTC after the merger.

ISSUE: Whether or not Associated Bank, the surviving corporation, may enforce the promissory note made by private respondent
in favor of CBTC, the absorbed company, after the merger agreement had been signed.

HELD:
- Yes.
- In the merger of two or more existing corporations, one of the combining corporations survives and continues the combined
business, while the rest are dissolved and all their rights, properties, and liabilities are acquired by the surviving corporation.
- There is no winding up of their affairs or liquidation of their assets, because the surviving corporation automatically
acquires all their rights, privileges, and powers, as well as their liabilities.
- There merger, however, does not become effective upon the mere agreement of the constituent corporations. it shall only be effective
upon the issuance by the SEC of a certificate or merger. The effectivity date is crucial for determining when its rights, privileges,
properties, as well as liabilities pass on to the surviving corporation.
- Note that the Agreement was made September 16, 1975; the record does not show when the SEC approved the same. Nonetheless,
the private respondent’s theory is that it took effect on the date of the execution of the agreement (September 16, 1975).

- Notwithstanding these uncertain dates, and assuming the effectivity date of the merger was the date of its execution, the agreement
clearly provides that all contracts, irrespective of the date of execution, entered into the name of CBTC shall be understood as
pertaining to the surviving bank, the petitioner.
- Therefore, no distinction should be made.

35
CASE DIGEST | Corporation Law (2019) Atty. Villanueva-Castro ib3h

34. BABST v. CA
G.R. NO. 99398
January 26, 2001

FACTS
● The complaint was commenced principally to enforce payment of promissory note and three letters of credit Elizalde Steel
Consolidated, Inc. (ELISCON) executed and opened with Commercial Bank and Trust Company (CBTC)
● ELISCON obtained from CBTC a loan of P8,015,900.84 evidenced by a promissory note, in which the former defaulted with
remaining payment of P2,795,240.67.
● The letters of credit, on the other hand were opened by ELISCON by CBTC using the credit facilities of Pacific Multi-Commercial
Corporation (MULTI); and subsequently, Antonio Roxas Chua and petitioner Chester Babst executed a Continuing suretyship
binding themselves jointly and severally liable to pay any existing indebtedness of MULTI to CBTC, with an extent of P8M each.
● In December 1980, BPI and CBTC entered into a merger, wherein BPI is the surviving corporation acquiring all the assets and
liabilities of CBTC. DBP took over ELISCON for failure to settle indebtedness.
● BPI then, as successor-in-interest of CBTC, instituted before Makati RTC a complaint for sum of money against ELISCON,
MULTI, and Babst.
● RTC held ELISCON liable, jointly and severally with MULTI and Babst pay BPI; CA affirmed the same, that BPI be paid.

ISSUE: Whether or not BPI has a valid cause of action

HELD:
- Yes.
- It is undisputed, as there is no question, that there was a valid merger between BPI and CBTC.
- It is settled that in the merger of two existing corporations, one of the corporations survives and continues the business, while the
other is dissolved and all its rights, properties, and liabilities are acquired by the surviving corporation.

36
CASE DIGEST | Corporation Law (2019) Atty. Villanueva-Castro ib3h

35. MINDANAO SAVINGS AND LOAN ASSOCIATION, INC. v. WILLKOM


G.R. NO. 178618
October 20, 2010

FACTS
● First Iligan Savings and Loan Association (FISLAI) and Davao Savings and Loan Association, Inc. (DSLAI) entered into a merger
in 1985 with DSLAI as the surviving corporation, but the articles of merger were not accepted by the SEC due to incomplete
documentation.
● On August 12, 1985, DSLAI changed its corporate name to MSLAI by way of amendment to its Articles of Incorporation.
● In 1990, because the business of MSLAI has failed, the Monetary Board ordered its closure and placed it in receivership on August
31, 1990.
● Private respondent Remedios Uy filed with the RTC Iligan City prior to the closure of MSLAI for a collection of money against
FISLAI, to which the RTC issued a summary decision in favor of Uy.
● On April 28, 1993, the sheriff levied on 6 parcels of land owned by FISLAI in Cagayan De Oro, in which Edward Willkom became
the highest bidder in the public auction thereof.
● MSLAI, represented by PDIC, filed before the Cagayan de Oro RTC a complaint for Annulment of Sale, Cancellation of Title and
Reconveyance of Properties against Respondents because it alleged that the sale was made without notice to PDIC, and that the
previous RTC decision was illegal and contrary to law because the assets of an institution placed under receivership such as MSLAI
should be deemed in custodia legis and should be exempt from any other order of levy, execution, etc.
● Respondents averred that MSLAI had no cause of action against them because MSLAI is a separate and distinct entity from FISLAI;
and the unofficial merger between FISLAI and DSLAI (now MSLAI) did not take effect because it did not comply with the
requirements prescribed by the Corporation COde.
● RTC dismissed the case for lack of juristiction; MSLAI failed to obtain favorable decision in CA, ruling that there was no merger.

ISSUE: Whether or not the merger between FISLAI and DSLAI (now MSLAI) valid and effective

HELD:
- No.
- In the merger of two or more existing corporations, one of the corporations survives and continues the combined business, while
the rest are dissolved and all their rights, properties, and liabilities are acquired by the surviving corporation.
- The merger, however, does not become effective upon the mere agreement of the constituent corporations. Since a merger or
consolidation involves fundamental changes in the corporation, as well as in the rights of stockholders and creditors, there must be
an express provision of law authorizing them. (Bases: Sec. 76-79, Corporation Code)
- The merger shall only be effective upon the issuance of a certificate of merger by the SECm subject to its prior determination that
the merger is not inconsistent with the Corporation Code or existing laws.
- In the case at bar, the articles of merger between FISLAI and DSLAI were not registered with the SEC due to incomplete
documentation. Though recognized by the Monetary Board, the fact remains that no certificate was issued by the SEC, thus,
incomplete.
- The issuance of the certificate of merger is crucial because not only does it bear out SEC’s approval but it also marks the moment
the consequences of a merger take place.
- Absorbed corporation ceases to exists;
- But its rights and properties, and liabilities, shall be taken and deemed transferred to and vested in the surviving
corporation.
- There being no merger between FISLAI and DSLAI, two corporations shall not be considered as one but two separate corporations.
It has personality separate and distinct from the persons composing it.
- Thus, the assets of FISLAI remain as its assets and cannot be considered as belonging to DSLAI and MSLAI.

37
CASE DIGEST | Corporation Law (2019) Atty. Villanueva-Castro ib3h

IV. ADDITIONAL CASES

36. BERNAS v. CINCO CINCO v. BERNAS


G.R. NO. 163356-57 G.R. NO. 163368-69
July 1, 2015

FACTS
● Petitioners herein, Bernas Group, were among the Board of Directors and Officers of the Makati Sports Club (MSC) whose terms
were about to expire either in 1998 or 1999.
● Respondents herein, (petitioners in GR 163368-69) were elected members of the Board of Directors and Officers of the club
during the 17 December 1997 Special Stockholders Meeting.
● The subject special stockholders meeting was due to the alarm of the MSC Oversight Committee (MSCOC) on the rumored
anomalies in handling the corporate funds, demanding that the Bernas Group resign from their positions, as resonating the
stockholders of the corporation representing at least 100 shares.
● Pursuant to such request, the MSCOC called the Special Stockholders Meeting and sent to all stockholders and members.
● During the meeting, the Bernas Group was unseated, and the Cinco group was elected.
● Bernas Group seeked nullification of the 17 December 1997 Special Stockholder’s Meeting before the Securities Investigation
and Clearing Department (SICD) of SEC .
● Meanwhile, the new directors found Bernas guilty of irregularities after investigation and resolved to expel him from the club by
selling his shares at public auction.
● Prior to the resolution of the case before the SEC, an Annual Stockholder’s Meeting was held on 20 April 1998 with which the
majority ratified the 1997 Special meeting and all acts and resolutions adopted therein.
● The 1999 Annual Stockholders Meeting was supervised by SEC due to filing of several petitions for and against the removal of
Bernas Group with which the stockholders ratified the 1997 Special meeting.
● It was again ratified during the 2000 Annual Stockholders Meeting.

● The SICD rendered a decsion holding that the 1997, 1998, 1999, meetings are invalid, and the expulsion of Bernas and sale of
his share are likewise null.
● The SEC en banc reversed the findings of SICD;
● CA declared the 1997 Special meeting invalid, while affirmed the actions taken in the 1998, 1999, and 2000 meetings.

ISSUE: WN the 1997 Special Meeting, and subsequent meetings, are valid

HELD:
- Partly yes; the Court resolved that the December 17, 1997 Special Stockholders Meeting is invalid, as well as the resolution
expelling Bernas from the club and the sale of his shares; but the Annual Stockholders Meeting of 1998, 1999, and 2000 are valid,
except the ratification of the expulsion and sale of shares of bernas.

- Sec. 28 of the Corporation Code provides that a special meeting of the stockholders or members of a corporation for the removal
of directors or trustees, or any of them, must be called by the secretary on order of the president, or on written demand of the
stockholders reresenting at least majority of the outstanding capital stock x x x.
- Corollarily, the pertinent provision of MSC by-laws provide that only the President and the Board of Directors are authorized to
call a special meeting; but if persons authorized refused to call a meting, then the stockholders representing at least 100 shares, upon
written request may file a petition to call a special stockholders meeting.
- In the instant case, there is no dispute that the 17 December 1997 meeting was called by the MSCOC, neither the president, nor by
the Board of Directors.
- Nowhere in the Corporation Code nor in the by-laws can it be gathered that the MSCOC is authorized to step in whenever there is
a beach of fiduciary duty and call a special meeting even if it be made upon request of the shareholders.

38
CASE DIGEST | Corporation Law (2019) Atty. Villanueva-Castro ib3h
- Since the MSCOC is not empowered to call such, the defect goes into the very authority of persons who made the call for the
meeting. Therefore, void, and cannot be ratified by subsequent Annual Stockholders’ Meeting.
- The Cinco group then cannot invoke the application of de facto ownership to justify the actions taken after the invalid election
because this is only limited to third persons who were originally not part pf the corporation but became such by reason of government
sequestered shares (Cojuangco, Jr. v. Roxas); neither they have colorable authority to authorize the removal of Bernas and the sale
of his shares, and the buyer cannot be deemed as buyer in good faith.

- By-laws are part of the fundamental law of the corporation with which the corporation and its directors must comply. Certainly, the
rules set in the by-laws are mandatory for every member of the corporation to respect.
- The 1998 Annual Stockholders Meeting was valid, sanctioned by Sec. 8 of MSC by-laws. (Sec. 8 provides for the annual meeting).
Sec. 8 do not provide for any requirement on who will call such meeting.
- The 1999 Meeting is likewise valid because it is also done in accordance to Section 8, but also the meeting was supervised by the
SEC, with which it gave rise to the presumption that those who won the election are duly elected to their positions and can therefore
be considered as de jure officers. The 2000 Annual Stockholders Meeting is likewise valid for the same reason as that in 1998.
- At this point, Bernas cannot invoke the hold-over principle as a shield to perpetuate themselves to office.

39
CASE DIGEST | Corporation Law (2019) Atty. Villanueva-Castro ib3h

37. Y-I LEISURE PHILIPPINES, INC. v. YU


G.R. NO. 207161
September 8, 2015

FACTS
● Mt. Arayat Development Co., Inc. (MADCI) was a real estate development corporation which, sometime in 1997, offered for sale
shares of a golf and country club in the vicinity of Mt. Arayat.
● Respondent James Yu bought 500 golf and 150 country club shares for P650,000.
● After payment, however, Yu discovered that the supposed golf and country club is inexistent. He was not able to receive refund
from MADCI.
● This prompted Yu to file with the RTC a complaint for collection of sum of money and damages against MADCI and its president
Rogelio Sangil.
● Before RTC, MADCI invoked the MOA between MADCI, Sangil, and petitioner Yats International, Inc. (YIL) that Sangil
undertook to redeem MADCI proprietary shares sold to third persons or settle in full all their claims for refund of payments. Hence,
MADCI averred that Sangil should be ultimately liable to refund payments.
● Yu thereafter impleaded YIL, YILPI, and YICRI upon discovery in the Registry of Deeds of Pampanga that substantially, all the
assets of MADCI, consisting of 120 hectares of land in Magalang, Pampanga, were sold to YIL, YILPI, and YICRI.
● The RTC held that MADCI and Sangil are solidarily liable, while exonerating YIL, YILPI, and YICRI. CA partly granted the
appeals, holding YIL, YILPI, and YICRI liable as well.

ISSUE: Whether or not petitioners, as transferees of all assets of MADCI, shall be liable to respondent.

HELD:
- Yes.
- The Nell doctrine provides for the general rule that the transfer of all the assets of a corporation to another shall not render the latter
liable to the liabilities of the transferor. The exceptions are: a) where the purchaser expressly or impliedly agrees to assume debts; b)
where the transaction amounts to a merger or consolidation of the corporations;
c) where the purchasing corporation is merely a continuation of the selling corporation, and d) where the transaction is entered into
fraudulently
- Of these exceptions, the case at bar bears discussion of the third exception, which Dean Cesar Villanueva contemplates as the
“business-enterprise transfer”, wherein the transferee’s corporation interest goes beyond the assets of the transferor’s assets and its
desires to acquire the latter’s business enterprise, including its goodwill.
- As a result of such sale, the transferor is merely left with its juridical existence, devoid of its industry and earning capacity. Sec. 40
of the Corporation Code contemplates this exception.
- Given that the transferee corporation acquired not only the assets, but also the business of the transferor corporation, then the
liabilities of the latter are inevitably assigned to the former.
- Fraud is not essential element for the application of the business-enterprise transfer; the transferee corporation may inherit the
liabilities of the transferor despite the lack of fraud due to the continuity of the latter’s business.
- The purpose of the business-enterprise transfer is to protect the creditors of the business by allowing them a remedy against the
new owner of the assets and business enterprise.
- As CA found out that MADCI had an entire asset of 120 hectares of land, its sale to petitioners rendered it incapable of continuing
its business. In this case, Sec. 40 must apply, and should have complied with.
- The petitioners have consequently inherited the liabilities of MADCI because they acquired all the assets of the latter.

40
CASE DIGEST | Corporation Law (2019) Atty. Villanueva-Castro ib3h

38. TOM v. RODRIGUEZ


G.R. NO. 215764
July 6, 2015

FACTS
● Golden Dragon International Terminals, Inc. (GDITI) is the exclusive shore reception facility service provider of Philippine Ports
Authority.
● Sometime in December 2008, Fidel Cu sold his 17,237 shares in GDITI to Virgilio Ramos, and Cirilo Basalo Jr.
● When the latter failed to pay the purchase price, Cu sold 15,233 shares to Edgar Lim, Eddie Ong, and Arnold Gunnacao, who also
did not pay the consideration.
● In 2009, Lim was elected President, Basalo as VP for VisMin, Ong as Treasurer and VP for Luzon, and Gunnacao as Director.
However, a group led by Ramos, including petitioner Tom, forcibly took over GDITI offices and performed functions as officers.
● While president Lim has an injunction case before Manila RTC because of the taking over, Cu resold his shares to Basalo, including
the shares subject of the previous sales made by Cu. Cu then seeked intervention in the injunction case, averring that he is still the
legal owner of the shares subject of the previous contracts with Ramos, Lim, Ong, and Gunnacao.

● The RTC Manila granted the intervention, directing Ramos et al to cease and desist from performing all acts of control and
management of GDITI, placing the authority to Cu;
● Thereafter, Cu executed an SPA to Cezar Mancao II as an authorized representative to exercise powers granted by the previous
RTC Order, as well as to perform acts of management and control over GDITI. Cu and Basalo also entered an Addendum to the
Agreement for the terms of payment of the shares of stock sold.
● Cu subsequently revoked the SPA, effectively reinstating the power of control to himself.
● This prompted Mancao and Basalo to file a complaint for Specific Performance with TRO and injunction in Nabunturan RTC
against Cu, and impleaded Tom on the allegation that Cu authorized him to exercise control and management over GDITI, to the
exclusion of other agents of GDITI
● On the other hand, Rodriguez, respondent, filed a complaint-in-intevention, alleging that Basalo authorized him through a MOA
to manage and control the operations of GDITI in Luzon area, with which the latter refused to honor the terms and conditions of
the MOA.
● The RTC Nabunturan found credence on the MOA, ordering Basalo to place Rodriguez as representative of Basalo.
● Tom filed for MR, then subsequently an appeal to CA. CA denied Tom’s prayer for TRO and injunction

ISSUE: Whether the prayer of TRO and injunction by Tom should be denied

HELD:
- No; CA committed grave abuse of discretion in denying Tom’s prayer for issuance of TRO and/or writ of preliminary injunction.
- The issuance of the injunctive writ is warranted to enjoin Nabunturan RTC from implementing the Order which will place Rodriguez
the control and management of GDITI.
- The Court has clarified that it appears to the records that Tom was actually elected as member of Board of Directors (actually, as
President) of GDITI while the injunction case in RTC Manila is pending during the Annual Stockholders Meeting in 2014
- This follows the well-entrenched rule that a corporation exercise its powers through its Board of Directors, except in instances
where the Corporation Code requires stockholders’ approval for certain specific acts. Basis: Sec. 23, Corporation Code.
- CA, in denying the prayer for issuance thereof, affirmed RTC’s decision placing the management and control of GDITI to
Rodriguez, a mere intervenor, on the basis of MOA between him and Basalo, in violation of Sec. 23 of the Corporation Code.

41
CASE DIGEST | Corporation Law (2019) Atty. Villanueva-Castro ib3h

39. GUILLERMO v. USON


G.R. NO. 198967
March 7, 2016

FACTS
● On March 11, 1996, respondent Crisanto Uson becan his employment with Royal Class Ventures Phils., Inc. (Royal Class) as an
accounting clerk, then subsequently promoted as accounting supervisor, until he was allegedly dismissed from employment on
December 20, 2000.
● Uson then filed with Sub-Regional Arbitration of Dagupan City of NLRC a complaint for illegal dismissal, wherein Royal Class
did not make any appearance thereto. The LA held in favor of Uson, ordering reinstatement from his position and payment of
backwages, among others.
● A Writ of Execution was issued thereafter, upon Uson’s motion, then an Alias Writ of Execution, and a Second Alias Writ of
Execution, after the first two were unsatisfied.
● After it was remain unsatisfied, Uson filed another Alias Writ of Execution, and to Hold Directors and Officers Liable for
Satisfaction of the Decision, alleging that in the Royal Class’ establishment is not on the corporation’s name but of Joel and Sons
Corporation, with which Jose Emmanuel Guillermo is the General Manager, who received the writ using the nickname “Joey”, and
pretended that he was the brother of Jose, and that the respondent’s corporation has been dissolved.
● The Labor Arbiter pierced the veil of corporate fiction, holding the officers of the corporation jointly and severally liable.
● Guillermo, filed a Motion for Reconsideration by way of special appearance with which the LA sustained the findings of the LAs
before her. NLRC dismissed Guillermo’s Appeal.
● CA denied also the appeal and upheld the findings of NLRC. CA held, citing jurisprudence that hold the president of corporation
liable for the latter’s obligation to illegally dismissed employees.
● Guillermo assails in this Petition saying that the piercing of the veil of corporate fiction allegedly discriminated him, as there are
other officers of the corporation; and that he was impleaded in the case only more than a year after the Decision had become final
and executory.

ISSUE: Whether or not an officer of a corporation may be included as judgment obligor in a labor case for the first time only after
the decision of the Labor Arbiter has become final and executory

HELD:
- In earlier labor cases, persons who were not originally impleaded in the case were, even during execution, held to be solidarily liable with the
employer corporation for the latter’s unpaid obligations to complainant-employees. This includes a newly-formed corporation which was considered
a mere conduit or alter-ego of the originally impleaded corporation, and/or the officers or stockholders of the latter corporation.
- Liability is attached in cases above even after final judgment and during execution, when there was a failure to collect from the employer
corporation the judgment debt awarded to its workers.
- These cases established the primacy of Sec. 31 of the Corporation Code in assigning such liability for a corporation’s debts, including judgment
obligations in labor cases.
- Under this provision, personal liability attaches only when, as enumerated therein, there is a willful and knowing assent to a patently unlawful
acts of the corporation; there is gross negligence or bad faith in directing the affairs of the corporation, or there is a conflict of interest resulting in
damages to the corporation.
- In cases where personal liability attaches, only the responsible officer, i.e. the person directly responsible for and who acted in bad faith in
committing the illegal dismissal or any act of violative of Labor Code, is held solidarily liable. They may be impleaded even after final judgment.
- In this case, satisfactory showing of fraud, bad faith, or malice must be present to justify the disgrading of corporate fiction, which is present in
this case.
- As reflected in the records of the case, there is indeed a pattern or scheme to avoid obligations to Uson and frustrate the execution of judgement
award. Guillermo is the person responsible in the actual running of the company and for the malicious illegal dismissal of the complainant, as well
as the role in dissolving the original obligor company in an obvious “scheme to avoild liability.” It was Guillermo, himself, as President and General
Manager who received summons to the case, and who, without justifiable cause, refused to receive all notices and orders of the LA.

42
CASE DIGEST | Corporation Law (2019) Atty. Villanueva-Castro ib3h

40. PIONEER INSURANCE v. MORNING STAR TRAVEL


G.R. NO. 198436
July 8, 2015

FACTS
● Morning Star is a travel and tours agency, accredited by International Air Transport, a Canadian corporation licensed to do business
in the Philippines, as a travel agent. They entered into a Passenger Sales Agency Agreement wherein Morning Star must report all
air transport ticket sales to IATA and account all payments received, Morning Star holding in trust all monies collected belo nging
to airline companies.
● IATA obtained a Credit Insurance Policy from Pioneer to assure itself from payments. Benny Wong, Morning Star president,
declared himself liable to indemnify Pioneer for any and all claims under the policy, among others.
● Morning Star had an accrued billing of P49,051,641.80 or $325,865.35 for period of December 16, to 31, 2002 which it failed to
remit, in which IATA declared Morning Star in default.
● IATA then demanded from Pioneer P109,728,051 or $427,834.14 representing Morning Star’s overdue account as of April 30,
2003, which Pioneer paid.
● Pioneer then demanded these amounts from Morning Star, with which it thereafter filed a Complaint for Collection of Sum of
Money and Damages against Morning Star and its shareholders and Directors. Morning Star, Benny and Estelita Wong were served
with summons.
● The RTC ruled in favor of petitioner and ordered respondents to jointly and severally pay Pioneer; CA held only Morning Star
liable.
● Pioneer, in this petition hinges on Sec. 31, Corporation Code that the directors were guilty of gross negligence or bad faith in
directing the affairs of the corporation, and the abnormally large indebtedness was incurred in fraud and bad faith, with Morning
Star having no intention to pay its debt and allegedly operating at a loss since 1998, with deficit of P12M as of December 31, 2000

ISSUE: WN piercing of the corporate veil is warranted

HELD:
- No. CA ruling upheld.
- A separate corporate personality shields corporate officers acting in good faith and within their scope of authority from personal
liability except for situations enumerated by law and jurisprudence.
- Among the exceptions comes from Sec. 31 of the Corporation Code which imputes personal liability a) where there is a willful and
knowing assent to a patently unlawful acts of the corporation; b) there is gross negligence or bad faith in directing the affairs of the
corporation; or c) there is a conflict of interest resulting in damages to the corporation.
- Bad faith means breach of a known duty through some motive or interest or ill will; it partakes of the nature of fraud. Piercing the
corporate veil to hold corporate officers personally liable for bad faith for the corporation’s debts requires “that the bad faith or
wrongdoing of the director must be established clearly and convincingly as bad faith is never presumed.”
- Badges of fraud were not proven in view of the fact that during the year 1999 the total assets of Morning Star is P134,361,353, and
total liabilities of P120,678,345, while in 2000 it has total assets of P150,579,421 and liabilities of P160,222,966 does not prove large
indebtedness and insolvency. Businesses may earn profits in some years and operate at a loss in others; it is in the nature of businesses
to take risks when making business judgments, and this includes taking loans and incurring liabilities.
- Furthermore, mere allegations that Morning Star Management Ventures Corporation and Pic ‘N Pac Mart (also owned by private
respondents) were doing relatively well at the time Morning Star was incurring huge losses and that the Morning Star Management
Ventures has the title over the land and building where the offices can be found do not prove that the individual respondents were
transferring Morning Star’s properties in fraud of its creditors absent proof that it was under the respondent’s name before.
- Note that it would offend due process rights to hold Morning Star Tour Planners, Inc. (another corporation held by private
respondents) responsible for Morning Star’s liabilities, who is not a party in this case.

43
CASE DIGEST | Corporation Law (2019) Atty. Villanueva-Castro ib3h

41. SIMNY G. GUY v. GILBERT G. GUY


G.R. NO. 184068
April 19, 2016

FACTS
● Goodland Company, Inc. is a family-owned corporation of the Guy family. Petitioner Simny Guy is a stockholder and member of
board of directors of the corporation; respondents are GCI stockholders allegedly elected as new directors by virtue of the assailed
stockholders’ meeting held on 7 September 2004.
● On August 31, 2004, petitioner was informed by Pe and Lim that they received a notice of specials stockholders meeting on 7
December 2004, with which petitioner received the same on 22 September 2004, or 15 days after the meeting. The meeting was
called by respondent Gilbert Guy, the Executive Vice President.
● Petitioners Simny Guy and Grace Cheu, allegedly a stockholder of record, seeked Nullification of Stockholders’ Meeting and
Election of Directors, as well us Acts and Resolutions before the Manila RTC alleging hat there was no previous notice to him, and
the meeting and the notice thereof was not called and issued by proper person.
● RTC granted the TRO enjoining respondents from acting or holding themselves out as new directors/officers.
● Consequently. the RTC ruled in favor of defendants, holding as the basis Art. II, Sec. 2 of the By-Laws that a special meeting may
be called upon order of the president or upon written request of stockholders registered as the owners of one-third of the total
outstanding stock and Art IV, Sec. 3 which provides that the Vice President shall exercise all the functions and perform all duties
if President is absent or disabled for any cause
● It was shown in the RTC that Gilbert owns 79.99% of shares of GCI, and incumbent president Francisco Guy Co Chia was
incapacitated by Alzhiemer’s Disease.
● CA affirmed RTC in toto.

ISSUE: WN the stockholders meeting was properly called and notices sent in compliance with the laws and by-laws

HELD:
-Yes.
- Sec. 50 of the Corporation Code provides that in special meetings of stockholders, “at least one (1) week written notice shall be
sent to all stockholders or members, unless otherwise provided in the by laws.”
- On the other hand, the GCI by-laws provide that the notice of meeting for every regular or special meeting “shall be prepared and
mailed to the registered post office address of each stockholder not less than five (5) days prior to the date set for such meeting x x
x”
- Both provisions were found by the court as clear and unambiguous: the provisions only require the sending/mailing of the notice
of a stockholders’ meeting to the stockholders of the corporation.
- The word “send” should be understood in its plain meaning, that is, “to deposit in the mail or deliver for transmission by any other
usual means of communication with postage or cost of transmission provided for and properly addressed x x x”
- Clearly, respondents are only mandated to notify petitioner by depositing in the mail the notice of the stockholders’ special meeting,
with postage provided. It appears that the notice was sent to petitioner through registered mail by respondents on 2 September 2004.
- Therefore, petitioner is considered to have received notice of the special stockholders’ meeting after said notice was properly mailed
by respondents.
- Anent the authority of Gilbert Guy to call the notice of special meeting, the evidence is conclusive that Gilbert owns 79.9% of
outstanding stock of GCI; pursuant to Art. II, Sec. 2 of the by-laws, Gilbert may validly call such special meeting.
- Furthermore, Sec. 25 of the Corporation Code only bars the President to be, at the same time, Secretary or Treasurer. Hence, under
Sec. 3, Art. IV of GCI by-laws, respondent Gilbert Guy, as VP of the corporation, is qualified to act as president.
- Anent the averment that Grace Cheu was not notified of the meeting, the Court held that a “stockholder of record” are those “whose
ownership of shares are duly registered in the stock and transfer book x x x.”
- Since RTC and CA found that Cheu was not stockholder of record of GCI, she is not entitled to be notified of the special
stockholder’s meeting.

44
CASE DIGEST | Corporation Law (2019) Atty. Villanueva-Castro ib3h

42. MALIXI v. MEXICALI PHILIPPINES


G.R. NO. 205061
June 8, 2016

FACTS
● Petitioner Emerita Malixi alleged that on August 12, 2008, she was hired by the respondents as a team leader assigned at the
delivery service without employment contract and ID.
● On October 2008, Mexicali’s training officer Teves informed her of the management’s intention to transfer and appoint her as store
manager at a newly-opened branch of Mexicali in Alabang Town Center, a joint venture between Mexicali and Calexico Food
Corporation. She then subsequently submitted a resignation letter as advised by the training officer.
● She then started working as store manager of Mexicali in ATC without employment contract and ID.
● On December 2008, she was compelled by Teves to sign an end-of-contract letter by reason of a criminal complaint for sexual
harassment she filed against Mexicali’s operations manager.
● When refused to sign, Mexicali’s administrative officer Luna personally went to the branch and caused the signing of the same.
she was informed that it was her last day of work on December 15, 2008.
● Petitioner filed a case before the NLRC alleging illegal dismissal.
● Respondents denied responsibility over the alleged dismissal, averring that petitioner has resigned in Mexicali and was rather an
employee of Calexico from October 2008, a franchisee of Mexicali and is a separate and distinct corporation.
● The LA pierced the corporate veil, having interlocking board of directors, hence holding Mexicali responsible for petitioner’s
dismissal.
● The NLRC held that Mexicali and Calexico are separate and distinct entities, but ordered Mexicali to reinstate petitioner because
they caused her resignation.
● CA upheld the NLRC decision.

ISSUE: Whether or not piercing of corporate veil is warranted

HELD:
- No.
- The Labor Arbiter failed to provide a clear justification for the piercing of corporate veil.
- Clear and convincing evidence is needed to warrant the application of the doctrine of piercing of corporate fiction.
- The Articles of Incorporation and By-Laws of both corporations show that they have distinct business purposes; they have different
set of incorporators; only two out of five directors of Mexicali are also directors of Caliexico.
- Note that the existence of interlocking directors, corporate officers, and shareholders is not enough justification to diresregard the
separate corporate personalities.
- To pierce the veil of corporate fiction, there should be clear and convincing proof that fraud, illegality, or inequity has been
committed against third persons.

45
CASE DIGEST | Corporation Law (2019) Atty. Villanueva-Castro ib3h

43. FOREST HILLS GOLF AND COUNTRY CLUB, INC. v. FIL-ESTATE PROPERTIES, INC.
G.R. NO. 206649
July 20, 2016
FACTS
● On March 31, 1993, Kingsville Construction and Development Corporation (Kingsville) and Kings Properties Corporation (KPC)
entered into a project agreement with respondent (FEPI) whereby the latter agreed to finance and cause the development of several
parcels of land by former into Forest Hills Residential Estates and Golf and Country Club.
● FEPI was tasked to incorporate petitioner and to perform development and construction work and other undertakings.
● On July 19, 1996, Rainier Madrid purchased two Class "A" shares at the secondary price of P380,000.00 each, and applied for a
membership to the club for P25,000.00.
● Due to the delayed construction of the second 18-Hole Golf Course, Madrid wrote demand letters to the Board of Directors of
petitioner against respondents FEPI and FEGDI. The Board of Directors, however, failed and/or refused to act on the demand
letters.
● Thus, on April 21, 2010, Madrid, in a derivative capacity on behalf of petitioner FHGCCI, filed with the Antipolo RTC a Complaint
for Specific Performance with Damages against FEPI.
● RTC dismissed the petition for lack of jurisdiction, without prejudice to the refiling of the same with the proper special commercial
court in Binangonan, Rizal.
● Petitioner directly filed a petition for certiorari before this Court.

ISSUE: W the derivative suit with ordinary civil suit for specific performance is cognizable by the regular courts or the special
commercial courts

HELD
- The complaint, denominated as a derivative suit for specific performance, falls under the jurisdiction of special commercial courts.
- Derivative suit is a remedy designed by equity as a principal defense of the minority shareholders against the abuses of the majority.
- Note that under the Corporation Code, the corporation’s power to sue is lodged with its board of directors or trustees. However,
when its officials refuse to sue, or are the ones to be sued, an individual stockholder may be permitted to institute a derivative suit to
enforce a corporate cause of action on behalf of the corporation in order to protect or vindicate its rights.
- Considering its purpose, a derivative suit, therefore, would necessarily touch upon the internal affairs of a corporation, hence among
the cases covered by Interim Rules of Procedure Governing Intra-corporate Controversies.
- Upon the enactment of RA 8799, jurisdiction over intra-corporate disputes, including derivative suits, is now vested in the RTCs
as special commercial courts.
- Lastly, Rule 8, Section 1 of the Interim Rules of Procedure Governing Intra-Corporate Controversies provides for the requisites of
a valid derivative suit, to wit:
a) was a stockholder or member at the time the acts or transactions subject of the action occurred and at the time the action was
filed;
b) exerted all reasonable efforts, and alleges the same with particularity in the complaint, to exhaust all remedies available under
the articles of incorporation, bylaws, laws or rules governing the corporation;
c) no appraisal right are available for the act or acts complained of; and
d) suit is not a nuisance or harassment suit.
- Petitioner failed to allege that he exerted all reasonable efforts to exhaust all remedies available, that no appraisal right is available,
and that the suit is not a nuisance or a harassment suit.

46
CASE DIGEST | Corporation Law (2019) Atty. Villanueva-Castro ib3h

44. KUKAN INTERNATIONAL CORPORATION v. REYES


G.R. NO. 182729
September 29, 2010

FACTS
● In March 1998, Kukan, Inc. conducted a bidding for the supply and installation of signages in a building being constructed in
Makati City.
● Private respondent Morales tendered the winning bid and was awarded the PhP 5 million contract but leter reduced to PhP
3,388,502.
● Despite his compliance with his contractual undertakings, Morales was only paid the amount of PhP 1,976,371.07, leaving a balance
of PhP 1,412,130.93, which Kukan, Inc. refused to pay. Morales filed a Complaint with the Manila RTC against Kukan, Inc, to
which the RTC ruled in favor of Morales.
● Morales moved for and secured a writ of execution against Kukan, Inc. The sheriff then levied upon various personal properties
found at what was supposed to be Kukan, Inc.’s office in Makati City. Alleging that it owned the properties thus levied and that it
was a different corporation from Kukan, Inc., Kukan International Corporation (KIC) filed an Affidavit of Third-Party Claim.
● In reaction to the third party claim, Morales interposed a motion praying for applying the principle of piercing the veil of corporate
fiction, it being alleged that both corporations are but one and the same entity. Morales filed a motion for Examination of Judgment
Debtors in a bid to establish the link between Kukan, Inc. and KIC.
● The RTC granted the motion; and the CA affirmed the same.

ISSUE: WN the principle of piercing the veil of corporate fiction is correctly applied

HELD:
- No.
- The piercing of corporate fiction, and the resulting treatment of two related corporations as one and the same juridical with respect
to a given transaction, is basically applied only to determine established liability;
- it is not available to confer on the court a jurisdiction it has not acquired, in the first place, over a party not impleaded in a case.
- Elsewise put, a corporation not impleaded in a suit cannot be subject to the court’s process of piercing the veil of its corporate
fiction.
- Aguedo Agbayani, a recognized author in Corporation Law, stated, “x x x the doctrine of piercing the veil of corporate fiction
comes to play only during the trial of the case after the court has already acquired jurisdiction over the corporation. x x x”
- The implication of the above comment is twofold:
(1) the court must first acquire jurisdiction over the corporation or corporations involved before its or their separate
personalities are disregarded; and
(2) the doctrine of piercing the veil of corporate entity can only be raised during a full-blown trial over a cause of action
duly commenced involving parties duly brought under the authority of the court
- In the case at bar, no full-blown trial involving KIC was had when the RTC disregarded the corporate veil of KIC, but rather, by a
mere motion of a party with whom it has no privity of contract and after the decision in the main case had already become final and
executory.
- Lastly, in instances when the Court pierced the veil of corporate fiction of two corporations, there was a confluence of the following
factors: 1) A first corporation is dissolved; 2) assets of the first corporation is transferred to a second corporation to avoid a financial
liability of the first corporation; and 3) both corporations are owned and controlled by the same persons such that the it should be
considered as a continuation and successor of the first corporation.
- In the instant case, however, the second and third factors are conspicuously absent. Mere ownership by a single stockholder of a
substantial number of shares of a corporation (Michael Chan owing 40% of common shares of both corporations) does not, by itself,
provide justification for disregarding the separate juridical personality.
- There is no showing that the incorporation, and the separate and distinct personality of KIC was used to defeat Morales’ right to
recover from Kukan, Inc.

47
CASE DIGEST | Corporation Law (2019) Atty. Villanueva-Castro ib3h

45. AGUIRRE II v. FQB+7, INC.


G.R. NO. 170770
January 9, 2013

FACTS:
- On October 5, 2004, petitioner Vitaliano Aguirre II filed, in his individual capacity and on behalf of FQB+7, Inc., a Complaint
against respondents Nathaniel, Priscila Bocobo, and Antonio De Villa (Antonio). - The Complaint alleged that FQB+7 was
established in 1985 with the following directors and subscribers:
Directors Subscribers
1. Francisco Q. Bocobo 1.Francisco Q. Bocobo
2. Fidel N. Aguirre 2. Fidel N. Aguirre
3. Alfredo Torres 3. Alfredo Torres
4. Victoriano Santos 4. Victoriano Santos
5. Victorino Santos 5. Victorino Santos
6. Vitaliano Aguirre II
7. Alberto Galang
8. Rolando B. Bechayda
- The Complaint further alleged that, sometime in April 2004, Vitaliano discovered a GIS) of FQB+7, filed by Francisco Bocobo's
heirs, Nathaniel and Priscila, as FQB+7's president and secretary/treasurer, respectively.
Directors Subscribers
1. Nathaniel D. Bocobo 1.Nathaniel D. Bocobo
2. Priscila D. Bocobo 2. Priscila D. Bocobo
3. Fidel N. Aguirre 3. Fidel N. Aguirre
4. Victoriano Santos 4. Victorino Santos
5. Victorino Santos 5. Victorino Santos
6. Consolacion Santos
- This prompted Vitaliano to write to the "real" Board of Directors (the directors reflected in the Articles of Incorporation),
questioning the validity and truthfulness of the alleged stockholders meeting held on September 3, 2002.
- He asked the "real" Board to rectify what he perceived as erroneous entries in the GIS, and to allow him to inspect the corporate
books and records, which ignored Vitaliano's request.
- On September 27, 2004, Nathaniel, in the exercise of his power as FQB+7's president, appointed Antonio as the corporation's
attorney-in-fact. Pursuant thereto, Antonio attempted to take over the farm, but was allegedly prevented by Fidel and his men.
- The Complaint asked for an injunction against them and for the nullification of all their previous actions, and also sought damages
for the plaintiffs and a declaration of Vitaliano's right to inspect the corporate records.
- On October 27, 2004, the trial court issued the writ of preliminary injunction after Vitaliano filed an injunction bond.
- The respondents filed a motion for an extension of 10 days to file the pleadings which the trial court denied motion for being a
prohibited pleading under the Interim Rules of Procedure Governing Intra-corporate Controversies under Republic Act (R.A.) No.
8799.
- The respondents filed a Petition for Certiorari and Prohibition on the ground that Branch 24 of the Manila RTC has no jurisdiction,
theorizing that Vitaliano's real goal in filing the Complaint was to maintain custody of the corporate farm in Quezon Province, the
jurisdiction belonging to the Department of Agrarian Reform (DAR), not to the Manila RTC. Respondents also invoked that FQB+7’s
Certificate of Registration has been revoked, hence dissolved.
- CA dismissed the petition of Aguirre, as well as holding that the RTC committed grave abuse of discretion. CA held that Section
122 of the Corporation Code allows a dissolved corporation to continue as a body corporate for the limited purpose of liquidating
the corporate assets and distributing them to its creditors, stockholders, and others in interest. CA determined that Vitaliano's
Complaint, being geared towards the continuation of FQB+7, Inc.'s business, should be dismissed

ISSUE: WN the RTC has jurisdiction over an intra-corporate dispute involving a dissolved corporation.

48
CASE DIGEST | Corporation Law (2019) Atty. Villanueva-Castro ib3h
HELD:
- Yes. For one, the complaint is only to determine and vindicate an alleged stockholder’s right to return of his stockholdings and to
participate in the election of directors, and corporation’s right to remove usurpers and strangers from its affairs, which is not mooted
by the dissolution.
- A corporation’s board of directors is not rendered functus officio by its dissolution. Since Sec. 122 allows a corporation to continue
its existence for a limited purpose, then necessarily there must be a board that will continue acting for and behalf of the dissolved
corporation for that purpose.
- Sec. 122 authorizes the dissolved corporation’s board of directors to conduct its liquidation within three years, while jurisprudence
even recognized the board’s authority as trustee for person in interest beyond the three-year period.
- Furthermore, a party’s stockholdings in a corporation, whether existing or dissolved, is a property right which he may vindicate
against another party who has deprived him thereof, which right is not dissolved upon dissolution.

49
CASE DIGEST | Corporation Law (2019) Atty. Villanueva-Castro ib3h

46. MARC II MARKETING v. JOSON


G.R. NO. 171993
December 12, 2011

FACTS
● Petitioner Marc II Marketing, Inc. is engaged in buying, marketing, selling, and distributing in retail or wholesale for export or
import household appliances and products and other items, who took over the business operations of Marc Marketing, Inc.
● Petitioner Lucila Joson is the President and majority stockholder of the corporation. On the other hand, respondent Alfredo Joson
was the General Manager, incorporator, director, and stockholder of petitioner corporation.
● Before petitioner corporation was officially incorporated, respondent has been engaged by Lucila to work as General Manager of
the petitioner corporation, formalized through a Management Contract. After the petitioner corporation was officially incorporated,
he continued his duties as General Manager.
● Pursuant to Sec. 1, Article IV of the petitioner’s bylaws, its corporate officers are: 1) Chairman, 2) President, 3) one or more Vice-
President(s), 4) Treasurer, and 5) Secretary. Its Board of Directors, may, from time to time, appoint such other officers as it may
determine to be necessary or proper.
● On June 1997, petitioner corporation decided to stop and cease its operations, and the respondent was apprised of the termination
of his services as General Manager since his services are no longer necessary for the winding up.
● Feeling aggrieved, respondent filed before the NLRC complaint for reinstatement and money claim.
● Notwithstanding the motion by petitioner to dismiss on the ground of lack of jurisdiction as that the case involves an intra-corporate
controversy, the LA held that the position of General Manager is not a corporate officer, and that the dismissal is illegal.
● The NLRC ruled in favor of petitioners, dismissing the complaint for want of jurisdiction, holding that the position is a corporate
officer.
● CA reinstated the decision of the Labor Arbiter.

ISSUE: Whether or not the respondent as General Manager of petitioner corporation is a corporate officer or a mere employee.

HELD:
- Corporate officers are those officers of a corporation who are given that character either by the Corporation Code or by
corporation’s by-laws.
- Sec. 25 of the Corporation Code specifically enumerates the corporate officers, to wit: 1) president; 2) secretary; 3) treasurer, and
4) such other officers as may be provided for in the by-laws.
- Conforming thereto, a position must be expressly mentioned in the bylaws in order to be considered as corporate office. An enabling
provision is not enough to make a position a corporate office; otherwise they could be considered only as employees or subordinate
officials.
- Therefore, corporate officers are exclusively those who are given that character either by the Corporation Code or by the
corporation’s bylaws.

- The Board has no power to create other Offices (other than those enumerated in the bylaws) without amending first the corporate
bylaws.
- A careful perusal of the petitioner’s bylaws show that the position was not among those enumerated; mere board resolution to that
effect cannot make such position a corporate office.
- Unless and until petitioner corporation’s bylaws is amended for the inclusion of General Manager in the list of its corporate officers,
such position cannot be considered as a corporate office within the realm of Sec. 25 of the Corporation Code.
- Though occupying a high-ranking and vital position in petitioner corporation, he can only be regarded as its employee or subordinate
official.

- This is to note that Sec. 25 safeguards to constitutionally enshrined right of every employee to security of tenure

50
CASE DIGEST | Corporation Law (2019) Atty. Villanueva-Castro ib3h

47. MINDANAO SAVINGS AND LOAN ASSOCIATION, INC. v. WILLKOM


G.R. NO. 178618
October 20, 2010

FACTS
● First Iligan Savings and Loan Association (FISLAI) and Davao Savings and Loan Association, Inc. (DSLAI) entered into a merger
in 1985 with DSLAI as the surviving corporation, but the articles of merger were not accepted by the SEC due to incomplete
documentation.
● On August 12, 1985, DSLAI changed its corporate name to MSLAI by way of amendment to its Articles of Incorporation.
● In 1990, because the business of MSLAI has failed, the Monetary Board ordered its closure and placed it in receivership on August
31, 1990.
● Private respondent Remedios Uy filed with the RTC Iligan City prior to the closure of MSLAI for a collection of money against
FISLAI, to which the RTC issued a summary decision in favor of Uy.
● On April 28, 1993, the sheriff levied on 6 parcels of land owned by FISLAI in Cagayan De Oro, in which Edward Willkom became
the highest bidder in the public auction thereof.
● MSLAI, represented by PDIC, filed before the Cagayan de Oro RTC a complaint for Annulment of Sale, Cancellation of Title and
Reconveyance of Properties against Respondents because it alleged that the sale was made without notice to PDIC, and that the
previous RTC decision was illegal and contrary to law because the assets of an institution placed under receivership such as MSLAI
should be deemed in custodia legis and should be exempt from any other order of levy, execution, etc.
● Respondents averred that MSLAI had no cause of action against them because MSLAI is a separate and distinct entity from FISLAI;
and the unofficial merger between FISLAI and DSLAI (now MSLAI) did not take effect because it did not comply with the
requirements prescribed by the Corporation COde.
● RTC dismissed the case for lack of juristiction; MSLAI failed to obtain favorable decision in CA, ruling that there was no merger.

ISSUE: Whether or not the merger between FISLAI and DSLAI (now MSLAI) valid and effective

HELD:
- No.
- In the merger of two or more existing corporations, one of the corporations survives and continues the combined business, while
the rest are dissolved and all their rights, properties, and liabilities are acquired by the surviving corporation.
- The merger, however, does not become effective upon the mere agreement of the constituent corporations. Since a merger or
consolidation involves fundamental changes in the corporation, as well as in the rights of stockholders and creditors, there must be
an express provision of law authorizing them. (Bases: Sec. 76-79, Corporation Code)
- The merger shall only be effective upon the issuance of a certificate of merger by the SECm subject to its prior determination that
the merger is not inconsistent with the Corporation Code or existing laws.
- In the case at bar, the articles of merger between FISLAI and DSLAI were not registered with the SEC due to incomplete
documentation. Though recognized by the Monetary Board, the fact remains that no certificate was issued by the SEC, thus,
incomplete.
- The issuance of the certificate of merger is crucial because not only does it bear out SEC’s approval but it also marks the moment
the consequences of a merger take place.
- Absorbed corporation ceases to exists;
- But its rights and properties, and liabilities, shall be taken and deemed transferred to and vested in the surviving
corporation.
- There being no merger between FISLAI and DSLAI, two corporations shall not be considered as one but two separate corporations.
It has personality separate and distinct from the persons composing it.
- Thus, the assets of FISLAI remain as its assets and cannot be considered as belonging to DSLAI and MSLAI.

51
CASE DIGEST | Corporation Law (2019) Atty. Villanueva-Castro ib3h

48. ROY III v. HERBOSA


G.R. NO. 207246
Novermber 22, 2016

FACTS
● As a result of the Court’s ruling in Gamboa v. Teves (652 SCRA 690), the SEC posted a Notice inviting the public to attend a public
dialogue and to submit comments on the draft memorandum circular on the guidelines to be followed in determining compliance
with the Filipino ownership requirement in public utilities under Sec. 11, Art. XII of the 1987 Constitution and the Gamboa
Decision.
● Petitioner Atty. Jose Roy III submitted his written comments on the draft guidelines.
● On May 2013, SEC through respondent Chairperson Teresita Herbosa, issued SEC-MC No. 8, wherein Sec. 2 provides that for
compliance of constitutional and statutory ownership requirement, the required percentage of Filipino ownership shall be applied
to both:
a) total number of outstanding shares of stock entitled to vote in the election of directors AND
b) the total number of outstanding shares of stock, whether or not entitled to vote in the election of directors.
● Petitioner, as lawyer and taxpayer, filed the Petition, assailing the validity of SEC-MC No. 8 for not conforming to the letter and
spirit of the Gamboa Decision and Resolution.
● He seeks to apply the 60-40 Filipino ownership requirement separately to each class of shares of a public utility corporation.

ISSUE: Whether SEC gravely abused its discretion in issuing SEC-MC No. 8 in the light of Gamboa Decision and Resolution

HELD:
- No.
- To recapitulate, Gamboa v. Teves consistently defined ‘capital’ in Sec. 11, Art. XII refers only to “shares of stock entitled to vote
in the election of directors, and this in the present case only to common shares, and not to the total outstanding capital stock
comprising both common and nonvoting preferred shares. x x x However, if the preferred shares also have the right to vote in the
election of directors, then the term “capital” shall include such preferred shares x x x.” In short, the term “capital” refers only to
shares of stock that can vote in the election of directors.
- In this regard, since Filipinos own at least 60% of the outstanding shares of stock entitled to vote directors, then the Filipino
stockholders control the corporation, i.e. they dictate corporate actions and decisions, and they have all the rights of ownership. And
as owners of the corporation, the economic benefits will necessarily accrue to them. Surely, these “true owners” will not allow any
dilution of their ownership and control if such move will not be beneficial to them.
- Furthermore, the Gamboa Resolution, after reviewing SEC and DOJ Opinions as well as the provisions of the Foreign Investments
Act, reiterated that both the Voting Control Test and the Beneficial Ownership to determine whether a corporation is a “Philippine
national”
- To this, the Final Word of the Gamboa resolution put to rest the interpretation of the term “capital,” to wit: Full beneficial ownership
of stocks, coupled with appropriate voting rights is essential.

- Sec. 2 of SEC-MC No. 8 clearly incorporates the Voting Control Test, and even goes beyond requiring a 60-40 ratio in the voting
stocks; it requires the 60-40 percentage ownership in the total number of outstanding shares of stock, whether voting or not.
- While SEC-MC No. 8 does not expressly mention the Beneficial Ownership Test, it does not follow that the SEC will not apply
this test in determining whether the shares claimed to be owned by Philippine nationals are Filipino.

Re: beneficial ownership test


- A beneficial owner means any person who, directly or indirectly, through any contract, arrangement, understanding, relationship
or otherwise, has or shares voting power, and/or investment returns or power
- Therefore, mere legal title is NOT ENOUGH to meet the required Filipino equity, he should also have full beneficial ownership of
the share. If the dividends and other fruits and accessions of the share do not accrue to a Filipino citizen or national, then that share
is excluded or not counted.

52
CASE DIGEST | Corporation Law (2019) Atty. Villanueva-Castro ib3h
- The Final Word of the Gamboa resolution reflects this interpretation: if the Filipino has the specific stock’s VOTING POWER, or
has the INVESTMENT POWER, or BOTH, then such Filipino is the “beneficial owner” of that “specific stock”, and these specific
stocks will be added together and the sum must be equivalent to at least 60% of the total outstanding shares of stock entitled to vote
in the election of directors and at least 60% of the total number of outstanding shares of stock, whether or not entitled to vote in the
election of directors.
- The Gamboa Decision held that preferred shares are to be factored in only if they are entitled to vote in the election of directors. If
preferred shares have no voting rights, then they cannot elect members of the board of directors, which wields control of the
corporation.
- Anent the 8 instances in Sec. 6 of the Corporation Code, the Gamboa Decision did not find them significant in resolving the issue
of the proper interpretation of the Constitutional provision.
- In this case, these matters require, at the outset, a favorable recommendation by the management to the board.
- Since Sec. 11, Art. XII requires an all-Filipino management team as a safeguard who will recommend any of the acts to
the board, with foreign participation in the board cannot exceed 40% of the total number of board seats, then the Filipino
directors comprise the majority, which do not even need the vote of the foreign directors to approve the intended corporate
act.
- After approval by the board, ⅔ of the outstanding capital stock must vote. GIven the supermajority vote requirement,
foreign shareholders cannot dictate upon their Filipino counterpart.
- After all, the right to vote in these 8 corporate matters in Sec. 6, is an acknowledgement of their right to ownership, not of
control.
- Therefore, a too restrictive definition of “capital”, one which was never contemplated in the Gamboa Decision, will surely have a
dampening effect on the business milieu by eroding the flexibility inherent in the issuance of preferred shares with varying terms and
conditions.
- Nowhere in the discussion of the definition of the term “capital” did the court mention the 60% Filipino equity requirement to be
applied to each class of shares.
- As a closing, and to wrap up the discussion, a public utility corporation or association, whether stock or non-stock, Filipino
shareholders or members wielding 60% voting power, elects the Filipino director who, in turn, together with other Filipino
directors comprising a majority of the board of directors or trustees, appoints and employs the all-Filipino management
team. This is what is envisioned by the Constitution to assure effective control by Filipinos.

49. ROY III v. HERBOSA


G.R. NO. 207246
April 18, 2017

RESOLUTION

- The Gamboa Decision already held, in uncertain terms, that what the Constitution requires is “full and legal beneficial ownership
of 60 percent of the outstanding capital stock, coupled with 60 percent of the voting rights x x x must rest in the hands of Filipino
nationals x x x” which is what SEC-MC No. 8 provides, viz:”the required percentage of FIlipino ownership shall be applied to BOTH
a) the total number of outstanding shares of stock entitled to vote in the election of directors; AND b) the total number of outstanding
shares of stock, whether or not entitled to vote x x x.”
- Full beneficial ownership in IRR of FIA provides that FULL BENEFICIAL OWNERSHIP of the stocks, coupled with appropriate
voting rights is essential.
- If the Filipino has the voting power of the specific stock, i.e. he can vote the stock or direct another to vote for him, or has the
investment power over the specific stock i.e. can dispose of the stock or direct another to dispose it for him, or both, i.e. he can vote
and dispose of that “specific stock” or direct another to vote or dispose it for him then such FIlipino is the “beneficial owner” of that
specific stock, which will be counted as part of the 60% Filipino ownership requirement under the Constitution.
- Since it would be apropos to state that since Filipinos own at least 60% of the outstanding shares of stock entitled to vote directors,
which is what the Constitution precisely requires, then the Filipino stockholders control the corporation.

53
CASE DIGEST | Corporation Law (2019) Atty. Villanueva-Castro ib3h

V. RIGHTS OF SHAREHOLDERS

50. LEE v. CA
G.R. NO. 93695
February 4, 1992

FACTS
- On November 15, 1985, a complaint for a sum of money was filed by the International Corporate Bank, Inc. against the private
respondents who, in turn, filed a third party complaint against Alfa Integrated Textile Mills (ALFA) and the petitioners on March
17, 1986.
- After denying petitioner’s motion to dismiss, the trial court issued an order requiring the issuance of an alias summons upon ALFA
through the DBP as a consequence of the petitioners' letter informing the court that the summons for ALFA was erroneously served
upon them considering that the management of ALFA had been transferred to the DBP. DBP claimed it had not taken over the
company which has a separate and distinct corporate personality and existence.
- The private respondents argued that the voting trust agreement executed between DBP and ALFA did not divest the petitioners of
their positions as president and executive vice president of ALFA so that service of summons upon ALFA through the petitioners as
corporate officers was proper.
- Trial court upheld the validity of service of summons through ALFA, then reversed after MR filed by petitioners by virtue of the
voting trust agreement. The court said that they were no longer corporate officers of ALFA, they could no longer receiver summons
on behalf of ALFA.
- Private respondents submitted a petition for certiorari before the CA. The trial court, unaware of the pending petition,declared as
final their decision.
- The CA thereafter decided to set aside the RTC decision.

ISSUE: Whether or not there was proper service of summons on ALFA through the petitioners as president and vice president of
ALFA

HELD:
- By its very nature, a voting trust agreement results in the separation of the voting rights of a stockholder from his other rights such
as the right to receive dividends, the right to inspect the books of the corporation, the right to sell certain interests in the assets of the
corporation and other rights to which a stockholder may be entitled until the liquidation of the corporation.
- A voting trust agreement may confer upon a trustee not only the stockholder's voting rights but also other rights pertaining to his
shares as long as the voting trust agreement is not entered "for the purpose of circumventing the law against monopolies and illegal
combinations in restraint of trade or used for purposes of fraud."
- There is no dispute as to the most immediate effect of a voting trust agreement on the status of a stockholder who is a party to its
execution—from legal titleholder or owner of the shares subject of the voting trust agreement, he becomes the equitable or beneficial
owner.
- The Corporation Code on eligibility of a director formally legalized the fact that in order to be eligible as a director, what is material
is the legal title, not beneficial ownership of, the stock as appearing on the books of the corporation.
- The facts of this case show that the petitioners, by virtue of the voting trust agreement executed in 1981 disposed of all their shares
through assignment and delivery in favor of the DBP, as trustee. Consequently, the petitioners ceased to own at least one share
standing in their names on the books of ALFA as required under Section 23 of the new Corporation Code.
- By virtue thereof, DBP became the stockholder of record with respect to the said shares of stocks. In the absence of a showing that
the DBP had caused to be transferred in their names one share of stock for the purpose of qualifying as directors of ALFA, the
petitioners can no longer be deemed to have retained their status as officers of ALFA which was the case before the execution of the
subject voting trust agreement.

54
CASE DIGEST | Corporation Law (2019) Atty. Villanueva-Castro ib3h

51. REPUBLIC (PCGG) v. SANDIGANBAYAN


G.R. NO. 107789
April 30, 2003

FACTS
● On August 7, 1991, PCGG conducted an Eastern Technologies Philippines, Inc. (ETPI) stockholders meeting during which a PCGG
controlled board of directors was elected.
● A special stockholders meeting was later convened by the registered ETPI stockholders wherein another set of board of directors
was elected, as a result of which two sets of such board and officers were elected.
● Private respondent Victor Africa, a stockholder of ETPI, alleging that the PCGG had since January 29, 1988 been “illegally
‘exercising’ the rights of stockholders of ETPI,” especially in the election of the members of the board of directors, filed the
questioned motion before the Sandiganbayan praying te “calling and holding of the [ETPI] annual stockholders meeting for 1992
under the court’s control and supervision and prescribed guidelines.”
● The PCGG did not object to the motion, provided that either 1) an Order be issued upholding the right to vote theall the Class A
shares of ETPI; or 2) if the right to vote the sequestered shares be not upheld, an Order be issued, among others, llowing PCGG to
vote 23.9% of the total subscription in ETPI, and directing the amendment of the AOI and By-laws of ETPI providing for the
minimum safeguards for the conservation of assets x x x prior to the calling of the stockholder’s meeting.
● Sandiganbayan issued the assailed Resolution resolving Africa’s motion, ordering the annual stockholders meeting of ETPI for
1992 be held. PCGG assailed the Resolution imputing GAD in ruling that the registered stockholders of ETPI had the right to vote,
while it cannot vote at least 23.9% of the outstanding capital stock.
● Meanwhile, PCGG, in early 1995, filed a “VERY URGENT PETITION FOR AUTHORITY TO HOLD SPECIAL
STOCKHOLDER’S MEETING FOR [THE] SOLE PURPOSE OF INCREASING [ETPI’s] CAPITAL STOCK,”to which the
Sandiganbayan issued a Resolution assailed herein as well, granting the petition, as well as entitling PCGG to vote the sequestered
shares of stock. The autorized capital stock was increased from P250M to P2.6 B.
● Africa filed before this Court a motion to cite the PCGG “and its accomplices” in contempt and “to nullify the ‘stockholders
meeting’ called/conducted by PCGG and its accomplices,” he contending that only this Court, and not the Sandiganbayan, has the
power to authorize the PCGG to call a stockholders meeting and vote the sequestered shares.

ISSUE: WN the PCGG can vote the sequesterd ETPI shares in the stockholders meeting for the election of the BOD

HELD:
- No. PCGG cannot exercise acts of dominion over property sequestered, frozen, or provisionally taken over. The Court, in arriving
to such decision, relies heavily with the case of Baseco v. PCGG.
- The act of sequestration[,] freezing or provisional takeover of property does not import or bring about a divestment of title over
said property; [it] does not make the PCGG the owner thereof. In relation to the property sequestered, frozen or provisionally taken
over, the PCGG is a conservator, not an owner.
- PCGG has only powers of administration, the intrusion into management should be restricted to the minimum degree necessary to
accomplish the legislative will, which is “to prevent the disposal or dissipation” of the business enterprise.
- The sequestered stock is not to be voted to replace directors, or revise the articles or by-laws, or otherwise bring about substantial
changes in policy, program or practice of the corporation except for demonstrably weighty and defensible grounds, and always in
the context of the stated purposes of sequestration or provisional takeover, i.e., to prevent the dispersion or undue disposal of the
corporate assets.
- The court developed a “two-tiered” test in determining whether the
PCGG may vote sequestered shares:
a) whether there is prima facie evidence showing that the said shares are ill-gotten and thus belong to the state; and
b) whether there is an immediate danger of dissipation thus necessitating their continued sequestration and voting by the PCGG
while the main issue pends with the Sandiganbayan.
- The two-tiered test, however, does not apply in cases involving funds of public character, wherein:

55
CASE DIGEST | Corporation Law (2019) Atty. Villanueva-Castro ib3h
a) where the government shares are taken over by private persons or entities who/which registered them in their own names,
and
b) where the capitalization or shares that were acquired with public funds somehow landed in private hands.
- This Court made no finding in PCGG v. SEC et al. that Africa dissipated ETPI’s assets. And Sandiganbayan, in its impugned
Resolution herein, further held that the PCGG could not vote the sequestered shares as “only the owners of the shares of stock of
subject corporation, their duly authorized representatives or their proxies, may vote the said shares.
- The Sandiganbayan held that the public character exception does not apply, in which case it should have proceeded to apply the
two-tiered test, but failed to do.
- The questions thus remain if there is prima facie evidence showing that the subject shares are ill-gotten and if there is imminent
danger of dissipation. This Court is not, however, a trier of facts, hence, it is not in a position to rule on the correctness of the PCGG’s
contention. Consequently, this issue must be remanded to the Sandiganbayan for resolution.

56
CASE DIGEST | Corporation Law (2019) Atty. Villanueva-Castro ib3h

52. REPUBLIC V. COCOFED


G.R. NO. 147062-64
December 14, 2001

FACTS
● PCGG, by virtue of EO 1 creating the commission in the recovery of the ill-gotten wealth, EO 2 stating that the ill-gotten assets
and properties are in the form of shares of stocks, among others issued and implemented numerous sequestrations, freeze orders,
and provisional takeovers of allegedly ill-gotten companies, assets and properties.
● Among the properties sequestered were shares of stock in the United Coconut Planters Bank (UCPB) registered in the names of
alleged “one million coconut farmers”, the so-called Coconut Industry Investment Fund companies (CIIF), and private respondent
Eduardo Cojuangco, Jr.
● In February 13, 2001, the Board of Directors of UCPB received from ACCRA Law Office a letter written on behalf of the
COCOFED and the alleged nameless one million coconut farmers, demanding the holding of a stockholders’ meeting for the
purpose of electing the board of directors, which the board approved.
● Thereater, “COCOFED, et al. and Ballares, et al.” filed the class action omnibus motion asking the Sandiganbayan to enjoin the
PCGG from voting the UCPB shares of stock registered in the respective names of the more than one million coconut farmers, and
those shares registered in the names of CIIF holding companies.
● The Sandiganbayan issued the assailed order, holding that COCOFED et al and Ballares, et al were acknowledged registered
stockholders of the UCPB and hence authorized to vote theirs shares of stock and themselves to be voted upon in the UCB
Stockholders’ Meeting.

ISSUE: WN petitioner PCGG may vote in the sequestered shares of stock

HELD:
- Yes. Court holds that the government should be allowed to continue voting those shares inasmuch as they were purchased with coconut levy
funds—funds that are prima facie public in character or, at the very least, are “clearly affected with public interest.
-As a general rule, the registered owner of the shares of a corporation exercises the right a and the privilege of voting. This principle applies even
to shares that are sequestered by the government, over which the PCGGas a mere conservator cannot, as a general rule, exercise acts of dominion.
- Baseco v. PCGG provided two clear “public character” exceptions, apart from the “two-tiered test”, under which the government is granted the
authority to vote the shares, to wit:
a) Where government shares are taken over by private persons or entities who/which registered them in their own names, and
b) Where the capitalization or shares that were acquired with public funds somehow landed in private hands.
- The exceptions are based on the common-sense principle that legal fiction must yield to truth; that public property registered in the names of non-
owners is affected with trust relations; and that the prima facie beneficial owner should be given the privilege of enjoying the rights flowing from
the prima facie fact of ownership.
- It is not disputed that the money used to purchase the sequestered UCPB shares came from the Coconut Consumer Stabilization F und (CCSF),
otherwise known, as the coconut levy fund. In the 1993 Republic v. Sandiganbayan Resolution, the Court stated that coconut levy funds were
clearly affected with public interest. Private respondents have not demonstrated satisfactorily that they hae legitimately become private funds.
- Public funds are those moneys belonging to the State or to any political subdivision of the State; more specifically, taxes, customs duties and
moneys raised by operation of law for the support of the government or for the discharge of its obligations.
- Following this definition, coconut levy funds undeniably satisfy the general definition of public funds, because of the following reasons:
1) Coconut levy funds are raised with the use of the police and taxing powers of the State;
2) They are levies imposed by the State for the benefit of the coconut industry and its farmers;
3) Respondents have judicially admitted that the sequestered shares were purchased with public funds;
4) COA reviews the use of coconut levy funds;
5) BIR, with acquiescence of private respondents, has treated them as public funds;
6) The very laws governing coconut levies recognize their public character.
- Having shown that the coconut levy funds are not only affected with public interest, but are in fact prima facie public funds, this Court believes
that the government should be allowed to vote the questioned shares, because they belong to it as the prima facie beneficial and true owner.

57
CASE DIGEST | Corporation Law (2019) Atty. Villanueva-Castro ib3h

53. EVANGELISTA v. SANTOS


G.R. NO. L-1721
May 19, 1950

FACTS
● This is an action by the minority stockholders of a corporation against its principal officer for damages resulting from his
mismanagement of its affairs and misuse of its assets.
● Plaintiffs are minority stockholders of Vitali Lumber Company, Inc., a Philippine corporation organized for the exploitation of a
lumber concession; that defendant holds more than 50 per cent of the stocks of said corporation and also is and always has been
the president, manager, and treasurer thereof; and that defendant, in such triple capacity, through fault, neglect, and abandonment
allowed its lumber concession to lapse and its properties and assets, among them machineries, buildings, warehouses, trucks, etc.,
to disappear, thus causing the complete ruin of the corporation and total depreciation of its stocks.
● The complaint therefore prays for judgment requiring defendant: (1) to render an account of his administration of the corporate
affairs and assets: (2) to pay plaintiffs the value of their respective participation in said assets on the basis of the value of the stocks
held by each of them; and (3) to pay the costs of suit.
● Defendant respondent filed a motion for dismissal of the complaint on ground of improper venue and that the complaint did not
state a cause of action in favor of plaintiffs.
● After hearing, the lower court rendered its order, granting the motion for dismissal on the two grounds alleged by the defendant.

ISSUE: Whether or not petitioners have the right to bring the action for their benefit

HELD:
- No. Note that the action is for damages resulting from mismanagement of the affairs and assets of the corporation by its principal
officers. The injury complained of is thus primarily to the corporation, so that the suit for the damages claimed should be by the
corporation rather than by the stockholders.
- While it is to the corporation the action should pertain in this nature of cases, if the officers of the corporation, who are the ones
called upon to protect their rights, refuse to sue, or where a demand upon them to file the necessary suit would be futile because they
are the very ones to be sued or because they hold the controlling interest in the corporation, then in that case any one of the
stockholders is allowed to bring suit.
- But the real party in these cases is the corporation, and not the plaintiff stockholder, so that such damages as may be recovered,
shall pertain to the corporation.
- In this case, the plaintiff stockholders have brought the action not for the benefit of the corporation but for their own, since they ask
that the defendant make good the losses occasioned by his mismanagement and pay to them the value of their respective participation
in the corporate assets on the basis of their respective holdings.
- Clearly, this cannot be done until all corporate debts, if there be any, are paid and the existence of the corporation terminated by
the limitation of its charter or by lawful dissolution, lest it would result in the appropriation by, and the distribution among them of
part of the corporate assets before the dissolution of the corporation.
- Note that the action stated in their complaint is susceptible of being converted into a derivative suit for the benefit of the corporation
by a mere change in the prayer. Such amendment, however, is not possible now, since the complaint has been filed in the wrong
court, so that the same has to be dismissed.

58
CASE DIGEST | Corporation Law (2019) Atty. Villanueva-Castro ib3h

54. CHUA v. CA
G.R. NO. 150793
November 19, 2004

FACTS
● On February 28, 1996, private respondent Lydia Hao, treasurer of Siena Realty Corporation, filed a complaint-affidavit with the
City Prosecutor of Manila charging Francis Chua and his wife, Elsa Chua, of four counts of falsification of public documents
pursuant to Article 172 in relation to Article 171 of the Revised Penal Code
● MeTC granted Chua’s motion to exclude complainant’s private counsels as private prosecutors and ordered the complainant’s
counsels to be excluded from actively prosecuting Criminal Case No. 285721. Hao moved for reconsideration but it was denied.
● Hao then filed a certiorari entitled Lydia C. Hao, in her own behalf and for the benefit of Siena Realty Corporation v. Francis
Chua, and the Honorable Hipolito dela Vega, Presiding Judge, Branch 22, Metropolitan Trial Court of Manila, before the RTC
of Manila, which it granted, reversing the MeTC.
● Petitioner appealed before the CA imputing grave abuse of discretion in allowing Siena Realty Corporation to be impleaded as co-
party even though it was not a party to the criminal case, which the CA denied.

ISSUE: Whether or not the criminal case in question is in nature, a derivative suit

HELD:
- The Court held in the negative. A derivative action is a suit by a shareholder to enforce a corporate cause of action. The corporation
is a necessary party to the suit. And the relief which is granted is a judgment against a third person in favor of the corporation.
- Similarly, if a corporation has a defense to an action against it and is not asserting it, a stockholder may intervene and defend on
behalf of the corporation.
- For a derivative suit to prosper, it is required that the minority stockholder suing for and on behalf of the corporation must allege
in his complaint that he is suing on a derivative cause of action on behalf of the corporation and all other stockholders similarly
situated who may wish to join him in the suit.
- In the criminal complaint filed by herein respondent, nowhere is it stated that she is filing the same in behalf and for the benefit of
the corporation. Thus, the criminal complaint including the civil aspect thereof could not be deemed in the nature of a derivative suit.
- However, since the subject of the falsification was the corporation’s project and the falsified documents were corporate documents,
the corporation is a proper party in the petition for certiorari because the proceedings in the criminal case directly and adversely
affected the corporation.

59
CASE DIGEST | Corporation Law (2019) Atty. Villanueva-Castro ib3h

55. EXPERT TRAVEL & TOURS, INC v. CA and KOREAN AIRLINES


G.R. NO. 152392
May 26, 2005

FACTS
● Korean Airlines (KAL) is a corporation established and registered in the South Korea and licensed to do business in the Philippines.
Its general manager in the Philippines is Suk Kyoo Kim.
● On September 6, 1999, KAL, through Atty. Aguinaldo, filed a complaint against ETI with the RTC of Manila, for the collection of
the an amount of P260,150.00. Atty. Aguinaldo,who indicated therein that he was the resident agent and legal counsel of KAL and
had caused the preparation of the complaint.
● ETI filed a motion to dismiss the complaint on the ground that Atty. Aguinaldo was not authorized to execute the verification and
certificate of non-forum shopping as required.
● KAL opposed the motion, contending that Atty. Aguinaldo was its resident agent and was registered as such with the SEC and also
the corporate secretary of KAL.
● Atty. Aguinaldo claimed that he had been authorized to file the complaint through a resolution of the KAL Board of Directors
approved during a special meeting held on June 25, 1999. This is further bolstered by an affidavit executed by Suk Kyoo Kim, who
alleged that the BOD had a special teleconference meeting that day, which authorized Atty. Aguinaldo to execute the certificate.
● RTC denied the motion to dismiss; it gave credence to the assertions of Kim and Atty. Aguinaldo
● Petitioners appealed to CA; CA dismissed appeal, saying that RTC cannot be faulted for taking judicial notice of the
teleconferencing where the authorization was made.

ISSUE: WN there

HELD:
- In the certification filed before the court, there was no allegation that Atty. Aguinaldo had been authorized to execute the certificate
of non-forum shopping by the respondent’s Board of Directors; moreover, no such board resolution was appended thereto or
incorporated therein.
- While Atty. Aguinaldo is the resident agent of the respondent in the Philippines, this does not mean that he is authorized to execute
the requisite certification against forum shopping.
- The corporation, such as the petitioner, has no powers except those expressly conferred on it by the Corporation Code and those
that are implied by or are incidental to its existence. In turn, a corporation exercises said powers through its board of directors and/or
its duly-authorized officers and agents. Physical acts, like the signing of documents, can be performed only by natural persons duly-
authorized for the purpose by corporate by-laws or by specific act of the board of directors.
- In this age of modern technology, the courts may take judicial notice that business transactions may be made by individuals through
teleconferencing.
- Teleconferencing is interactive group communication through an electronic medium. This type of group communication may be
used in a number of ways, and have three basic types: (1) video conferencing—television-like communication augmented with sound;
(2) computer conferencing—printed communication through keyboard terminals, and (3) audio-conferencing—verbal
communication via the telephone with optional capacity for telewriting or telecopying.
- In the case at bar, the courts are not convinced that one is conducted; even if there had been one, the Court is not inclined to believe
that a board resolution was duly passed specifically authorizing Atty. Aguinaldo to file the complaint and execute the required
certification against forum shopping. Reasons: 1) there was no allegation made regarding the special teleconference meeting in the
complaint nor in the certification; 2) the resolution was not appended originally in the certification.

60
CASE DIGEST | Corporation Law (2019) Atty. Villanueva-Castro ib3h

56. GONZALES v. PNB


G.R. NO. L-33320
May 30, 1983

FACTS
● Petitioner Ramon Gonzales instituted this complaint before the Manila CFI Court of First Instance of Manila a special civil action
for mandamus against the herein respondent praying that the latter be ordered to allow him to look into the books and records of
the respondent bank in order to satisfy himself as to the truth of the published reports on some obligations which the PNB either
financed or guaranteed.
● Petitioner alleged that his written request was denied by PNB.
● This petition was due to the fact that the trial court dismissed the petition for mandamus.

ISSUE: WN petitioner has the right to inspect books and records of PNB.

HELD:
- No. It is now expressly required as a condition for such examination (in the Corporation Code) that the one requesting it must not
have been guilty of using improperly any information secured through a prior examination, and that the person asking for such
examination must be “acting in good faith and for a legitimate purpose in making his demand.”
- Although the petitioner has claimed that he has justifiable motives in seeking the inspection of the books of the respondent bank,
he has not set forth the reasons and the purposes for which he desires such inspection, except to satisfy himself as to the truth of
published reports regarding certain transactions entered into by the respondent bank and to inquire into their validity. The
circumstances under which he acquired one share of stock in the respondent bank purposely to exercise the right of inspection do not
argue in favor of his good faith and proper motivation.
- Admittedly he sought to be a stockholder in order to pry into transactions entered into by the respondent bank even before he
became a stockholder. He could have been impelled by a laudable sense of civic consciousness, but it could not be said that his
purpose is germane to his interest as a stockholder.
- Furthermore, to allow inspection sought by the petitioner would be violative of the provisions of its charder (RA No. 1300), which
provides that the PNB “shall not reveal to any person other than the President of the Philippines, the Secretary of Finance, and the
Board of Directors the details of the inspection or investigation, nor shall they give any information relative to the funds in its custody
x x x”
- It is therefore not correct to claim, therefore, that the right of inspection under Section 74 of the new Corporation Code may apply
in a supplementary capacity to the charter of the respondent bank.

61
CASE DIGEST | Corporation Law (2019) Atty. Villanueva-Castro ib3h

57. FEDERATED LPG ASSOCIATION v. DEL ROSARIO


G.R. NO. 202639
November 9, 2016

FACTS
- Petitioner, through counsel Atty. Genesis M. Adarlo sought assistance from the Criminal Investigation and Detection Group, Anti-
Fraud and Commercial Crimes Division (CIDG-AFCCD) of the PNP in the surveillance, investigation, apprehension, and
prosecution of certain persons and establishments for acts violative of BP 33 for refilling LPG cylinders without any written
authorization, underfilling LPG cylinders, and refilling LPG cylinders without giving any receipt therefor.
- CIDG affirmed that ACCS Ideal Gas Corporation has been refilling branded LPG cylinders in its refilling plant at G. Araneta
Avenue, Quezon City.
- Based on reasonable grounds to believe that ACCS was in violation of BP 33 after a test-buy, P/Supt. Esguerra filed with the Manila
RTC applications for search warrant, which was implemented.
- Complaints-Affidavits were filed against Antonio Del Rosario and respondents for violation of BP 33, as amended.
- Antonio, the General Manager of ACCS, while admitting that fact, denied that the company is engaged in illegal trading and
underfilling. Respondents, on the other hand, averred that they are merely incorporators/stockholders of ACCS who have no active
participation in the operation, management and control of the business.
- P/Spt. Esguerra replied, stressing that pursuant to Sec. 4, BP 33, the President, General Manager, Managing Partner, or such other
officer charged with the management of the business affairs of the corporation, or the employee responsible for the violation shall
be criminally liable.
- Antonio, being, the GM, is criminally liable. Respondents, he averred, are likewise liable because they were listed in the AOI as
incorporators, and thus deemed as directors of ACCS. He cited the By-Laws of ACCS and Sec. 23 of the Corporation Code that all
business shall be conducted and all property of the corporation controlled and held by the BOD.
- DOJ only found probable cause against Antonio; CA sustained the SOJ.

ISSUE: WN respondents, as members of the Board of Directors of ACCS, be criminally prosecuted for violation of BP 33.

HELD
- No.
- The CA ratiocinated that by the election or designation of Antonio as General Manager of ACCS, the daily business operations of
the corporation were vested in his hands and had ceased to be the responsibility of respondents as members of the Board of Directors.
Respondents, therefore, were not officers charged with the management of the business affairs who could be held liable pursuant to
paragraph 3, Section 4 of BP 33.
- Sec. 4, BP 33 provides,
“When the offender is a corporation, partnership, or other juridical person, the president, the general manager, managing
partner, or such other officer charged with the management of the business affairs thereof or employee responsible for the
violation shall be criminally liable. x x x”
- A member of the Board of Directors of a corporation, cannot, by mere reason of such membership, be held liable for the
corporation’s probable violation of BP 33.
- If one is not the President, General Manager or Managing Partner, it is imperative that it first be shown that he/she falls under the
catch-all “such other officer charged with the management of the business affairs,” before s/he can be prosecuted.
- The matter of being an officer charged with the management of the business affairs is a factual issue which must be alleged and
supported by evidence. Here, there is no dispute that neither of the respondents was the President, General Manager, or Managing
Partner of ACCS. There were no allegation of that sort.
- The Board only concerns itself in the business affairs by setting administrative and operational policies. It is actually the President
under Section 2, Article IV of the said bylaws who is vested with wide latitude in controlling the business operations of the
corporation.
- Respondents who were mere members of the Board of Directors and not shown to be charged with the management of the business
affairs were thus correctly dropped as respondents in the complaints.

62
CASE DIGEST | Corporation Law (2019) Atty. Villanueva-Castro ib3h

58. AGDAO LANDLESS RESIDENTS ASSOC. v. MARAMION


G.R. NO. 188642
October 17, 2016

FACTS
● Dakudao & Sons, Inc. (Dakudao) executed six Deeds of Donation in favor of petitioner Agdao Landless Residents Association,
Inc. (ALRAI) covering 46 titled lots (donated lots). One Deed of Donation prohibits ALRAI, as donee, from partitioning or
distributing individual certificates of title of the donated lots to its members, within a period of five years from execution, unless a
written authority is secured from Dakudao. The Board of directors and stockholders meetings held on January 5, 2000 and January
9, 2000, respectively, members of ALRAI resolved to directly transfer 10 of the donated lots to individual members and
nonmembers of ALRAI.
● Respondents filed a Complaint against petitioners. Respondents alleged that petitioners expelled them as members of ALRAI, and
that petitioners are abusing their powers as officers, among others by 1) partially distributing the donated lands, 2) illegally
expelling them as members of ALRAI without due process, and 3) unable to show the books of accounts of ALRAI.
● Petitioners alleged that ALRAI transferred lots to Alcantara as attorney’s fees ALRAI owed to her late husband, who was the legal
counsel of ALRAI; and Javonillo and Armentano as president and secretary of ALRAI, made a lot of sacrifices for ALRAI, and
that Dela Cruz provided financial assistance to ALRAI; and that respondents have no personality to sue, being removed due to their
absences in meetings.
● RTC ruled in favor of respondents; the court a quo treated the case as an intra-corporate dispute. It found respondents to be bona
fide members of ALRAI, and their expulsion was without due process, and annulled all transfers of the donated lands. CA affirmed
with modification upholding the transfers of Dela Cruz and Loy (buyer of Alcantara).

ISSUE: Whether the dismissal is valid; and whether the respondents have personality to sue; and whether the transfer of the lots are
valid

HELD
Re: dismissal of the members from the association and its concomitant right to inspect records
- Both courts found that in terminating respondents from ALRAI, petitioners deprived them of due process. Petitioners failed to substantiate the
allegations of nonpayment of membership dues and failure to attend meetings as a basis for their termination. Even assuming that petitioners were
able to prove these allegations, the automatic termination of respondents’ membership in ALRAI is still not warranted, as it is not a ground for their
automatic termination of membership.
- Moreover, the notice for July 29, 2001 meeting for the termination of membership was signed by President only 2 days beforehand, in violation
of the three-day notice requirement, as required in ALRAI’s by-laws. The petitioners have not adduced evidence showing that the expelled members
were notified of any meeting that they were given the opportunity to be heard.
- Clearly, members proved to be in arrears in the payment of monthly dues, contributions, or assessments shall only be automatically suspended;
while members who shall be absent from any meeting without any justifiable cause shall only be liable for a fine. Nowhere in the ALRAI
Constitution does it say that the foregoing actions shall cause the automatic termination of membership.
- There being no valid termination of respondents’ membership in ALRAI, respondents remain as its existing members and entitled to inspect the
records and demand accounting of its funds.

Re: Respondents’ personality to sue


- A derivative suit, is one which is instituted by a shareholder or a member of a corporation, for and in behalf of the corporation for its protection
from acts committed by directors, trustees, corporate officers, and even third persons. The whole purpose of the law authorizing a derivative suit is
to allow the stockholders/members to enforce rights which are derivative (secondary) in nature, i.e., to enforce a corporate cause of action.
- In strict sense, the first cause of action, and the reliefs sought, should have been brought through a derivative suit. The first cause of action pertains
to the corporate right of ALRAI involving its corporate properties which it owned by virtue of the Deeds of Donation. In derivative suits, the real
party-in-interest is the corporation, and the suing stockholder is a mere nominal party.
- However, the court treated this case as one pursued by the corporation itself, because:
1) The court a quo has jurisdiction to hear and decide the controversy, RA 8799, in relation to PD 902-A involving devices or schemes
employed by x x x the board of directors x x x which may be detrimental to the public and/or x x x members of associations or
organizations registered with the Commission;
2) The petitioners did not object to the institution of the case;
63
CASE DIGEST | Corporation Law (2019) Atty. Villanueva-Castro ib3h
3) The respondents prayed for reliefs for the benefit of ALRAI;
- The reliefs seek to preserve them for the corporate interest of ALRAI. Clearly then, any benefit that may be recovered is
accounted for, not in favor of respondents, but for the corporation, who is the real party-in-interest. Therefore, the occasion for
the strict application of the rule that a derivative suit should be brought in order to protect and vindicate the interest of the
corporation does not obtain under the circumstances of this case.
4) There is substantial compliance with the requirements of a derivative suit: a) party should be a shareholder as of the time the act or
transaction complained of; b) he has tried to exhaust intra-corporate remedies; and c) cause of action actually devolves on the corporation.
- While it is true that the complaining stockholder must show that he has exhausted all the means within his reach to attain
within the corporation the redress for his grievances, demand is unnecessary if the exercise will result in futility. To the Court,
the threat of expulsion against respondents is sufficient to forestall any expectation of further demand for relief from petitioners.

Re: Validity of the transfer of lots


- One of the objectives of ALRAI, considering its primary purpose of assistance in uplifting and promoting better living conditions to all members,
and the public in general is “to uplift and promote better living condition, “x x xby providing its members humble shelter and decent housing.”
- The Corporation Code provides that the power of a corporation to validly grant or convey any of its real or personal properties is circumscribed
by its primary purpose. It is therefore important to determine whether the grant or conveyance is pursuant to a legitimate corporate purpose, or is at
least reasonable and necessary to further its purpose.
- The Court held that the transfers of corporate properties to Javonillo, Armentano, Dela Cruz, Alcantara, and others are bereft of any corporate
purpose. Reasons:
1) Re: Dela Cruz - There is want of evidence whether the amount of the financial help that Dela Cruz extended to ALRAI is commensurate
to the amount of the property transferred to him. The lack of evidence on this point is prejudicial to ALRAI because ALRAI had parted
with its property without any means by which to determine whether the transfer is fair and reasonable under the circumstances.
2) Re: Alcantara - In this case however, petitioners did not substantiate the extent of the services that Atty. Pedro Alcantara rendered for
ALRAI. In fact, no engagement or retainer contract was ever presented to prove the terms of their agreement, neither the value of the
properties transferred, and hence there is no basis to determine whether the transfer of property is reasonable under the cir cumstances.
The subsequent sale of this property is likewise void, for one cannot sell a property over which he did not have the right to own, in the
first place.
3) Re: Javonillo and Armentano - Reasons such as raising money to meet expenses, and representation to BIR to waive the donor’s tax do
not suffice to prove any legitimate corporate purpose in the transfer of properties. It cannot be said that it was a compensation, for it is
well-settled that directors of corporations presumptively serve without compensation; so that while the directors, in assigning themselves
additional duties, act within their power, they nonetheless act in excess of their authority by voting for themselves compensation for such
additional duties. Even then, aside from the claim of petitioners, there is no showing that they rendered extraordinary or unusual services
to ALRAI.

- Being those entrusted with the management of its affairs, the directors or trustees and other officers of a corporation occupy a fiduciary relation
towards it, and cannot be allowed to contract with the corporation, directly or indirectly, or to sell property to it, or purchase property from it, where
they act both for the corporation and for themselves. Consequently, Sec. 32 of the Corporation Code requires that contracts involving the corporation
and directors/officers/trustees should be ratified by a vote representing at least two-thirds of the members in a meeting called for the purpose.
- The records do not show that transfers to the petitioners’ officers comply with the above provision for there was no allegation that a resolution
was passed to that effect; that there was disclosure of adverse interest; and that the transfer was fair and reasonable.
- These reasons show that the transfers to Javonillo and Armentano, the officers of ALRAI, as such, are void.

64
CASE DIGEST | Corporation Law (2019) Atty. Villanueva-Castro ib3h

59. HALLEY v. PRINTWELL


G.R. NO. 157549
May 30, 2011

FACTS
- The petitioner was an incorporator and original director of Business Media Philippines, Inc. (BMPI), which, at its incorporation on
November 12, 1987, had an authorized capital stock of P3,000,000.00 divided into 300,000 shares of which 75,000 were initially
subscribed.
- BMPI commissioned Printwell for the printing of magazine Philippines, Inc. BMPI
placed with Printwell several orders on credit, evidenced by invoices
and delivery receipts totaling P316,342.76. Considering that BMPI
paid only P25,000.00, Printwell sued BMPI on January 26, 1990 for
the collection of the unpaid balance of P291,342.76 in the RTC.
- Printwell amended the complaint to implead the defendants all the original stockholders to recover on their unpaid subscriptions.
- Defendants averred that they all had paid their subscriptions in full; that BMPI had a separate personality from those of its
stockholders; that Rizalino C. Viñeza; and that the directors and stockholders of BMPI had resolved to dissolve BMPI during the
annual meeting held on February 5, 1990.
- RTC ruled in favor of Printwell applying the trust fund doctrine, holding the defendant stockholders liable to Printwell pro rata. CA
affirmed RTC, and pierced to corporate veil to make the stockholders liable.

ISSUE: WN the unpaid stockholders shall be held liable for the obligation of the corporation

HELD:
- Yes.
- The trust fund doctrine enunciates a rule that the property of a corporation is a trust fund for the payment of creditors, but such
property can be called a trust fund ‘only by way of analogy or metaphor.’ As between the corporation itself and its creditors it is a
simple debtor, and as between its creditors and stockholders its assets are in equity a fund for the payment of its debts.
- Subscriptions to the capital of a corporation constitute a fund to which creditors have a right to look for satisfaction of their claims
and that the assignee in insolvency can maintain an action upon any unpaid stock subscription in order to realize assets for the
payment of its debts.
- The creditor is allowed to maintain an action upon any unpaid subscriptions and thereby steps into the shoes of the corporation for
the satisfaction of its debt. To make out a prima facie case in a suit against stockholders of an insolvent corporation to compel them
to contribute to the payment of its debts by making good unpaid balances upon their subscriptions, it is only necessary to establish
that the stockholders have not in good faith paid the par value of the stocks of the corporation.
- Ostensibly, therefore, the petitioner’s mere submission of the receipt issued in exchange of the check did not satisfactorily
establish her allegation of full payment of her subscription. Indeed, she could not even inform the trial court about the identity of her
drawee bank, and about whether the check was cleared and its amount paid to BMPI.
- However, RTC is erroneous in prorating the liability. The prevailing rule is that a stockholder is personally liable for the financial
obligations of the corporation to the extent of his unpaid subscription.

65
CASE DIGEST | Corporation Law (2019) Atty. Villanueva-Castro ib3h

60. NESTOR CHING v. SUBIC BAY GOLF AND COUNTRY CLUB, INC.
G.R. No. 174353
September 10, 2014

FACTS
● Petitioners Nestor Ching and Andrew Wellington filed a Complaint with the RTC of Olongapo City on behalf of the members of
Subic Bay Golf and Country Club, Inc. (SBGCCI) against the said country club and its Board of Directors and officers, impleading
its President, Treasurer, Corporate Secretary, and directors.
● Petitioners aver that the corporation sold shares to them at US$ 20,000 per share, and they were presented SBGCCI’s AOI stating
that no dividends shall incur in favor of the members and shareholders are only entitled to pro rata share of the assets upon the
corporation’s dissolution. They allege that an amendment to the AOI was not disclosed to them, stating that shareholders shall not
have any proprietary right over the properties of the corporation. This, in petitioners view, is in fraud of its stockholders.
● Furthermore, petitioners aver that the Board of Directors did not call any stockholders’ meeting in violation of Sec. 50 of the
Corporation Code, neither they presented any financial statements in violation of Sec. 75, CC. These, alongside with other acts of
fraud alleged by the petitioners, were made as a bases to pray for enjoining the Board of Directors from acting as such.
● The RTC dismissed the complaint, citing as a basis that they failed to exhaust their remedies within the corporation itself, and that
they only have two shares altogether, comprising 0.24% of the outstanding shares of the corporation, an indication that it is a
nuisance suit. Therefore, they did not comply with the requirements of a valid derivative suit.
● Petitioners appealed to CA, which affirmed the RTC decision.

ISSUE: WN there is a valid derivative suit

HELD:
- No. The petition is unmeritorious.
- While the caption of the Complaint also names the “Subic Bay Golfers and Shareholders, Inc. for and in behalf of all its members,”
petitioners did not attach any authorization from said alleged corporation or its members to file the Complaint. Thus, the Complaint
is deemed filed only by petitioners and not by SBGSI.
- The reliefs sought in the Complaint, namely that of enjoining defendants from acting as officers and Board of Directors of the
corporation, the appointment of a receiver, and the prayer for damages in the amount of the decrease in the value of the shares of
stock, clearly show that the Complaint was filed to curb the alleged mismanagement of SBGCCI, and therefore accrue to the
corporation itself.
- Petitioners’ only possible cause of action as minority stockholders against the actions of the Board of Directors is the common law
right to file a derivative suit. The legal standing of minority stockholders to bring derivative suits is not a statutory right, there being
no provision in the Corporation Code or related statutes authorizing the same, but is instead a product of jurisprudence based on
equity. However, it cannot prosper without complying with the requirements set under Sec. 1, Rule 8 of Interim Rules on Intra-
corporate controversies, to wit:
(1) He was a stockholder or member at the time the acts or transactions subject of the action occurred and at the time the action was filed;
(2) He exerted all reasonable efforts, and alleges the same with particularity in the complaint, to exhaust all remedies available under the
articles of incorporation, bylaws, laws or rules governing the corporation or partnership to obtain the relief he desires;
(3) No appraisal rights are available for the act or acts complained of; and
(4) The suit is not a nuisance or harassment suit.
- The Court do not agree with the trial court that petitioners failed to comply with the fourth requisite. Although petitioners account
for only two shares from the outstanding capital stock, it is enough to file a derivative suit in favor of the corporation.
- Anent the second requisite, petitioners failed to state with particularity in the Complaint that they had exerted all reasonable efforts
to exhaust all remedies available under the articles of incorporation, by-laws, and laws or rules governing the corporation to obtain
the relief they desire. Indeed, even if petitioners thought it was futile to exhaust intra-corporate remedies, they should have stated the
same in the complaint, as well as the reasons for such opinion. Failure to state either will render the court, motu proprio or upon
motion, dismissing the complaint.

66
CASE DIGEST | Corporation Law (2019) Atty. Villanueva-Castro ib3h

VI. SUBSCRIPTION CONTRACT

61. LIM TAY v. CA


G.R. NO. 126891
August 5, 1998

FACTS
● Respondent-Appellee Sy Guiok secured a loan from the petitioner in the amount of P40,000 payable within six (6) months. To
secure the payment of the aforesaid loan and interest thereon. Sy Lim also secured a loan from petitioner. Both contracted a Contract
of Pledge with the petitioner pledging their respective shares to the respondent corporation, Go Fay & Co., Inc by which Respondent
Guiok and Sy Lim endorsed their respective shares of stock in blank and delivered the same to the petitioner.
● For failure to pay their respective loans, on October, 1990, the petitioner filed a ‘Petition for Mandamus’ against Go Fay & Co.,
Inc. with the SEC praying to direct the corporate secretary for the registration of the stock transfer and issue new certificates to
Lim Tay.
● The SEC hearing officer dismissed the complaint, holding that he failed to prove the legal basis for the secretary of the to be
compelled to register stock transfers in favor of the petitioner and to issue new certificates of stock under his name.
● CA also ruled against the petitioner, holding that petitioner failed to establish his clear and legal right to the writ of mandamus
prayed by him.

ISSUE: Whether the petitioner is entitled to the relief of mandamus as against the respondent Go Fay & Co., Inc. for the transfer of
pledged shares

HELD:
- No. The petition is unmeritorious.
- In order that a writ of mandamus may issue, it is essential that the person petitioning for the same has a clear legal right to the thing
demanded and that it is the imperative duty of the respondent to perform the act required. It is simply a command to exercise a power
already possessed and to perform a duty already imposed.
- In the present case, petitioner has failed to establish a clear legal right, for the petitioner does not have any ownership rights at all.
- At the outset, it must be underscored that petitioner did not acquire ownership of the shares by virtue of the contracts of pledge.
There is no showing that petitioner made any attempt to foreclose or sell the shares through public or private auction, as st ipulated
in the contracts of pledge and as required by Article 2112 of the Civil Code. Hence, the pledgor remains the owner of the pledged
shares.
- Anent the alleged ownership by prescription, the prescriptive period within which to demand the return of the thing pledged should
begin to run only after the payment of the loan and a demand for the thing has been made, because it is only then that respondents
acquire a cause of action for the return of the thing pledged, but no demand was made.
- Neither novation can vest the petitioner ownership, as it cannot be presumed by (a) respondents’ indorsement and delivery of the
certificates of stock covering the 600 shares (as this is delivery is expressly stipulated in the pledge contract), (b) petitioner’s receipt
of dividends from 1980 to 1983, and (c) the fact that respondents have not instituted any action to recover the shares since 1980.
- The duty of a corporate secretary to record transfers of stocks is ministerial. However, he cannot be compelled to do so when the
transferee’s title to said shares has no prima facie validity or is uncertain. More specifically, a pledgor, prior to foreclosure and sale,
does not acquire ownership rights over the pledged shares and thus cannot compel the corporate secretary to record his alleged
ownership of such shares on the basis merely of the contract of pledge.

67
CASE DIGEST | Corporation Law (2019) Atty. Villanueva-Castro ib3h

62. RURAL BANK OF LIPA v. CA


G.R. NO. 124535
September 28, 2001

FACTS
● Reynaldo Villanueva, Sr., a stockholder of the Rural Bank of Lipa City, executed a Deed of Assignment, wherein he assigned his
shares, as well as those of eight (8) other shareholders under his control with a total of 10,467 shares, in favor of the stockholders
of the Bank represented by its directors.
● Villanueva, with his spouse acknowledge a debt of P4,000,000 to the bank, with which the Villanueva spouses assured the Board
that their debt would be paid on or before December 31 of that same year; otherwise, the Bank would be entitled to liquidate their
shareholdings.
● When the Villanueva spouses failed to settle their obligation to the Bank on the due date, the Board sent the Bank on the due date,
the Board sent them a letter demanding: (1) the surrender of all the stock certificates issued to them; and (2) the delivery of sufficient
collateral to secure the balance of their debt amounting to P3,346,898.54
● Villanuevas ignored the bank’s demands, hence their shares of stock were converted into Treasury Stocks. Later, the Villanuevas,
through their counsel, questioned the legality of the conversion of their shares. They also appealed to the corporation the fact that
they were not notified of the consequent 1994 stockholders’ meeting and election of directors and officers.
● Villanueva spouses filed with the Securities and Exchange Commission (SEC), a petition for annulment of the stockholders’
meeting and election of directors and officers on January 15, 1994, with damages and prayer for preliminary injunction, and was
joined with other co-petitioners.
● Their contentions are: (1) they were conducted in violation of the by-laws of the Rural Bank; (2) they were not given due notice of
said meeting and election; (3) they were deprived of their right to vote despite their being holders of common stock; (4) their names
were irregularly excluded from the list of stockholders; and (5) the candidacy of petitioner Avelina Villanueva for directorship was
arbitrarily disregarded during the said meeting.
● SEC hearing officer granted the injunction, and also granting the Omnibus Motion to suspend the 1995 annual meeting. Petitioners
filed certiorari with the SEC en banc, which it denied. CA also denied the appeal, citing Sec. 63 of the Corporation Code.

ISSUE: Whether or not there is a valid transfer of shares to the bank

HELD:
- No. Sec. 63 of the Corporation Code provides:
“x x x Shares of stocks so issued are personal property and may be transferred by delivery of the certificate or certificates indorsed by
the owner or his attorney-in-fact or other person legally authorized to make the transfer. No transfer, however, shall be valid, except as
between the parties, until the transfer is recorded in the books of the corporation so as to show the names of the parties to the transaction,
the date of the transfer, the number of the certificate or certificates and the number of shares transferred. x x x”
- For a valid transfer of stocks, there must be strict compliance with the mode of transfer prescribed by law. The requirements are:
(a) There must be delivery of the stock certificate; (b) The certificate must be endorsed by the owner or his attorney-in-fact or other
persons legally authorized to make the transfer; and (c) To be valid against third parties, the transfer must be recorded in the books
of the corporation. As it is, compliance with any of these requisites has not been clearly and sufficiently shown.
- While the assignment may be valid and binding on the petitioners and private respondents, it does not necessarily make the transfer
effective. Therefore, the private respondents cannot, as yet, be deprived of their rights as stockholders, until and unless the issue of
ownership and transfer of the shares in question is resolved with finality. The SEC Hearing Officer therefore did not commit
GADALEJ in issuing the injunction.

68
CASE DIGEST | Corporation Law (2019) Atty. Villanueva-Castro ib3h

63. PONCE v. ALSONS CEMENT


G.R. NO. 139802
December 10, 2002

FACTS
● On January 25, 1996, petitioner Vicente C. Ponce, filed a complaint with the SEC for mandamus and damages against defendants
Alsons Cement Corporation and its corporate secretary Francisco M. Giron praying that judgment be rendered ordering respondents
(a) to issue in his name certificates of stocks covering the 239,500 shares of stocks and its legal increments and (b) to pay him
damages.
● He alleged that he and Fausto Gaid executed a “Deed of Undertaking” and “Indorsement” whereby the latter acknowledges that
the former is the owner of said shares and he was therefore assigning/endorsing the same to Ponce. From the time of incorporation
of VCC, and the indorsement (1968) up the present (1996), no certificates of stock corresponding to the 239,500 subscribed and
fully paid shares of Gaid were issued in the name of Fausto G. Gaid and/or Ponce, and despite repeated demands, the corporation
refuse to issue certificates representing such shares in violation of his right to secure a certificate in his name.
● SEC Hearing Officer dismissed the petition on motion of the defendant, due to the fact that no transfer was recorded, and that there
was no certificate of stock to speak of; SEC en banc reversed the Hearing Officer and instructed to proceed with the case.
Respondents appealed to the CA, which it held that in the absence of any allegation that the transfer of the shares between Fausto
Gaid and Vicente C. Ponce was registered in the stock and transfer book of ALSONS, Ponce failed to state a cause of action.

ISSUE: WN petitioner has a cause of action for the writ of mandamus

HELD:
- No. While it is undisputed that Gaid is an incorporator and original subscriber of the said shares, it is also undisputed that petitioner
had not made a previous request to the corporate secretary of Alsons to record the alleged transfer of shares.
- Sec. 63 of the Corporation Code provides:
“No transfer, however, shall be valid, except as between the parties, until the transfer is recorded in the books of the corporation so as
to show the names of the parties to the transaction, the date of the transfer, the number of the certificate or certificates and the number
of shares transferred.”
- A transfer of shares of stock not recorded in the stock and transfer book of the corporation is non-existent as far as the non-existent
as far as the corporation is concerned. The corporation looks only to its books for the purpose of determining who its shareholders
are. It is only when the transfer has been recorded in the stock and transfer book that a corporation may rightfully regard the transferee
as one of its stockholders. Unless and until such recording is made the demand for the issuance of stock certificates to the alleged
transferee has no legal basis.
- A petition for mandamus fails to state a cause of action where it appears that the petitioner is not the registered stockholder and
there is no allegation that he holds any power of attorney from the registered stockholder, from whom he obtained the stocks, to
make the transfer, the deed of undertaking with indorsement presented by petitioner does not establish, on its face, his right to demand
for the registration of the transfer and the issuance of certificates of stocks.
- Therefore, absent an allegation that the transfer of shares is recorded in the stock and transfer book of respondent ALSONS, there
appears no basis for a duty or clear legal obligation that can be imposed upon the respondent corporate secretary, so as to justify the
issuance of the writ of mandamus to compel him to perform the transfer of the shares to petitioner. At this point, an essential requisite
of a writ of mandamus is missing.
- Nevertheless, considering the law does not prescribe a period within which the registration must be effected, the action to enforce
the right does not accrue until there has been a demand and a refusal concerning the transfer. In the present case, petitioner’s complaint
for mandamus must fail, not because of laches or estoppel, but because he had alleged no cause of action sufficient for the issuance
of the writ.

69
CASE DIGEST | Corporation Law (2019) Atty. Villanueva-Castro ib3h

64. ONG YONG v. TIU


G.R. NO. 14476 & 144629
April 8, 2003

RESOLUTION
FACTS
● Masagana Citimall is a commercial complex owned and managed by the First Landlink Asia Development Corporation (FLADC),
fully owned by the Tius. order to recover from its floundering finances, the Ong Group composed of Ong Yong, were invited by
the Tius to invest in FLADC.
● By the Pre-Subscription Agreement, both parties agreed to maintain equal shareholdings in FLADC with the Ongs investing cash
while the Tius contributing property. Furthermore, the parties agreed that 6 directors of FLADC were to be nominated from the
Ong Group, while 5 directors thereof were to be nominated from the Tiu Group.
● The controversy between the two parties arose when the Ongs refused to credit the number of FLADC shares in the name of
Masagana Telamart, Inc. commensurate to its 1,902.30 square meter property contribution; also when they refused to credit the
number of FLADC shares in favor of the Tius commensurate to their 151 square meter property contribution; and when David S.
Tiu and Cely Y. Tiu were proscribed from assuming and performing their duties as Vice-President and Treasurer, respectively of
FLADC
● Tius then sought the Securities and Exchange Commission (SEC) confirmation of their rescission of the Pre-Subscription
Agreement, to which the SEC Hearing Officer confirmed the rescission. Through an omnibus motion by the Ongs, the SEC Hearing
Officer reversed the decision, only insofar as the investment amounting to P70 M which is not part of the capital stock but an
indebtedness of the FLADC. SEC en banc affirmed its decision but reversed the ruling as to the P70 M. CA affirmed the SEC en
banc.
● The Supreme Court, in its 2002 decision, affirmed the CA ruling in favor of the respondents, hence, this MR.

ISSUE: WN respondents could legally rescind the Pre-Subscription Agreement

HELD:
- No. When the Tius invited the Ongs to invest in FLADC as stockholders, an increase of the authorized capital stock became
necessary to give each group equal (50-50) shareholdings as agreed upon in the Pre-Subscription Agreement. The authorized capital
stock was thus increased from 500,000 shares to 2,000,000 shares, with the Ongs subscribing to 1,000,000 shares and the Tius to
549,800 more shares in addition to their 450,200 shares to complete 1,000,000 shares. Thus, the subject matter of the contract was
the 1,000,000 unissued shares of FLADC stock allocated to the Ongs. Since these were unissued shares, the parties Pre-Subscription
Agreement was in fact a subscription contract as defined under Section 60 of the Corporation Code, to wit:
“Any contract for the acquisition of unissued stock in an existing corporation or a corporation still to be formed shall be
deemed a subscription within the meaning of this Title, notwithstanding the fact that the parties refer to it as a purchase or
some other contract”
- Thus, the subscription contract (denominated by the parties as a Pre-Subscription Agreement) was, from the viewpoint of the law,
one between the Ongs and FLADC, not between the Ongs and the Tius. Considering therefore that the real contracting parties to the
subscription agreement were FLADC and the Ongs alone, a civil case for rescission on the ground of breach of contract filed by the
Tius in their personal capacities will not prosper. Though FLADC was represented by the Tius in the subscription contract, FLADC
had a separate juridical personality from the Tius.
- Although the Tius were adversely affected by the Ongs unwillingness to let them assume their positions, rescission due to breach
of contract is definitely the wrong remedy for their personal grievances. The Corporation Code, SEC rules and even the Rules of
Court provide for appropriate and adequate intra-corporate remedies, other than rescission, in situations like this. Rescission is
certainly not one of them.
- Assuming arguendo that the Tius possess the legal standing to sue for rescission based on breach of contract, said action will
nevertheless still not prosper since rescission will violate the Trust Fund Doctrine and the procedures for the valid distribution of
assets and property under the Corporation Code.
- WHEREFORE, the 2002 decision by the Court is reversed.

70
CASE DIGEST | Corporation Law (2019) Atty. Villanueva-Castro ib3h

65. ANDAYA v. RURAL BANK OF CABADBARAN


G.R. NO. 188769
August 3, 2016

FACTS
● Andaya bought from Chute 2,200 shares of stock in the Rural Bank of Cabadbaran for P220,000, which the former duly endorsed
and delivered the certificates of stock, and requested the respondent corporation to register the transfer and issue certificate of stock
in favor of petitioner. Andaya separately communicated to the respondent’s corporate secretary the request.
● The request of Chute was denied on ground that existing stockholders have the right of refusal on the shares ought to be sold, and
that if no other stockholder buys them, then she could proceed with the sale. On the other hand, the request of Andaya was denied
by the bank because of conflict of interest - that Andaya is the CEO and President of Green Bank of Caraga, a competitor bank.
● Petitioner then filed for mandamus before the Cabadbaran RTC, seeking to compel them to record the transfer in the bank’s stock
and transfer book and to issue new certificates of stock in his name.
● Cabadbaran RTC ruled that petitioner Andaya was not entitled to the remedy of mandamus, since the transfer of the subject shares
of stock had not yet been recorded in the corporation’s stock and transfer book, and the registered owner, Chute, had not given him
a special power of attorney to make the transfer, citing Ponce v. Alsons Cement Corporation.

ISSUE: Whether petitioner may file a petition for mandamus to compel the respondent to record the stock transfer and issue
certificates of stock

HELD:
- Yes. It is already settled jurisprudence that the registration of a transfer of shares of stock is a ministerial duty on the part of the
corporation. Aggrieved parties, a bona fide transferee, may then resort to the remedy of mandamus to compel corporations that
wrongfully or unjustifiably refuse to record the transfer or to issue new certificates of stock. This legal right inherently flows from
the transferee’s established ownership of the stocks.
- Transferees of shares of stock are real parties-in-interest having a cause of action for mandamus to compel the registration of the
transfer and the corresponding issuance of stock certificates.
- Consequently, the Court ruled that Andaya has been able to establish that he is a bona fide transferee of the shares of stock of Chute,
as proven by the documentary evidence he presented, among others: 1) a notarized Sale of Shares of Stock covering the shares in
question; 2) a Documentary Stamp Tax Declaration; 3) Capital Gains Tax Return; and 4) the certificate of stock duly endorsed and
delivered to petitioner.
- There is no doubt that Andaya had the standing to initiate an action for mandamus to compel the Rural Bank of Cabadbaran to
record the transfer of shares in its stock and transfer book and to issue new stock certificates in his name. The Court’s ruling in Ponce
is unavailing, in view of undisputed documentary evidence presented by petitioner, together with the letter sent to Chute regarding
her request, an evidence that she herself requested the registration of the stock transfer.
- On whether mandamus shall lie in favor of petitioner, the Court remanded the case to RTC, as it involves factual matters that need
to be threshed out in the lower court.

71
CASE DIGEST | Corporation Law (2019) Atty. Villanueva-Castro ib3h

66. ANNA TENG v. SEC


G.R. NO. 184332
February 17, 2016

FACTS
● Ting Ping purchased 480 shares of TCL Sales Corporation (TCL) from Peter Chiu (Chiu), his brother Teng Ching Lay (Teng
Ching), who was also the president and operations manager of from Ismaelita Maluto.
● Upon Teng Ching’s death, his son Henry Teng (Henry) took over the management of TCL. To protect his shareholdings with TCL,
Ting Ping on August 31, 1989 requested TCL’s Corporate Secretary, herein petitioner Teng, to enter the transfer in the Stock and
Transfer Book of TCL for the proper recording of his acquisition, to which the latter refused.
● Ting Ping then filed for petition for mandamus in SEC which was granted; SEC en banc affirmed the decision; CA dismissed
petition; and finally, in G.R. No. 127777 in 2001, also denied petitioner’s petition.
● By reason thereof, SEC issued a writ of execution, but Teng filed an interpleader to settle the ownership of shares previously owned
by Teng Ching, as to which the RTC held partially in favor of Henry as the owner of such shares. Thereafter, an Ex Parte Motion
for the issuance of Alias Writ of Execution was filed by Ting Ping to enforce the recording of the shares he acquired from Chiu
and Maluto and also payment of damages as held by the Court. Teng moved to quash the motion on ground, among others that
there is discrepancy in the number of shares owned by Maluto transferred to Ting Ping. Ting Ping opposed the motion to quash,
manifesting its willingness to surrender the certificates from Chiu and Maluto to expedite the recording. Teng pointed out that the
opposition by Ting Ping did not include the certificates of stock, surmising that the certificates are either destroyed or lost.
● SEC denied to motion to quash; CA dismissed petition for certiorari by petitioners.

ISSUE: WN the surrender of the certificates of stock is a requisite before registration of the transfer may be made in the corporate
books and for the issuance of new certificates in its stead

HELD:
- No. Sec. 63 of the Corporation Code provides:
“ x x x Shares of stock so issued are personal property and may be transferred by delivery of the certificate or certificates indorsed by
the owner or his attorney-in-fact or other person legally authorized to make the transfer. x x x”
- Under Sec. 63 (when read as a whole), certain minimum requirements for the validity of transfer must be complied to wit: a)
delivery of the stock certificate; b) indorsement by the owner; and c) recording to the books of the corporation to be valid against
third persons. It is the delivery of the certificate, coupled with the endorsement by the owner or his duly authorized representative
that is the operative act of transfer of shares from the original owner to the transferee.
- Section 63, however, pertains to the delivery of the certificate of shares by the transferor to the transferee, that is, from the original
stockholder named in the certificate to the person or entity the stockholder was transferring the shares to, whether by sale or some
other valid form of absolute conveyance of ownership.
- It is thus clear that Teng’s position — that Ting Ping must first surrender Chiu’s and Maluto’s respective certificates of stock before
the transfer to Ting Ping may be registered in the books of the corporation does not have legal basis. It is not a requisite before the
conveyance may be recorded in its books.
- In ruling favorably for the private respondent, the Court stressed that a corporation, either by its board, its bylaws, or the act of its
officers, cannot create restrictions in stock transfers. In transferring stock, the secretary of a corporation acts in purely ministerial
capacity, and does not try to decide the question of ownership. He may even be compelled by mandamus.
- In the case at bar, G.R. No. 129777 upheld Ting Ping’s titles to the shares; and that petitioner cannot refuse to record the transfer
on grounds of discrepancy because the respective duties of the secretary to record the transfer is purely ministerial.
- However, it is necessary, for the issuance of the new certificates of shares in favor of transferee, that the original certificates of
stock be surrendered to the corporation.

72
CASE DIGEST | Corporation Law (2019) Atty. Villanueva-Castro ib3h

67. PHILIPPINE ASSOCIATED SMELTING AND REFINING CORPORATION v. LIM


G.R. NO. 172948
October 5, 2016

FACTS
● Philippine Associated Smelting and Refining Corporation (hereafter PASAR) is a corporation duly organized and existing engaged
in copper smelting and refining. Respondents Lim, et. al., on the other hand, are former senior officers and currently shareholders
of PASAR.
● Petition for Injunction and Damages with prayer for Preliminary Injunction and/or Temporary Restraining Order, dated February
4, 2004 was filed by PASAR seeking to restrain petitioners from demanding inspection of its confidential and inexistent records,
with which the RTC ruled in favor of PASAR, holding that right to inspect corporate records may be restricted. Thus, pending the
determination of which records are confidential or inexistent, the petitioners should be enjoined from inspecting the books.
● CA, on the other hand, held that there is no basis in issuing the injunctive writ, hence this petition.

ISSUE: Whether injunction properly lies to prevent respondents from invoking their right to inspect.

HELD:
- No; the petition is without merit.
- For an action for injunction to prosper, the applicant must show the existence of a right, as well as the actual or threatened violation
of this right. Thus, an injunction must fail where there is no clear showing of both an actual right to be protected and its threatened
violation, which calls for the issuance of an injunction.
- Corporation Code, in Sec. 74 thereof, provides that a stockholder has the right to inspect the records of all business transactions of
the corporation and the minutes of any meeting at reasonable hours on business days. The stockholder may demand in writing for a
copy of excerpts from these records or minutes, at his or her expense.
- However, Sec. 74 provides limitations in this right. These limitations are expressly provided as defenses in actions filed under
Section 74. Thus, this Court has held that a corporation’s objections to the right to inspect must be raised as a defense:
“ x x x further, That it shall be a defense to any action under this section that the person demanding to examine and copy excerpts from
the corporation’s records and minutes has improperly used any information secured through any prior examination of the records or
minutes of such corporation or of any other corporation, or was not acting in good faith or for a legitimate purpose in making his
demand.”
- The clear provision in Section 74 of the Corporation Code is sufficient authority to conclude that an action for injunction and,
consequently, a writ of preliminary injunction filed by a corporation is generally unavailable to prevent stockholders from exercising
their right to inspection. Therefore, stockholders cannot be prevented from gaining access to the records of all business transactions
of the corporation; and minutes of any meeting of stockholders or the board of directors, including their various committees and
subcommittees.
- Good faith and a legitimate purpose are presumed. It is the duty of the corporation to allege and prove with sufficient evidence the
facts that give rise to a claim of bad faith as to the existence of an illegitimate purpose. Specifically, corporations may raise their
objections to the right of inspection through affirmative defense in an ordinary civil action for specific performance or damages, or
through a comment (if one is required) in a petition for mandamus. The corporation or defendant or respondent still carries the burden
of proving (a) that the stockholder has improperly used information before; (b) lack of good faith; or (c) lack of legitimate purpose.

73
CASE DIGEST | Corporation Law (2019) Atty. Villanueva-Castro ib3h

VII. NON-STOCK CORPORATIONS

68. LONG v. BASA


G.R. NO. 134963-64
September 27, 2001

FACTS
● ‘The Church In Quezon City (Church Assembly Hall), Incorporated” (“CHURCH” for brevity), was organized as “an entity of the
brotherhood in Christ.
● As a “brotherhood in Christ,” the CHURCH embraced the “Principles of Faith” that “every member or officer” thereof “shall,
without mental reservation, adhere strictly to the doctrine, teaching and faith being observed by the (CHURCH) in proclaiming the
Gospel of Christ, to save lost souls, to lead men in worshipping the true God, in accordance with the Holy Bible.
● The members of the CHURCH vested upon the Board of Directors the absolute power absolute power “(to preserve and protect
the(ir) faith and expel a member of the CHURCH. Admission is so exacting, that only “persons zealous of the Gospel, faithful in
Church work and of sound knowledge of the Truth, as the Board of Directors shall admit to membership, shall be members of the
Church. The procedure for the expulsion of an erring or dissident member is prescribed in Article VII (paragraph 4) of the CHURCH
By-law which provides:
“If it is brought to the notice of the Board of Directors that any member has failed to observe any regulations and By-laws of
the Institution (CHURCH) or the conduct of any member has been dishonorable or improper or otherwise injurious to the
character and interest of the Institution, the Board of Directors may b(y) resolution without assigning any reason therefor expel
such member from such Institution and he shall then forfeit his interest, rights and privileges in the Institution.”
● Board of Directors observed that certain members of the CHURCH, including petitioners herein, exhibited “conduct which was dishonorable,
improper and injurious to the character and interest of the (CHURCH)” by “introducing (to the members) doctrines and teachings which were not
based on the Holy Bible” and the Principles of Faith embraced by the CHURCH. The Church advised these members to “correct their ways” and
warned them that if they persist in this highly improper conduct, they would be dropped from the church. These warnings were made since 1988,
but the petitioners ignored these admonitions.
● Thereafter, Board of Directors, during August 30, 1993 regular meeting held for the purpose of reviewing and updating the membership list of
the CHURCH, removed from the said list certain names of members, including petitioners herein, for espousing doctrines inimical or injurious
to the Principles of Faith of the CHURCH. Consequently, the updated membership list was submitted to the SEC.
● Petitioner then filed before the SICD seeking the annulment of the latest membership list, and seeking reinstatement on grounds that the expulsion
was done without prior notice and hearing, hence praying for a TRO to enjoin the BOD from usurping the positions.
● SEC Hearing Officer ruled that the expulsion was in order, done in accordance with the by-laws, to which the SEC en banc affirmed the same.
● Petitioner filed a separate case with the SEC regarding the expulsion itself, to which the SEC en banc set aside the expulsion made against the
petitioners. CA set aside this decision by the SEC en banc.

ISSUE: Whether the expulsion of petitioners from the membership of the CHURCH by its Board of Directors through a resolution issued on August
30, 1993 is in accordance with law.

HELD:
- Yes; the petition is unmeritorious.
- The issue of the validity of the expulsion had long been resolved and declared valid by the SEC en banc in its decision dated July
11, 1994 in SEC EB Case No. 389, to which they did not appeal from. Therefore, this decision becomes final and executory, and it
is a gross disregard to basic legal precepts to reverse later on the decision rendered by the same tribunal.
- The By-laws of the CHURCH, which the members have expressly adhered to, does not require the Board of Directors to give prior
notice to the erring or dissident members in cases of expulsion. This is evident from the procedure for expulsion prescribed in Article
VII (paragraph 4) of the By-laws. the only requirements therefore before any member may be expelled are: a) that the Board is
notified that a member has failed to observe the regulation of the Church and its By-laws, and b) a resolution is passed expelling the
member concerned.
- It is thus clear that a member who commits any of the causes for expulsion enumerated in paragraph 4 of Article VII may be
expelled by the Board of Directors, through a resolution, without giving that erring member any notice prior to his expulsion. The
resolution need not even state the reason for such action.

74
CASE DIGEST | Corporation Law (2019) Atty. Villanueva-Castro ib3h
- It must be stressed the peculiarity of a religious organization; that the relationship between a religious corporation and its members
is the latter’s absolute adherence to a common religious or spiritual belief. Once this basis ceases, membership in the religious
corporation must also cease. Thus, generally, there is no room for dissension in a religious corporation. Therefore, church action is
conclusive upon the civil courts. The Corporation Code leaves the matter of ecclesiastical discipline to the church authorities.
- Moreover, the petitioners really have no reason to bewail the lack of prior notice in the By-laws. As correctly observed by the Court
of Appeals, they have waived such notice by adhering to those By-laws by becoming members of the church voluntarily.
- Assuming the expulsion falls under the twin requirements of due notice and hearing, petitioners were given more than sufficient
notice of their impending expulsion, as shown by the records. Consequently, the expulsion was not tainted with any arbitrary
treatment from the members of the Board of Directors who, since 1988 up to August 30, 1993, or approximately five (5) years, have
patiently exhorted and warned the dissident members. This, according to the Court, constitutes as more than sufficient notice before
their expulsion in 1993.

75
CASE DIGEST | Corporation Law (2019) Atty. Villanueva-Castro ib3h

69. STA. CLARA HOMEOWNERS’ ASSOCIATION v. SPS. GASTON


G.R. NO. 141961
January 23, 2002

FACTS
● Private respondents herein were residents of San Jose Avenue, Sta. Clara Subdivision, Mandalagan, Bacolod City. They purchased
their lots in the said subdivision sometime in 1974, and at the time of purchase, there was no mention or requirement of membership
in any homeowners’ association.
● They have remained as non-members of SCHA, and non-members of the association were issued ‘non-member’ gatepass stickers
for their vehicles for identification by the security guards manning the subdivision’s entrances and exits.
● In Marcy 1998, SCHA disseminated a board resolution which decreed that only its members in good standing were to be issued
stickers for use in their vehicles. On separate incidents, son of private respondents were required to show his driver’s license by
the guards on duty despite knowing him personally, and also the private respondents were prevented from entering the subdivision
and proceeding to his residential abode when its security guards demanded him his driver’s license for identification. They alleged
that it had caused private respondents to suffer moral damage.
● Petitioners filed for motion to dismiss by reason of improper venue as it is a intra-corporate dispute; that its by-laws contains a
provision that ‘all real estate owners in Sta. Clara Subdivision automatically become members of the association.’
● RTC denied the motion to dismiss; CA dismissed also, holding that RTC had jurisdiction over the dispute.

ISSUE: WN private respondents are members of SCHA

HELD:
- No. Petition is unmeritorious.
- To answer the question of jurisdiction proffered by petitioners, the issue of membership of respondents was raised.
- The constitutionally guaranteed freedom of association includes the freedom not to associate. The right to choose with whom one
will associate oneself is the very foundation is and essence of that partnership. It should be noted that the provision guarantees the
right to form, an association. It does not include the right to compel others to form or join one.
- Private respondents cannot be compelled to become members of the SCHA by the simple expedient of including them in its Articles
of Incorporation and By-laws without their express or implied consent.
- Despite the mutual advantage of lot owners to band themselves together to promote their common welfare, this could only be
possible if the owners voluntarily agree, directly, or indirectly, to become members of the association.
- Other than the said Articles of Incorporation and By-laws, there is no showing that private respondents have agreed to be SCHA
members.

76
CASE DIGEST | Corporation Law (2019) Atty. Villanueva-Castro ib3h

70. PADCOM CONDOMINIUM CORPORATION v. ORTIGAS CENTER


G.R. NO. 146807
May 9, 2002

FACTS
● Petitioner Padcom Condominium Corporation (hereafter PADCOM) owns and manages the Padilla Office Condominium Building
(PADCOM Building) located at Emerald Avenue, Ortigas Center, Pasig City. Padcom acquired the land on which the building
stands under a Deed of Sale with Tierra Development Corporation (TDC)
● Among the terms in the Deed of Sale was the requirement that the transferee and its successor-in-interest must become members
of an association for realty owners and long-term lessees in the area later known as the Ortigas Center. The Association (Ortigas
Center Association, Inc) was organized in 1982. It sought the collection of membership dues of P2,724.40 per month from
PADCOM.
● Later, PADCOM owed the Association P639,961.47, representing membership dues, interests and penalty charges from April 1983
to June 1993. Therefore, the Association filed a complaint for collection of sum of money before the trial court.
● PADCOM contended that it is a non-stock, nonprofit association, and for it to become a special member of the Association, it
should first apply for and be accepted for membership by the latter’s Board of Directors. No automatic membership was apparently
contemplated in the Association’s Bylaws. It further contended that it could not be compelled to become a member without violating
the its right to freedom of association; and since it was not a member, then they are not liable for membership dues, interest and
penalties.
● Trial court dismissed the complaint; CA reversed trial court’s dismissal, hence this petition.

ISSUE: WN petitioner is a member of the Association, and hence liable for membership dues

HELD:
- Yes; petition should be denied.
- It is undisputed that when the land in question was bought by PADCOM’s predecessor-in-interest, TDC, from OCLP, the sale
bound TDC to comply with paragraph (G) of the covenants, conditions and restrictions of the Deed of Sale, which binds the owner
of the land to be a member of the association which will be formed later on. Such lien is inseparable from the property as it is a right
in rem, a burden on the property whoever its owner may be.
- As emphasized earlier, the provision on automatic membership was annotated in the Certificate of Title and made a condition in
the Deed of Transfer in favor of PADCOM. Consequently, it is bound by and must comply with the covenant
- The Court is not persuaded by PADCOM’s contention that the Bylaws of the Association requires application for membership and
acceptance thereof by the Board of Directors. As lot owner, PADCOM is a regular member of the Association. No application for
membership is necessary. If at all, acceptance by the Board of Directors is a ministerial function considering that PADCOM is
deemed to be a regular member upon the acquisition of the lot pursuant to the automatic membership clause annotated in the
Certificate of Title of the property and the Deed of Transfer.
- Neither is the Court convinced by PADCOM’s contention that the automatic membership clause is a violation of its freedom of
association. PADCOM was never forced to join the association. It could have avoided such membership by not buying the land from
TDC. Nobody forced it to buy the land when it bought the building with the annotation of the condition or lien on the Certificate of
Title thereof and accepted the Deed. PADCOM voluntarily agreed to be bound by and respect the condition, and thus to join the
Association.
- By the reasons foregoing, PADCOM is liable for membership dues.

77
CASE DIGEST | Corporation Law (2019) Atty. Villanueva-Castro ib3h

71. TAN v. SYCIP


G.R. NO. 153468
August 17, 2006

FACTS
● Grace Christian High School (GCHS) is a nonstock, nonprofit educational corporation withfifteen (15) regular members, who also
constitute the board of trustees.
● At the time of the annual members meeting held on April 6, 1998, there were only eleven living member-trustees, four had already
died. Out of the 11, 7 attended the meeting through their respective proxies. The meeting was convened over the objection of Atty.
Antonio Pacis, who argued that there was no quorum. Petitioners were voted to replace the four deceased member-trustees.
● Before the SEC, petitioners maintained that deceased member-trustees should not be counted in the computation of the quorum,
because, upon their death, members automatically lost all their rights and interests in the corporation.
● SEC Hearing Officer declared the meeting null and void for lack of quorum, ruling that the determination of quorum depends on
the number specified in the AOI, not simply the number of living members. SEC affirmed in toto; CA dismissed the appeal for
failure to comply with the requirements of Non-Forum Shopping.

ISSUE: Whether dead members should still be counted in the determination of the quorum, for purposes of conducting the annual
members’ meeting

HELD:
- No. One of the most important rights of a qualified shareholder or member is the right to vote—either personally or by proxy—for
the directors or trustees who are to manage the corporate affairs. In the absence of an express charter or statutory provision to the
contrary, the general rule is that every member of a nonstock corporation, and every legal owner of shares in a stock corporation, has
a right to be present and to vote in all corporate meetings.
- Section 52 of the Corporation Code states:
“Section 52. Quorum in Meetings.—Unless otherwise provided for in this Code or in the by-laws, a quorum shall consist of the
stockholders representing a majority of the outstanding capital stock or a majority of the members in the case of non-stock corporations.”
- In stock corporations, the presence of a quorum is ascertained and counted on the basis of the outstanding capital stock. Here, only
stock actually issued and outstanding may be voted.
- In nonstock corporations, the voting rights attach to membership Each member shall be entitled to one vote unless so limited,
broadened, or denied in the articles of incorporation or bylaws. The Court hold that when the principle for determining the quorum
for stock corporations is applied by analogy to nonstock corporations, only those who are actual members with voting rights should
be counted and not necessarily the numerical constant that may originally be specified in the articles of incorporation. Hence, it is
the basis to determine whether it constitutes the quorum.
- Section 25 of the Code specifically provides that a majority of the directors or trustees, as fixed in the articles of incorporation,
shall constitute a quorum for the transaction of corporate business (unless the articles of incorporation or the bylaws provide for a
greater majority). If the intention of the lawmakers was to base the quorum in the meetings of stockholders or members on their
absolute number as fixed in the articles of incorporation, it would have expressly specified so. Otherwise, the only logical conclusion
is that the legislature did not have that intention.
- Membership in and all rights arising from a nonstock corporation are personal and non-transferable, unless the articles of
incorporation or the bylaws of the corporation provide otherwise. Under the By-Laws of GCHS, membership in the corporation shall,
among others, be terminated by the death of the member. Sec. 91 of the Corporation Code provides that termination extinguishes all
the rights of a member of the corporation, unless otherwise provided in the articles of incorporation or the bylaws.
- Hence, dead members who are dropped from the membership roster in the manner and for the cause provided for in the By-Laws
of GCHS are not to be counted in determining the requisite vote in corporate matters or the requisite quorum for the annual members’
meeting. With 11 remaining members, the quorum in the present case should be 6. There being quorum, the annual members’ meeting
was valid.

78
CASE DIGEST | Corporation Law (2019) Atty. Villanueva-Castro ib3h

VIII. CLOSE CORPORATIONS

72. DULAY ENTERPRISES v. CA


G.R. NO. 91889
August 27, 1993

FACTS
● Petitioner Manuel R. Dulay Enterprises, Inc., a domestic corporation with the following as members of its Board of Directors:
Manuel R. Dulay with 19,960 shares and designated as president, treasurer and general manager; Atty. Virgilio E. Dulay with 10
shares and designated as vice-president; Linda E. Dulay with 10 shares; Celia Dulay-Mendoza with 10 shares; and Atty. Plaridel
C. Jose with 10 shares and designated as secretary, owned a property covered by TCT No. 178804 and known as Dulay Apartment.
● Petitioner corporation obtained various loans, and had to borrow money from petitioner Virgilio Dulay for the construction of its
hotel project. As a result of the loan, petitioner Virgilio Dulay occupied one of the unit apartments in the subject property.
● On December 23, 1976, Manuel Dulay, by virtue of Board Resolution No. 18, sold the subject property to private respondents
spouses Veloso in the amount of P300,000, with which Manuel Dulay was given 2 years, in a separate Memorandum, to repurchase
the subject property.
● Unknown to Manuel Dulay, Maria Veloso mortgaged the subject property to private respondent Manuel Torres. Private respondent
Maria Veloso failed t pay Torres, hence it was foreclosed in favor of Torres. Both Veloso and Dulay were unable to redeem the
subject property within the 1 year statutory period for redemtion, hence Torres sought consolidation of ownership of the property
with the Pasay City RD, and later on, a motion for issuance for writ of possession.
● Trial court ordered Torres to implead the petitioner corporation as an indispensable party upon intervention of Virgilio Dulay,
alleging that Manuel Dulay was never authorized by the petitioner corporation to sell or mortgage the property.
● MeTC Pasay ruled in favor of private respondents; RTC ruled in favor of respondents, as well as CA.

ISSUE: WN the conveyance of a corporate property made solely by the corporate president is valid

HELD
- Yes. The petition is not meritorious.
- Sec. 101 of the Corporation Code provides:
“When board meeting is unnecessary or improperly held. Unless the by-laws provide otherwise, any action by the directors
of a close corporation without a meeting shall nevertheless be deemed valid if:
xxx
2. All the stockholders have actual or implied knowledge of the action and make no prompt objection thereto in
writing; or
xxx
4. All the directors have express or implied knowledge of the action in question and none of them makes prompt
objection thereto in writing.”
- Petitioner corporation is classified as a close corporation and consequently a board resolution authorizing the sale or mortgage of
the subject property is not necessary to bind the corporation for the action of its president. A corporate action taken at a board meeting
without proper call or notice in a close corporation ratified by the absent director unless the latter promptly files his written objection
with the secretary of the corporation after having knowledge of the meeting which, in this case, petitioner Virgilio Dulay failed to
do.
- Petitioners’ claim that the sale of the subject property by its president, Manuel Dulay, to private respondents is null and void as the
alleged Board Resolution No. 18 was passed without the knowledge and consent of the other members of the board of directors
cannot be sustained.
- Virgilio was a signatory witness to the execution of the post-dated Deed of Absolute Sale of the subject property in favor of private
respondent Torres indicates that he was aware of the transaction executed.
- Therefore, petitioner corporation is liable for the act of Manuel Dulay and the same of the subject property by Manuel Dulay is
valid and binding.

79
CASE DIGEST | Corporation Law (2019) Atty. Villanueva-Castro ib3h

73. SAN JUAN STRUCTURAL STEEL FABRICATORS v. CA


G.R. NO. 129459
September 29, 1998

FACTS
- On February 14, 1989, petitioner entered intoan agreement with Motorich Sales Corporation for the transfer of a parcel of land by
the latter. Andres Co represented the petitioner as president, while Motorich was represented by its treasurer, Nenita Lee Gruenberg.
- On the day it was agreed that Nenita was supposed to deliver the title, petitioner was ready with the amount corresponding the
balance, but Nenita did not appear. Motorich then refused to execute the Deed of Assignment necessary to transfer the certificate of
title.
- Before the RTC with which petitioner filed a complaint, Motorich Sales Corporation and Nenita Lee Gruenberg interposed as
affirmative defense that the President and Chairman of Motorich did not sign the agreement adverted to in par. 3 of the amended
complaint; that Mrs. Gruenberg’s signature on the agreement is inadequate to bind Motorich. The other signature, that of Mr.
Reynaldo Gruenberg, President and Chairman of Motorich, is required.
- RTC dismissed the complaint; CA affirmed RTC except it orders Nenita Gruenberg to refund P100,000 paid by the petitioner.
- San Juan raised the issue that Nenita was actually the wife of the President of Motorich; that Nenita and her husband owns 98% of
the corporation’s capital stocks; that as such, it is a close corporation and that makes Nenita and the President as principal stockholders
who do not need any authorization from the corporate board; that in this case, the corporate veil may be properly pierced.

ISSUE: WN the Agreement is valid and can bind the corporation; WN the corporation is a close corporation.

HELD:
- No; the petition is devoid of merit.
- A corporation is a juridical person separate and distinct from its stockholders or members. Accordingly, the property of the
corporation is not the property of the corporation and is not the property of its stockholders or members, and may not be sold without
express authorization from the corporation’s board of directors.
- In the case at bar, Respondent Motorich categorically denies that it ever authorized Nenita Gruenberg, its treasurer, to sell the
subject parcel of land. Consequently, petitioner had the burden of proving that Nenita Gruenberg was in fact authorized to represent
and bind Motorich in the transaction. Petitioner failed to discharge this burden. Petitioner cannot assume that she, by virtue of her
position, was authorized to sell the property of the corporation. Selling is obviously foreign to a corporate treasurer’s function
- Anent the allegation that Motorich is a close corporation, and used as a basis for its claim in Dulay Enterprises v. CA, the Corporation
Code defines a close corporation as follows:
SEC. 96. Definition and Applicability of Title.—A close corporation, within the meaning of this Code, is one whose articles of
incorporation provide that: (1) All of the corporation’s issued stock of all classes, exclusive of treasury shares, shall be held of record by
not more than a specified number of persons, not exceeding twenty (20); (2) All of the issued stock of all classes shall be subject to one
or more specified restrictions on transfer permitted by this Title; and (3) The corporation shall not list in any stock exchange or make
any public offering of any of its stock of any class. Notwithstanding the foregoing, a corporation shall be deemed not a close corporation
when at least two-thirds (2/3) of its voting stock or voting rights is owned or controlled by another corporation which is not a close
corporation within the meaning of this Code. x x x.”
- The articles of incorporation of Motorich does not contain any provision stating the items mentioned in Sec. 96, hence, from its
articles, it is clear that Respondent Motorich is not a close corporation. It does not become one either, just because Spouses Reynaldo
and Nenita Gruenberg owned 99.866% of its subscribed capital stock. Mere ownership by a single stockholder or by another
corporation of all or nearly all of the capital stock of the corporation is not, of itself, sufficient ground for disregarding the separate
corporate personalities.
- Petitioners cannot rely on the ruling in Dulay for the factual milieu therein is not on all fours with the present case; as the sale of
real property in Dulay was contracted by the president of a close corporation with the knowledge and acquiescence of its board of
directors.

80
CASE DIGEST | Corporation Law (2019) Atty. Villanueva-Castro ib3h

IX. RELIGIOUS CORPORATIONS

74. IGLESIA EVANGELICA METODISTA v. LAZARO


G.R. NO. 184088
July 6, 2010

FACTS
● 1909, Bishop Nicolas Zamora established the petitioner Iglesia Evangelica Metodista En Las Islas Filipinas, Inc. (IEMELIF) as a
corporation sole with Bishop Zamora acting as its "General Superintendent." In 1948, the IEMEF enacted a by-laws establishing
the Consistory, serving as its Board of Directors.
● Over the years, it remained a corporation sole on paper (with all corporate powers theoretically lodged in the hands of one member,
the General Superintendent), it had always acted like a corporation aggregate. So in 1973 General Conference, the general
membership agreed to change its organizational structure from a corporation sole to a corporation aggregate, which was approved
by the SEC. The corporate papers of IEMEF remained unaltered as a corporation sole however.
● Upon the advise of SEC in 2001, respondent Bishop Nathanael Lazaro, its General Superintendent, instructed all their congregations
to take up the matter with their respective members for resolution. Subsequently, the general membership approved the conversion,
prompting the IEMELIF to le amended articles of incorporation with the SEC. Bishop Lazaro filed an affidavit-certification in
support of the conversion.
● Petitioners Reverend Nestor Pineda, et. al. filed before the Manila RTC motion for nullification of the amended articles of
incorporation claiming that a complete shift from IEMELIF's status as a corporation sole to a corporation aggregate required, not
just an amendment of the IEMELIF's articles of incorporation, but a complete dissolution of the existing corporation sole followed
by a re-incorporation.
● RTC dismissed the action; CA affirmed that of RTC.

ISSUE: WN a corporation sole may be converted into a corporation aggregate by mere amendment of its articles of incorporation

HELD:
- Yes. A corporation sole, cited under Sec. 110 of the Corporation Code, is "one formed by the chief archbishop, bishop, priest,
minister, rabbi or other presiding elder of a religious denomination, sect, or church, for the purpose of administering or managing,
as trustee, the affairs, properties and temporalities of such religious denomination, sect or church." On the other hand, a corporation
aggregate is formed for the same purpose, consisting of two or more persons.
- Given that Sec. 109 CC allows the application to religious corporations of the general provisions governing non-stock corporations,
the power to amend its AOI lies on its members, requiring ⅔ of their votes for the approval. If such approval mechanism is made to
operate in a corporation sole, its one member in whom all the powers of the corporation technically belongs, needs to get the
concurrence of two-thirds of its membership. The one member, here the General Superintendent, is but a trustee, according to Section
110 of the Corporation Code, of its membership.
- Here, the evidence shows that the IEMELIF's General Superintendent, respondent Bishop Lazaro, who embodied the corporation
sole, had obtained, not only the approval of the Consistory that drew up corporate policies, but also that of the required two thirds
vote of its membership. The one member, with the concurrence of two-thirds of the membership of the organization for whom he
acts as trustee, can self-will the amendment. He can, with membership concurrence, increase the technical number of the members
of the corporation from "sole" or one to the greater number authorized by its amended articles.
- There is no point to dissolving the corporation sole of one member to enable the corporation aggregate to emerge from it.
- Besides, it worked out its amendment upon the initiative and advice of the SEC.

81
CASE DIGEST | Corporation Law (2019) Atty. Villanueva-Castro ib3h

X. DISSOLUTION

75. CARLOS GELANO v. CA


G.R. NO. L-39050
February 24, 1981

FACTS
● Private respondent Insular Sawmill, Inc. leased the paraphernal property of petitioner-wife Guillermina Gelano for P1,200 a month.
Petitioner-husband Carlos Gelano received from the corporation cash advances on account of rentals to be paid by the corporation
on the land, obtaining P25,950 from 1947-1950. Out of that amount, Carlos was only able to pay P5,950, refusing to pay the unpaid
balance. Guillermina likewise refused to pay because the said amount was for the personal account of her husband.
● Furthermore, on various occasions from 1948 to 1949 petitioners made credit purchases of lumber materials with a total price of
P1,120.46, and paid P91 in view of a P83 discount. P946.46 unpaid credit purchase was not paid.
● Private respondent through Joseph Tan Yoc Su, executed a joint and several promissory note with Carlos Gelano for China Banking
Corporation in the amount of P8,000. Upon maturity of the promissory note, the bank collected from the respondent corporation
the amount of P9,106.00 including interests, by debiting it from the corporation's current account with the bank. Petitioner Carlos
Gelano was able to pay private respondent the amount of P5,000.00 but the balance of P4,106.00 remained unsettled.
● Private respondent then filed a complaint for collection against the Manila CFI on May 29, 1959.
● The trial court ruled in favor of private respondent in November 20, 1964. The court though was unaware that private respondent
shortened its term of existence up to December 31, 1960 only. CA rendered a decision holding petitioner spouses jointly and
severally liable on the claim.
● It is only after they have received the copy of decision on August 24, 1973 did they come to know that Insular Sawmill dissolved
way back on December 31, 1960. Hence, they filed for motion to dismiss the case and or reconsideration of the decision of the
Court of Appeals on grounds that the case was prosecuted even after dissolution of private respondent as a corporation and that a
defunct corporation cannot maintain any suit for or against it without first complying with the requirements of the winding up of
the affairs of the corporation and the assignment of its property rights within the required period.
● After receipt of such motion, private respondent filed a Petition for Receivership before CFI Manila.

ISSUE: Whether a corporation, whose corporate life had ceased by the expiration of its terms of existence, could still continue
prosecuting and defending suits after its dissolution and beyond the period of three (3) years to wind up its affairs, without having
undertaken any step to transfer its assets to a trustee or assignee.

HELD
- Section 77 of the Corporation Law provides that corporation shall "be continued as a body corporate for three (3) years after the
time when it would have been . . . dissolved, for the purpose of prosecuting and defending suits by or against it . . .," so that, thereafter,
it shall no longer enjoy corporate existence for such purpose; Section 78 of the same authorizes the corporation "at any time during
said three years . . . to convey all of its property to trustees for the benefit of members, stockholders, creditors and other interested,"
evidently for the purpose, among others, of enabling said trustees to prosecute and defend suits by or against the corporation begun
before the expiration of said period.
- In the case at bar, Insular Samill, Inc. still has the right until December 31, 1963 to prosecute in its name the present case. After the
expiration of said period, the corporation ceased to exist for all purposes and it can no longer sue or be sued.
- However, a corporation that has a pending action and which cannot be terminated within the three-year period after its dissolution
is authorized under Section 78 to convey all its property to trustees to enable it to prosecute and defend suits by or against the
corporation beyond the three-year period
- Although private respondent did not appoint any trustee, the counsel who prosecuted and defended the interest of the corporation
in the instant case and who in fact appeared in behalf of the corporation may be considered a trustee of the corporation at least with
respect to the matter in litigation only. The word “trustee” could include the counsel to whom was entrusted in the instant case, the
prosecution of the suit filed by the corporation.
82
CASE DIGEST | Corporation Law (2019) Atty. Villanueva-Castro ib3h
- Therefore, there was a substantial compliance with Section 78 of the Corporation Law and as such, private respondent Insular
Sawmill, Inc. could still continue prosecuting the present case even beyond the period of three (3) years from the time of its
dissolution.

Other cases on Corporate dissolution/Liquidation


NB: This is a collection of doctrines; maker do not claim exclusive ownership over the contents

CLARION PRINTING HOUSE, INC. v. NLRC


G.R. NO. 148372
June 27, 2005

From the provisions of P.D. No. 902-A, as amended, the appointment of a receiver or management committee by the SEC
presupposes a finding that, inter alia, a company possesses sufficient property to cover all its debts but "foresees the impossibility of
meeting them when they respectively fall due" and "there is imminent danger of dissipation, loss, wastage or destruction of assets of
other properties or paralization of business operations."

With the appointment of a management receiver in September 1997, however, all claims and proceedings against CLARION,
including labor claims, were deemed suspended during the existence of the receivership. The labor arbiter, the NLRC, as well as the
CA should not have proceeded to resolve respondent’s complaint for illegal dismissal and should instead have directed respondent
to lodge her claim before the then duly-appointed receiver of CLARION.

LINGKOD MANGGAGAWA SA RUBBERWORLD v. RUBBERWORLD


G.R. NO. 153882
January 29, 2007

The law is clear: upon the creation of a management committee or the appointment of a rehabilitation receiver, all claims for actions
"shall be suspended accordingly." No exception in favor of labor claims is mentioned in the law. Since the law makes no distinction
or exemptions, neither should this Court. Ubi lex non distinguit nec nos distinguere debemos. Allowing labor cases to proceed clearly
defeats the purpose of the automatic stay and severely encumbers the management committee's time and resources. The said
committee would need to defend against these suits, to the detriment of its primary and urgent duty to work towards rehabilitating
the corporation and making it viable again. To rule otherwise would open the floodgates to other similarly situated claimants and
forestall if not defeat the rescue efforts. Besides, even if the NLRC awards the claims of private respondents, its ruling could not be
enforced as long as the petitioner is under the management committee.

PD 902-A is clear that “ all action for claims against corporation, partnerships or association under management or receivership
pending before any court, tribunal, board or body shall be suspended accordingly.’’ The law did not make any exception in favor of
labor claims. The justification for such to enable the management committee to exercise its powers free from interference that might
hinder or prevent the “rescue’’ of the debtor company. To allow the labor case to proceed would open the defeat the rescue effort of
the management committee. Even if an award is given, the ruling could not enforce as long as petitioner is under management
committee.

GARCIA v. PAL
G.R. NO. 164856
August 29, 2007

Upon appointment by the SEC of a rehabilitation receiver, all actions for claims against the corporation pending before any court,
tribunal or board shall ipso jure be suspended. The purpose of the automatic stay of all pending actions for claims is to enable the
rehabilitation receiver to effectively exercise its/his powers free from any judicial or extra-judicial interference that might unduly
hinder or prevent the rescue of the corporation. More importantly, the suspension of all actions for claims against the corporation
embraces all phases of the suit, be it before the trial court or any tribunal or before this Court. No other action may be taken, including

83
CASE DIGEST | Corporation Law (2019) Atty. Villanueva-Castro ib3h
the rendition of judgment during the state of suspension. It must be stressed that what are automatically stayed or suspended are the
proceedings of a suit and not just the payment of claims during the execution stage after the case had become final and executory.
Furthermore, the actions that are suspended cover all claims against the corporation whether for damages founded on a breach of
contract of carriage, labor cases, collection suits or any other claims of a pecuniary nature. No exception in favor of labor claims is
mentioned in the law.

SOBRAJUANITE v. ASB DEVELOPMENT CORP.


G.R. NO. 165675
September 30, 2005

Section 6(c) of PD No. 902-A empowers the SEC:


c) To appoint one or more receivers of the property, real and personal, which is the subject of the action pending before the
Commission . . . whenever necessary in order to preserve the rights of the parties-litigants and/or protect the interest of the
investing public and creditors: . . . Provided, finally, That upon appointment of a management committee, rehabilitation
receiver, board or body, pursuant to this Decree, all actions for claims against corporations, partnerships or associations
under management or receivership pending before any court, tribunal, board or body shall be suspended accordingly.

The purpose for the suspension of the proceedings is to prevent a creditor from obtaining an advantage or preference over another
and to protect and preserve the rights of party litigants as well as the interest of the investing public or creditors. Such suspension is
intended to give enough breathing space for the management committee or rehabilitation receiver to make the business viable again,
without having to divert attention and resources to litigations in various fora. The suspension would enable the management
committee or rehabilitation receiver to effectively exercise its/his powers free from any judicial or extrajudicial interference that
might unduly hinder or prevent the "rescue" of the debtor company. To allow such other action to continue would only add to the
burden of the management committee or rehabilitation receiver, whose time, effort and resources would be wasted in defending
claims against the corporation instead of being directed toward its restructuring and rehabilitation.

As between creditors, the key phrase is "equality is equity." When a corporation threatened by bankruptcy is taken over by a receiver,
all the creditors should stand on equal footing. Not anyone of them should be given any preference by paying one or some of them
ahead of the others. This is precisely the reason for the suspension of all pending claims against the corporation under receivership.
Instead of creditors vexing the courts with suits against the distressed firm, they are directed to file their claims with the receiver who
is a duly appointed officer of the SEC.

PANLILIO v. RTC
G.R. NO. 173846
February 2, 2011

Corporate rehabilitation connotes the restoration of the debtor to a position of successful operation and solvency, if it is shown that
its continued operation is economically feasible and its creditors can recover more, by way of the present value of payments projected
in the rehabilitation plan, if the corporation continues as a going concern than if it is immediately liquidated. It contemplates a
continuance of corporate life and activities in an effort to restore and reinstate the corporation to its former position of successful
operation and solvency, the purpose being to enable the company to gain a new lease on life and allow its creditors to be paid their
claims out of its earnings.

A principal feature of corporate rehabilitation is the suspension of claims against the distressed corporation. Section 6 (c) of
Presidential Decree No. 902-A, as amended, provides for suspension of claims against corporations undergoing rehabilitation. In this
vein, the term "claim" has been construed to refer to debts or demands of a pecuniary nature, or the assertion to have money paid.

The prosecution of the officers of the corporation has no bearing on the pending rehabilitation of the corporation, especially since
they are charged in their individual capacities. Such being the case, the purpose of the law for the issuance of the stay order is not
compromised, since the appointed rehabilitation receiver can still fully discharge his functions as mandated by law. It bears to stress

84
CASE DIGEST | Corporation Law (2019) Atty. Villanueva-Castro ib3h
that the rehabilitation receiver is not charged to defend the officers of the corporation. If there is anything that the rehabilitation
receiver might be remotely interested in is whether the court also rules that petitioners are civilly liable.

VILLAMOR v. UMALE
GR NO. 172843
September 24, 2014

While devices and schemes of the board of directors, business associates, or officers amounting to fraud under Rule 1, Section 1(a)(1)
of the Interim Rules are causes of a derivative suit, it is not always the case that derivative suits are limited to such causes or that
they are necessarily derivative suits. Rule 8, Section 1 of the Interim Rules of Procedure for Intra Corporate Controversies (Interim
Rules) provides the 5 requisites for filing derivative suits.

SECTION 1. Derivative action. – A stockholder or member may bring an action in the name of a corporation or association,
as the case may be, provided that:
(1)He was a stockholder or member at the time the acts or transactions subject of the action occurred and at the
time the action was filed;
(2)He exerted all reasonable efforts, and alleges the same with particularity in the complaint, to exhaust all remedies
available under the articles of incorporation, by-laws, laws or rules governing the corporation or
partnership to obtain the relief he desires;
(3)No appraisal rights are available for the act or acts complained of; and
(4)The suit is not a nuisance or harassment suit.

The fifth requisite for filing derivative suits, while not included in the enumeration, is implied in the first paragraph of Rule 8, Section
1 of the Interim Rules: The action brought by the stockholder or member must be "in the name of [the] corporation or association."
This requirement has already been settled in jurisprudence. Thus, it is important that the corporation be made a party to the case. The
corporation must be joined as party because it is its cause of action that is being litigated and because judgment must be a res judicata
against it.”

Stockholder/s’ suits based on fraudulent or wrongful acts of directors, associates, or officers may also be individual suits or class
suits. Individual suits are filed when the cause of action belongs to the individual stockholder personally, and not to the stockholders
as a group or to the corporation.

85
CASE DIGEST | Corporation Law (2019) Atty. Villanueva-Castro ib3h

XI. FOREIGN CORPORATIONS

77. FACILITIES MANAGEMENT CORPORATION v. DE LA OSA


G.R. NO. L-38649
March 26, 1979

FACTS
● Leonardo dela Osa filed before the Court of Industrial Relations seeking for reinstatement with full backwages, as well as recovery
of his overtime compensation, swing shift, and graveyard shift differentials. He was employed by respondents in various positions.
He averred that on December, 1965 to August, 1966, inclusive, he rendered overtime services daily, and that this entire period was
divided into swing and graveyard shifts to which he was assigned, but he was not paid both overtime and night shift premiums
despite his repeated demands from respondents.
● Respondents (petitioners herein) filed a motion to dismiss on the ground that CIR has no jurisdiction over the instant case, alleging
that they are domiciled in Wake Island, beyond the jurisdiction of the Philippine Government, that respondent J. V. Catuira, though
an employee of respondent corporation presently stationed in Manila, is without power and authority of legal representation; and
that the employment contract between petitioner and respondent corporation carries the approval of the Department of Labor of the
Philippines.
● CIR ruled in favor of Dela Osa, the contract of employment between the parties litigant was shown to have been originally executed
and subsequently renewed in Manila, as asserted by petitioner and not denied by respondents. Hence, any dispute arising therefrom
should necessarily be determined in the place or venue where it was contracted.

ISSUE: Whether petitioner has been ‘doing business in the Philippines’ so that the service of summons upon its agent in the
Philippines vested the court with jurisdiction.

HELD:
- Yes. CA noted that petitioner had to appoint Jaime V. Catuira, 1322 A. Mabini, Ermita, Manila ‘as agent for FMC with authority
to execute Employment Contracts and receive, in behalf of that corporation, legal services from and be bound by processes of the
Philippine Courts of Justice, for as long as he remains an employee of FMC, and that Catuira represented it in this country for the
purpose of making arrangement for approval by the DOLE of the employment of Filipinos recruited by the company as its own
employees for assignment abroad. Catuira then was a liaison officer representing FMC in the Philippines.
- RA 5455 illustrated ‘doing business’, among them as follows:
1) Soliciting orders, purchases or service contracts x x x;
2) Appointing a representative or distributor who is domiciled in the Philippines, unless said representative or distributor has an
independent status; x x x
3) xxx
4) Opening offices, whether called ‘liaison, offices, agencies or branches, unless proved otherwise.
xxx
10) Any other act or acts that imply a continuity of commercial dealings or arrangements, and contemplate to that extent the performance
of acts or works, or the exercise of some of the functions normally incident to, or in the progressive prosecution of, commercial gain or
of the purpose and objective of the business organization.
- The object of Sections 68 and 69 of the Corporation Law was to prevent the corporation from acquiring a domicile for the purpose
of business without taking the steps necessary to render it amenable to the suit in the local courts. (Aenta Casualty v. Pacific Star
Line) While Aetna Casualty is not engaged in the business of insurance in the Philippines, it is not barred from filing the instant case
because it is merely collecting a claim assigned to it by the consignee.
- Therefore, in the viewpoint of serving summons under the Rules of Court, petitioner is a foreign corporation doing business in the
Philippines.
- Indeed, if a foreign corporation, not engaged in business in the Philippines, is not barred from seeking redress from courts in the
Philippines, a fortiori that same corporation cannot claim exemption from being sued in Philippine courts for acts done against a
person or persons in the Philippines.

86
CASE DIGEST | Corporation Law (2019) Atty. Villanueva-Castro ib3h

77. HOME INSURANCE COMPANY v. EASTERN SHIPPING LINES


G.R. NO. L-34382
July 20, 1983

FACTS
● S. Kajita & Co., on behalf of Atlas Consolidated Mining & Development Corporation, shipped on board the SS 'Eastern Jupiter'
from Osaka, Japan, 2,361 coils of 'Black Hot Rolled Copper Wire Rods.' The vessel was owned and operated by Eastern Shipping
Lines. The shipment was insured with petitioner. The shipment arrived with net loss/shortage, some in bad order. Plaintiff then
paid consignee and demanded payment against the carrier after subrogating consignee, which the carrier refused to pay.
● The plaintiff, as they aver, is a foreign insurance company duly authorized to do business in the Philippines through its agent, Mr.
VICTOR H. BELLO, of legal age and with office address at Oledan Building, Ayala Avenue, Makati, Rizal.
● The lower court dismissed the complaints on ground that the plaintiff failed to prove its capacity to sue, holding that:
"A suing foreign corporation, the plaintiff, has to plead affirmatively and prove either that the transaction upon which it
bases its complaint is an isolated one, or that it is licensed to transact business in this country, failing which, it will be
deemed that it has no valid cause of action x x x plaintiff is doing business in the Philippines. Consequently, it must have a
license under Section 68 of the Corporation Law before it can be allowed to sue.”

ISSUE: WN petitioner, a foreign insurance company, has capacity to sue

HELD:
- Yes. When the complaints in these two cases were filed, the petitioner had already secured the necessary license to conduct its
insurance business in the Philippines. It could already file suits.
- Petitioner was, therefore, telling the truth when it averred in its complaints that it was a foreign insurance company duly authorized
to do business in the Philippines through its agent Mr. Victor H. Bello.
- However, when the insurance contracts which formed the basis of these cases were executed, the petitioner had not yet secured the
necessary licenses and authority.
- The applicable provisions of the old Corporation Law, are as follows:
“Sec. 68. No foreign corporation or corporations formed, organized, or existing under any laws other than those of the
Philippine Islands shall be permitted to transact business in the Philippine Islands until after it shall have obtained a license
for that purpose from the chief of the Mercantile Register of the Bureau of Commerce and Industry, (Now Securities and
Exchange Commission. x x x
xxx
Sec. 69. No foreign corporation or corporation formed, organized, or existing under any laws other than those of the
Philippine Islands shall be permitted to transact business in the Philippine Islands or maintain by itself or assignee any suit
for the recovery of any debt, claim, or demand whatever, unless it shall have the license prescribed in the section
immediately preceding.”
- In this regard, the objective of the law was to subject the foreign corporation to the jurisdiction of our courts. The Corporation Law
must be given a reasonable, not an unduly harsh, interpretation which does not hamper the development of trade relations and which
fosters friendly commercial intercourse among countries.
- The Corporation Law provides that there is a penalty for transacting business without registration: imprisonment for not less than
6 months nor more than two years or payment of a fine; and the foreign corporation or its assignee may not maintain any suit for the
recovery of any debt, claim, or demand whatever. The Corporation Law however is silent on whether the contract executed by a
foreign corporation with no capacity to sue is null and void ab initio.
- Our jurisprudence leans towards the view that such contracts are voidable only, not void. There is therefore, no question that the
contracts are enforceable. The requirement of registration affects only the remedy. It is, therefore, not necessary to declare the contract
null and void even as against the erring foreign corporation. The penal sanction for the violation and the denial of access to our courts
and administrative bodies are sucient from the viewpoint of legislative policy.
- Nonetheless, the lack of capacity at the time of the execution of the contracts was cured by the subsequent registration is also
strengthened by the procedural aspects of these cases.

87
CASE DIGEST | Corporation Law (2019) Atty. Villanueva-Castro ib3h

78. MENTHOLATUM CO., INC. v. MANGALIMAN


G.R. NO. 47701
June 27, 1941

FACTS
● On October 1, 1935, the Mentholatum Co., Inc., and the Philippine-American Drug, Co., Inc. instituted an action in the Court of
First Instance of Manila, civil case No. 48855, against Anacleto Mangaliman, Florencio Mangaliman and the Director of the Bureau
of Commerce for infringement of trade mark and unfair competition.
● It seeks to restrain Mangaliman from selling their product “Mentholiman”, and also render accounting of their sales and profits and
pay damages.
● The complaint states that Mentholatum, Inc. is a Kansas corporation manufacturing “Mentholatum”, and that the Philippine-
American Drug Co., Inc., is its exclusive distributing agent in the Philippines authorized by it to look after and protect its interests.
● CFI Manila rendered judgment in favor of the complainants; reversed by CA holding that Mentholatum, Inc. conducts business
transactions in the Philippines, and by Sec. 69 of the Corporation Law, it may not maintain the present suit.
● Petitioners maintain before this Court that Mentholatum Co., Inc., has not sold personally any of its products in the Philippines;
that the Philippine-American Drug Co., Inc., like fifteen or twenty other local entities, was merely an importer of the products of
the Mentholatum Co., Inc., and that the sales of the Philippines-American Drug Co., Inc., were its own and not for the account of
the Mentholatum Co., Inc.

ISSUE: Whether petitioners could prosecute the instant action without having secured the license required in Sec. 69 of the
Corporation Law.

HELD:
- Sec. 69 of the Corporation Law reads:
“No foreign corporation or corporation formed, organized, or existing under any laws other than those of the Philippine Islands shall be
permitted to transact business in the Philippine Islands or maintain by itself or assignee any suit for the recovery of any debt, claim or
demand whatever, unless it shall have the license prescribed x x x”
- While it is undisputed that plaintiff (petitioner) is a foreign corporation and is not licensed to do business in the Philippines, the
question remains as to whether the said corporation is or is not transacting business in the Philippines.
- No general rule or governing principle can be laid down as to what constitutes "doing" or "engaging in" or "transacting" business.
Indeed, each case must be judged in the light of its peculiar environmental circumstances. The true test, however, seems to be whether
the foreign corporation is continuing the body or substance of the business or enterprise for which it was organized or whether it has
substantially retired from it and turned it over to another.
- The term implies a continuity of commercial dealings and arrangements, and contemplates, to that extent, the performance of acts
or works or the exercise of some of the functions normally incident to, and in progressive prosecution of, the purpose and object of
its organization.
- CA concluded that Mentholatum, through Philippine-American Drug Co., Inc., has been doing business in the Philippines since
1929, at least. It follows that whatever transactions the Philippine-American Drug Co., Inc., had executed in view of the law, the
Mentholatum Co., Inc., being a foreign corporation doing business in the Philippines without the license required by section 68 of
the Corporation Law, it may not prosecute this action for violation of trade mark and unfair competition. Neither may the Philippine-
American Drug Co., Inc., maintain the action as an agent, being his representative character and derivative authority.
- The right of the petitioner conditioned upon compliance with the requirement of section 69 of the Corporation Law to protect its
rights, is hereby reserved.

88
CASE DIGEST | Corporation Law (2019) Atty. Villanueva-Castro ib3h

79. ERIKS PTE. LTD. v. CA


G.R. NO. 118843
February 6, 1997

FACTS
● Petitioner Eriks Pte. Ltd. is a non-resident foreign corporation engaged in the manufacture and sale of elements used in sealing
pumps, valves and pipes for industrial purposes, valves and control equipment used for industrial fluid control and PVC pipes and
fittings for industrial uses. It alleges that they are organized and existing under the laws of Singapore and not licensed to do business
in the Philippines and is suing on an isolated transaction.
● Private respondent Delfin Enriquez, Jr., do ordered and received from petitioner various elements used in sealing pumps, valves,
pipes and control equipment, PVC pipes and fittings. Eriks sold its items to Enriquez for 16 times from Jan 17 1989 to May 31,
1989.
● Transfer of goods were made also with a 90-day credit term, but Enriquez failed/refused to settle his account when demands are
made.
● Petitioner corporation then filed with Makati RTC, for the recovery of SGD 41, 939.63, or its equivalent in Philippine currency.
The trial court dismissed the action on the ground that petitioner is a foreign corporation doing business in the Philippines without
a license. CA affirmed the same action, holding that the transactions herein are not “isolated or casual transaction.”

ISSUE: Whether petitioner may maintain an action in the Philippine courts considering that it has no license to do business in the
country.

HELD:
- Sec. 133 of the Corporation Code provides:
“Sec. 133. Doing business without a license . — No foreign corporation transacting business in the Philippines without a license, or its
successors or assigns, shall be permitted to maintain or intervene in any action, suit or proceeding in any court or administrative agency
of the Philippines; but such corporation may be sued or proceeded against before Philippine courts or administrative tribunals on any
valid cause of action recognized under Philippine laws."
- The aforementioned provision prohibits, not merely absence of the prescribed license, but it also bars a foreign corporation "doing
business" in the Philippines without such license access to our courts. A foreign corporation without such license is not ipso facto
incapacitated from bringing an action. A license is necessary only if it is "transacting or doing business" in the country.
- RA 7042 produced a statutory definition on ‘doing business’: “shall include soliciting orders, service contracts, opening offices, whether
called 'liaison' offices or branches; appointing representatives or distributors domiciled in the Philippines or who in any calendar year stay in the
country for a period or periods totaling one hundred eight(y) (180) days or more; participating in the management, supervision or control of any
domestic business, firm, entity or corporation in the Philippines; and any other act or acts that imply a continuity of commercial dealings or
arrangements, and contemplate to that extent the performance of acts or works, or the exercise of some of the functions normally incident to, and
in progressive prosecution of, commercial gain or of the purpose and object of the business organization x x x”
- The purpose of the law is to subject the foreign corporation doing business in the Philippines to the jurisdiction of our courts. It is
not to prevent the foreign corporation from performing single or isolated acts, but to bar it from acquiring a domicile for the purpose
of business without first taking the steps necessary to render it amenable to suits in the local courts.
- Factual records show clear and unmistakable intention on the part of petitioner to continue the body of its business in the Philippines
is more than apparent. The sale by petitioner of the items covered by the receipts, which are part and parcel of its main product line,
was actually carried out in the progressive prosecution of commercial gain and the pursuit of the purpose and object of its business,
pure and simple. The grant and extension of credit terms shows also an intention to continue transacting with private respondent,
since credit is extended only to customers with whom there is an intention to maintain long-term relationship.
- Hence, these series of transactions in question could not have been isolated or casual transactions. What is determinative of "doing
business" is not really the number or the quantity of the transactions, but more importantly, the intention of an entity to continue the
body of its business in the country.
- Accordingly, petitioner must be held to be incapacitated to maintain the action a quo against private respondent.
- Nonetheless, its subsequent acquisition of license will cure the lack of capacity at the time of the execution of the contract (no res
judicata)

89
CASE DIGEST | Corporation Law (2019) Atty. Villanueva-Castro ib3h

80. MERRILL LYNCH FUTURES v. CA


G.R. NO. 97816
July 24, 1992

FACTS
● On November 23, 1987, Merrill Lynch Futures, Inc. (hereafter, simply ML FUTURES) led a complaint with the Regional Trial
Court at Quezon City against the Spouses Pedro M. Lara and Elisa G. Lara for the recovery of a debt and interest thereon, damages,
and attorney's fees.
● ML Futures is a non-resident foreign corporation, not doing business in the Philippines, duly organized and existing under and by
virtue of the laws of Delaware, USA.
● On September 28, 1983 it entered into a Futures Customer Agreement with the defendant spouses (Account No. 138-12161), in
virtue of which it agreed to act as the latter's broker for the purchase and sale of futures contracts in the U.S.; that the Lara spouses
knew and were duly advised that Merrill Lynch Philippines, INc. was not a broker in futures contracts and dud not have a license
from SEC to operate as a commodity trading advisor
● Over the years, and indebtedness of the ensuing balance of USD 84,836.27 was incurred by spouses Lara to ML Futures, the former
refusing to pay alleging that the transactions were null and void because Merrill Lynch Philippines, Inc. had no license to poperate
as a commodity and/or financial futures broker.
● ML Futures then filed a complaint before the Quezon City RTC, with which respondents moved to dismiss the complaint on ground
that ML Futures had no legal capacity to sue since it is not the real party in interest.
● Trial Court promulgated an order sustaining the motion to dismiss. CA affirmed the trial court decision, invoing Sec. 133 of the
Corporation Code, holding that plaintiff is doing business in this country, due to series of business acts, contracts, and transactions
from 1983 to 1987.

ISSUE: Whether ML Futures is prohibited from suing in Philippine courts

HELD:
- No. Facts on record adequately establish that ML FUTURES, operating in the United States, had indeed done business with the
Lara Spouses in the Philippines over several years, had done so at all times through Merrill Lynch Philippines, Inc. (MLPI), a
corporation organized in this country, and had executed all these transactions without ML FUTURES being licensed to so transact
business here, and without MLPI being authorized to operate as a commodity futures trading advisor.
- However, a party is estopped to challenge the personality of a corporation after having acknowledged the same by entering into a
contract with it. There would seem to be no question that the Laras received benefits generated by their business relations with ML
FUTURES. Those business relations, according to the Laras themselves, spanned a period of seven (7) years; and they evidently
found those relations to be of such profitability as warranted their maintaining them for that not insignificant period of time.
- Given these facts, and assuming that the Lara Spouses were aware from the outset that ML FUTURES had no license to do business
in this country and MLPI, no authority to act as broker for it, it would appear quite inequitable for the Laras to evade payment of an
otherwise legitimate indebtedness due and owing to ML FUTURES upon the plea that it should not have done
business in this country in the first place.

90
CASE DIGEST | Corporation Law (2019) Atty. Villanueva-Castro ib3h

81. AGILENT TECHNOLOGIES SINGAPORE (PTE) LTD., v. INTEGRATED SILICON


G.R. NO. 154618
April 14, 2004

FACTS
● Petitioner Agilent Technologies Singapore (Pte.), Ltd. ("Agilent") is a foreign corporation, which, by its own admission, is not
licensed to do business in the Philippines. Respondent on the other hand is a private domestic corporation, 100% foreign owned,
engaged in the business of manufacturing and assembling electronics components.
● A Value Added Assembly Services Agreement (VAASA) was entered between Hewlett-Packard Singapore and Integrated Silicon
wherein nder the terms of the VAASA, Integrated Silicon was to locally manufacture and assemble fiber optics for export to HP-
Singapore. HP-Singapore, for its part, was to consign raw materials to Integrated Silicon; transport machinery to the plant of
Integrated Silicon; and pay Integrated Silicon the purchase price of the finished products.
● Integrated Silicon filed for complaint for Specific Performance and Damages against Agilent and its officers, alleging that Agilent
breached the parties’ oral agreement to extend the VAASA.
● Summons for Agilent were served to Atty. Ramon Quisumbing, who returned the same on the claim that he was not registered
agent of Agilent. He later entered for appearance to assail the court’s jurisdiction over the person of Agilent.
● Another complaint filed by Agilent for Specific Performance and Replevin, where respondents filed a motion to dismiss by which
the trial court denied. CA set aside the assailed order, and ordered the dismissal of the first case.

ISSUE: WN Agilent has legal capacity to file suit

HELD:
- Yes. The principles regarding the right of a foreign corporation to bring suit in Philippine courts may condensed in four statements:
1) if a foreign corporation does business in the Philippines without a license, it cannot sue before the Philippine courts;
2) if a foreign corporation is not doing business in the Philippines, it needs no license to sue before Philippine courts on an
isolated transaction or on a cause of action entirely independent of any business transaction;
3) if a foreign corporation does business in the Philippines without a license, a Philippine citizen or entity which has contracted
with said corporation may be estopped from challenging the foreign corporation’s corporate personality in a suit brought
before Philippine courts; and
4) if a foreign corporation does business in the Philippines with the required license, it can sue before Philippine courts on any
transaction.
- As provided for in Mentholatum v. Mangaliman, the court discusses two general tests to determine whether a foreign corporation
can be considered as “doing business” in the Philippines:
a) substance test: whether the foreign corporation is continuing the body of the business or enterprise for which it was
organized or whether it has substantially retired from it and turned it over to another; and
b) continuity test: continuity of commercial dealings and arrangements, and contemplates, to that extent, the performance of
acts or works or the exercise of some of the functions normally incident to, and in the progressive prosecution of, the purpose
and object of its organization
- An analysis of Section 1 of the Implementing Rules and Regulations of the FIA would demonstrate that the acts enumerated in the
VAASA do not constitute “doing business” in the Philippines. Acts include:
1) Mere investment as a shareholder by a foreign entity in domestic corporations duly registered to do business, and/or the
exercise of rights as such investor;
2) Having a nominee director or officer to represent its interest in such corporation;
3) Appointing a representative or distributor domiciled in the Philippines which transacts business in the representative’s or
distributor’s own name and account;
4) The publication of a general advertisement through any print or broadcast media;
5) Maintaining a stock of goods in the Philippines solely for the purpose of having the same processed by another entity in
the Philippines;
6) Consignment by a foreign entity of equipment with a local company to be used in the processing of products for export;

91
CASE DIGEST | Corporation Law (2019) Atty. Villanueva-Castro ib3h
7) Collecting information in the Philippines; and
8) Performing services auxiliary to an existing isolated contract of sale which are not on a continuing basis, such as
installing in the Philippines machinery it has manufactured or exported to the Philippines, servicing the same, training
domestic workers to operate it, and similar incidental services. (emphasis supplied ib)
- By the clear terms of VAASA, Agilent’s activities in the Philippines were confined to (1) maintaining a stock of goods in the
Philippines solely for the purpose of having the same processed by Integrated Silicon; and (2) consignment of equipment with
Integrated Silicon to be used in the processing of products for export.
- Therefore, Agilent cannot be deemed to be “doing business” in the Philippines. Respondents’ contention that Agilent lacks the legal
capacity to file suit is therefore devoid of merit. As a foreign corporation not doing business in the Philippines, it needed no license
before it can sue before our courts.

92
CASE DIGEST | Corporation Law (2019) Atty. Villanueva-Castro ib3h

82. AIR CANADA v. COMMISSIONER OF INTERNAL REVENUE


G.R. NO. 169507
January 11, 2016

FACTS
● Air Canada is a "foreign corporation organized and existing under the laws of Canada[.]" 5 On April 24, 2000, it was granted an
authority to operate as an offline carrier by the Civil Aeronautics Board, subject to certain conditions, which authority would expire
on April 24, 2005. Air Canada does not have flights originating from or coming to the Philippines and does not operate any airplane
in the Philippines.
● Air Canada engaged the services of Aerotel Ltd. Corp. as its general sales agent in the Philippines.
● For the period ranging from the third quarter of 2000 to the second quarter of 2002, Air Canada, through Aerotel, led quarterly and
annual income tax returns and paid the income tax on Gross Philippine Billings in the total amount of P5,185,676.77
● On November 2002, Air Canada led a written claim for refund of alleged erroneously paid income taxes amounting to
P5,185,676.77 before the Bureau of Internal Revenue, Revenue District Office in Makati, finding basis from the revised definition
of Gross Philippine Billings under Sec. 28(A)(3)(a) of the 1997 NIRC, which is as follows:
Sec. 28. x x x
(A) Tax on Resident Foreign Corporations
xxx
(3) International Carrier - An international carrier doing business in the Philippines shall pay two and one-half percent (2 ½%) on its Gross Philippine
Billings as defined hereunder:
xxx
(a) x x x gross revenue derived from carriage of persons, excess baggage, cargo and mail originating from the Philippines in a continuous and
uninterrupted ight, irrespective of the place of sale or issue and the place of payment of the ticket or passage document. x x x
● Air Canada also filed a Petition for Review before the CTA, which denied the Petition holding that Air Canada was engaged in
business in the Philippines through a local agent and should be taxed as a resident foreign corporation at the regular rate of 32%.
CTA en banc affimed the findings.

ISSUE: Whether Air Canada, as an offline international carrier selling passage documents through a general sales agent in the
Philippines, is a resident corporation within the meaning of Sec. 28 (A)(1) of the 1997 NIRC.

HELD:
- Yes. An offline international air carrier selling passage tickets in the Philippines, through a general sales agent, is a resident foreign
corporation doing business in the Philippines.
- Air Canada is not liable to tax on Gross Philippine Billings under Sec. 28(A)(1) of the 1997 NIRC, but is a resident foreign
corporation for income tax purposes. RA 7042 provides guidance with the definition of “doing business” with regard to foreign
corporations where it states that:
“the phrase "doing business" shall include soliciting orders, service contracts, opening offices, whether called "liaison" offices or branches;
appointing representatives or distributors domiciled in the Philippines or who in any calendar year stay in the country for a period or periods
totalling one hundred eighty (180) days or more; participating in the management, supervision or control of any domestic business, firm, entity
or corporation in the Philippines; and any other act or acts that imply a continuity of commercial dealings or arrangements, and contemplate to
that extent the performance of acts or works, or the exercise of some of the functions normally incident to, and in progressive prosecution of,
commercial gain or of the purpose and object of the business organization x x x”
- An offline carrier is "any foreign air carrier not certificated by the Civil Aeronautics Board, but who maintains office or who has
designated or appointed agents or employees in the Philippines, who sells or offers for sale any air transportation in behalf of said
foreign air carrier and/or others, or negotiate for, or holds itself out by solicitation, advertisement, or otherwise sells, provides,
furnishes, contracts, or arranges for such transportation." Petitioner is undoubtedly “doing business” or “engaged in trade or business”
in the Philippines. Nothing on records show that Aerotel solicited orders alone and for its own account and without interference from,
let alone direction of, petitioner.
- Petitioner is, therefore, a resident foreign corporation that is taxable on its income derived from sources within the Philippines.
Petitioner's income from sale of airline tickets, through Aerotel, is income realized from the pursuit of its business activities in the
Philippines.

93
CASE DIGEST | Corporation Law (2019) Atty. Villanueva-Castro ib3h

83. STEELCASE, INC. v. DESIGN INTERNATIONAL SELECTIONS, INC.


G.R. NO. 171995
April 18, 2012

FACTS
● Petitioner Steelcase, Inc. (Steelcase) is a foreign corporation existing under the laws of Michigan, United States of America
(U.S.A.), and engaged in the manufacture of office furniture with dealers worldwide. Respondent, on the other hand, Design
International Selections, Inc. (DISI) is a corporation existing under Philippine Laws and engaged in the furniture business,
including the distribution of furniture.
● Sometime in 1986 or 1987, Steelcase and DISI orally entered into a dealership agreement whereby Steelcase granted DISI the
right to market, sell, distribute, install, and service its products to end-user customers within the Philippines. The business
relationship was terminated sometime in 1999 after the agreement was breached with neither party admitting any fault.
● Steelcase filed a complaint for sum of money against DISI alleging that the latter had an unpaid account of USD 600,000. On the
other hand, DISI alleged that the complainant failed to state a cause of action and to contain the required allegations on Steelcase's
capacity to sue in the Philippines despite the fact that it (Steelcase) was doing business in the Philippines without the required
license to do so.
● RTC dismissed the complaint, concluding that Steelcase was “doing business” in the Philippines upon unwitting revelation that
Steelcase participated in the operations of DISI, and since it did not have the license to do business in the country, it was barred
from seeking redress from our courts until it obtained license to do so. CA affirmed RTC order.

ISSUE: Whether Steelcase is doing business in the Philippines without a license; Whether DISI is estopped from challenging
Steelcase’s legal capacity to sue.

HELD:
- No. Steelcase is an unlicensed foreign corporation not doing business in the Philippines. The Implementing Rules and Regulations
of FIA 1991 provides, among others the following acts shall not be deemed “doing business” in the Philippines:
xxx
3. Appointing a representative or distributor domiciled in the Philippines which transacts business in the representative’s or distributor’s
own name and account; x x x
- The appointment of a distributor in the Philippines is not sufficient to constitute "doing business" unless it is under the full control
of the foreign corporation. On the other hand, if the distributor is an independent entity which buys and distributes products, other
than those of the foreign corporation, for its own name and its own account, the latter cannot be considered to be doing business in
the Philippines.
- In the case at bar, DISI was founded in 1979 and independently owned and managed by spouses Bantug. In addition to Steelcase
products, DISI also distributed products of other companies including carpet tiles, relocatable walls and theater settings.
- Therefore, the only reasonable conclusion that can be reached is that DISI was an independent contractor, distributing various
products of Steelcase and of other companies, acting in its own name and for its own account.

- Anent the second issue, If indeed Steelcase had been doing business in the Philippines without a license, DISI would nonetheless
be estopped from challenging the former's legal capacity to sue.
- It cannot be denied that DISI entered into a dealership agreement with Steelcase and profited from it for 12 years from 1987 until
1999. DISI admits that it complied with its obligations under the dealership agreement by exerting more effort and making substantial
investments in the promotion of Steelcase products. Unquestionably therefore, entering into a dealership agreement with Steelcase
charged DISI with the knowledge that Steelcase was not licensed to engage in business activities in the Philippines.
- The Court has time and again upheld the principle that a foreign corporation doing business in the Philippines without a license
may still sue before the Philippine courts a Filipino or a Philippine entity that had derived some benet from their contractual
arrangement because the latter is considered to be estopped from challenging the personality of a corporation after it had
acknowledged the said corporation by entering into a contract with it.

94
CASE DIGEST | Corporation Law (2019) Atty. Villanueva-Castro ib3h

84. PIONEER INTERNATIONAL v. GUADIZ


G.R. NO. 156848
October 11, 2007

FACTS
● Antonio D. Todaro (Todaro) led a complaint for sum of money and damages with preliminary attachment against PIL, Pioneer
Concrete Philippines, Inc. (PCPI), Pioneer Philippines Holdings, Inc. (PPHI), John G. McDonald (McDonald), and Philip J. Klepzig
(Klepzig). As alleged, petitioner PIL is a corporation duly organized under Australian laws, while PCPI and PPHI are corporations
duly organized under Philippine laws. McDonald is CEO of PIL’s HK office, while Klepzig is the President and Managing Director
of PPHI and PCPI.
● Todaro met with several PIL representatives asking him if he could join in establishing a pre-mixed concrete plant and overseeing
its operations in the Philippines. The agreement, as shown in the Annex, is that after three months, Todaro shall be employed, on
a permanent basis, as its managing director or CEO in the Philippines. Later on, Klepzig to Todaro provides that their agreement
will cease and they will withdraw the offer in view of Todaro’s refusal to consider their terms of offer of permanent employment.
● As an answer to the complaint, PIL asserted that the trial court has no jurisdiction over PIL because PIL is a foreign corporation
not doing business in the Philippines. PIL also questioned the service of summons on it, served on De Leon, Klepzig’s Executive
Assistant.
● Trial court ruled in favor of Todaro, asserting its jurisdiction over PIL. CA affirmed the trial court in toto.

ISSUE: Whether or not the PIL is a foreign corporation not doing business in the Philippines and the court acquire jurisdiction upon
them

HELD
- PIL was doing business in the Philippines when it negotiated Todaro’s employment with PPHI. Sec. 3(d) of RA 7042 states,:
“[t]he phrase "doing business " shall include soliciting orders, service contracts, opening offices, whether called "liaison" offices or
branches; x x x and any other act or acts that imply a continuity of commercial dealings or arrangements and contemplate to that extent
the performance of acts or works, or the exercise of some of the functions normally incident to, and in progressive prosecution of
commercial gain or of the purpose and object of the business organization: x x x
- PIL's alleged acts in actively negotiating to employ Todaro to run its pre-mixed concrete operations in the Philippines, which acts
are hypothetically admitted in PIL's motion to dismiss, are not mere acts of a passive investor in a domestic corporation. Such are
managerial and operational acts in directing and establishing commercial operations in the Philippines.
- The behavior of the various Pioneer corporations shoots down their defense that the corporations have separate and distinct
personalities, managements, and operations. The various Pioneer corporations were all working in concert to negotiate an
employment contract between Todaro and PPHI, a domestic corporation.
- Lastly, for the purpose of service of summons, the phrase "doing business in the Philippines" in the former version of Section 12,
Rule 14 now reads "has transacted business in the Philippines." The scope is thus broader in that it is enough for the application of
the Rule that the foreign private juridical entity "has transacted business in the Philippines."
- When summons is served on a foreign juridical entity, there are three prescribed ways: (1) service on its resident agent designated
in accordance with law for that purpose, (2) service on the government official designated by law to receive summons if the
corporation does not have a resident agent, and (3) service on any of the corporation's officers or agents within the Philippines.

95
CASE DIGEST | Corporation Law (2019) Atty. Villanueva-Castro ib3h

85. COLUMBIA PICTURES, INC., v. CA


G.R. NO. 110318
August 28, 1996

FACTS
● Complainants, through counsel, lodged a formal complaint with the National Bureau of Investigation for violation of PD No. 49,
as amended, and sought its assistance in their anti-film piracy drive, where NBI agents made discreet surveillance on various video
establishments including Sunshine Home Video, Inc.
● A search warrant was also applied by the NBI, against Sunshine. The search warrant was served at about 1:45 PM on December
14, 1987 where various video tapes of duly copyrighted motion pictures/films owned by private complainants, machines,
equipment, television sets, paraphernalia, materials, accessories, etc.
● A Motion to Lift the Order of Search Warrant was filed but was denied for lack of merit. CA lifted the search warrant.

ISSUE: Whether or not Columbia Pictures, Inc. has legal standing in our courts, they being corporation not licensed to do business
in the Philippines.

HELD
- The Corporation Code provides:
“Sec. 133. Doing business without a license . — No foreign corporation transacting business in the Philippines without a license, or its
successors or assigns, shall be permitted to maintain or intervene in any action, suit or proceeding in any court or administrative agency
of the Philippines; but such corporation may be sued or proceeded against before Philippine courts or administrative tribunals on any
valid cause of action recognized under Philippine laws.”
- The obtainment of a license prescribed by Section 125 of the Corporation Code is not a condition precedent to the maintenance of
any kind of action in Philippine courts by a foreign corporation. It is not the absence of the prescribed license but "doing business"
in the Philippines without such license which debars the foreign corporation from access to our courts. In other words, although a
foreign corporation is without license to transact business in the Philippines, it does not follow that it has no capacity to bring an
action. Such license is not necessary if it is not engaged in business in the Philippines.
- Since there are statutory provisions in many jurisdictions are determinative of what constitutes doing business or transacting
business, where there are no such definition or qualification is laid down regarding acts or transactions falling within its purview, it
is held that all the combined acts of a foreign corporation in the State must be considered, and every circumstance is material which
indicates a purpose on the part of the corporation to engage in some part of its regular business in the State.
- Based on Sec. 133 of the Corporation Code, petitioners are not barred from maintaining the present action. There is no showing
that, under our statutory or case law, petitioners are doing, transacting, engaging in or carrying on business in the Philippines as
would require obtention of a license before they can seek redress from our courts.
- The fact that petitioners are admittedly copyright owners or owners of exclusive distribution rights in the Philippines of motion
pictures or films does not convert such ownership into an indicium of doing business which would require them to obtain a license
before they can use upon a cause of action in local courts.
- There is no point in seeing how exercising one's legal and property rights and taking steps for the vigilant protection of said rights,
particularly the appointment of an attorney-in-fact, can be deemed by and of themselves to be doing business here.

96
CASE DIGEST | Corporation Law (2019) Atty. Villanueva-Castro ib3h

86. GLOBAL BUSINESS HOLDINGS, INC. v. SURECOMP SOFTWARE, B.V.


G.R. NO. 173463
October 13, 2010

FACTS
- Respondent Surecomp Software, B.V. (Surecomp), a foreign corporation duly organized and existing under the laws of the
Netherlands, entered into a software license agreement with Asian Bank Corporation (ABC), a domestic corporation, for the use of
its IMEX Software System (System) in the bank's computer system for a period of twenty (20) years.
- A year later, ABC merged with petitioner GLobal Business Holdings, Inc. with Global as the surviving corporation. Global found
the System unworkable for its operations, and informed Surecomp of its decision to discontinue with the agreement and to stop
further payments thereon.
- Surecomp then filed a complaint for breach of contract before the Makati RTC. Surecomp alleged that it is a foreign corporation
not doing business in the Philippines and is suing on an isolated transaction. it installed the System in ABC's computers for a
consideration of US$298,000.00 as license fee. ABC also undertook to pay Surecomp professional services, which included on-site
support and development of interfaces, and annual maintenance fees for five (5) subsequent anniversaries, and committed to purchase
one (1) or two (2) Remote Access solutions at discounted prices. In a separate transaction, ABC requested Surecomp to purchase on
its behalf a software called MF Cobol Runtime with a promise to reimburse its cost.
- Global filed a motion to dismiss based on the fact that Surecomp had no capacity to sue because it was doing business in the
Philippines without a license.
- RTC held Global in estoppel from denying Surecomp’s capacity to sue; CA affirmed the RTC decision.

ISSUE: Whether Global is estopped from questioning Surecomp’s capacity to sue

HELD
- Yes. The determination of a corporation's capacity is a factual question that requires the elicitation of a preponderant set of facts.
As a rule, unlicensed foreign non resident corporations doing business in the Philippines cannot file suits in the Philippines.
- A corporation has a legal status only within the state or territory in which it was organized. For this reason, a corporation organized
in another country has no personality to file suits in the Philippines. In order to subject a foreign corporation doing business in the
country to the jurisdiction of our courts, it must acquire a license from the Securities and Exchange Commission and appoint an agent
for service of process. Without such license, it cannot institute a suit in the Philippines.
- exception to this rule is the doctrine of estoppel. Global is estopped from challenging Surecomp’s capacity to sue. A foreign
corporation doing business in the Philippines without license may sue in Philippine courts a Filipino citizen or a Philippine entity
that had contracted with and benefited from it. A party is estopped from challenging the personality of a corporation after having
acknowledged the same by entering into a contract with it.
- Due to Global's merger with ABC and because it is the surviving corporation, it is as if it was the one which entered into contract
with Surecomp. In the merger of two existing corporations, one of the corporations survives and continues the business, while the
other is dissolved, and all its rights, properties, and liabilities are acquired by the surviving corporation

97
CASE DIGEST | Corporation Law (2019) Atty. Villanueva-Castro ib3h
NB:
Republic Act No. 7042
SEC. 3. Definitions. — As used in this Act:
xxx xxx xxx
● (d) the phrase "doing business” shall include soliciting orders, service contracts, opening offices, whether called 'liaison'
offices or branches; appointing representatives or distributors domiciled in the Philippines or who in any calendar year
stay in the country for a period or periods totalling one hundred eight(y) (180) days or more; participating in the
management, supervision or control of any domestic business, firm, entity or corporation in the Philippines; and any
other act or acts that imply a continuity of commercial dealings or arrangements, and contemplate to that extent the
performance of acts or works, or the exercise of some of the functions normally incident to, and in progressive
prosecution of, commercial gain or of the purpose and object of the business organization: Provided, however , That the
phrase "doing business" shall not be deemed to include mere investment as a shareholder by a foreign entity in domestic
corporations duly registered to do business, and/or the exercise of rights as such investors; nor having a nominee director
or officer to represent its interests in such corporation; nor appointing a representative or distributor domiciled in the
Philippines which transacts business in its own name and for its own account.

Rule I, Sec. 1, Implementing Rules and Regulations of RA 7042

f. "Doing business" shall include soliciting orders, service contracts, opening offices, whether liaison offices or branches;
appointing representatives or distributors, operating under full control of the foreign corporation, domiciled in the Philippines or
who in any calendar year stay in the country for a period totalling one hundred eighty [180] days or more; participating in the
management, supervision or control of any domestic business, firm, entity or corporation in the Philippines; and any other act or
acts that imply a continuity of commercial dealings or arrangements, and contemplate to that extent the performance of acts or
works, or the exercise of some of the functions normally incident to and in progressive prosecution of commercial gain or of the
purpose and object of the business organization.

The following acts shall not be deemed "doing business" in the Philippines:

1. Mere investment as a shareholder by a foreign entity in domestic corporations duly registered to do business, and/or the exercise
of rights as such investor;
2. Having a nominee director or officer to represent its interest in such corporation;
3. Appointing a representative or distributor domiciled in the Philippines which transacts business in the representative's or
distributor's own name and account;
4. The publication of a general advertisement through any print or broadcast media;
5. Maintaining a stock of goods in the Philippines solely for the purpose of having the same processed by another entity in the
Philippines;
6. Consignment by a foreign entity of equipment with a local company to be used in the processing of products for export;
7. Collecting information in the Philippines; and
8. Performing services auxiliary to an existing isolated contract of sale which are not on a continuing basis, such as installing in
the Philippines machinery it has manufactured or exported to the Philippines, servicing the same, training domestic workers to
operate it, and similar incidental services.

98
CASE DIGEST | Corporation Law (2019) Atty. Villanueva-Castro ib3h

87. AGUIRRE II v. FQB+7, INC.


G.R. NO. 170770
January 9, 2013

FACTS:
- On October 5, 2004, petitioner Vitaliano Aguirre II filed, in his individual capacity and on behalf of FQB+7, Inc., a Complaint
against respondents Nathaniel, Priscila Bocobo, and Antonio De Villa (Antonio). - The Complaint alleged that FQB+7 was
established in 1985 with the following directors and subscribers:
Directors Subscribers
1. Francisco Q. Bocobo 1.Francisco Q. Bocobo
2. Fidel N. Aguirre 2. Fidel N. Aguirre
3. Alfredo Torres 3. Alfredo Torres
4. Victoriano Santos 4. Victoriano Santos
5. Victorino Santos 5. Victorino Santos
6. Vitaliano Aguirre II
7. Alberto Galang
8. Rolando B. Bechayda
- The Complaint further alleged that, sometime in April 2004, Vitaliano discovered a GIS) of FQB+7, filed by Francisco Bocobo's
heirs, Nathaniel and Priscila, as FQB+7's president and secretary/treasurer, respectively.
Directors Subscribers
1. Nathaniel D. Bocobo 1.Nathaniel D. Bocobo
2. Priscila D. Bocobo 2. Priscila D. Bocobo
3. Fidel N. Aguirre 3. Fidel N. Aguirre
4. Victoriano Santos 4. Victorino Santos
5. Victorino Santos 5. Victorino Santos
6. Consolacion Santos
- This prompted Vitaliano to write to the "real" Board of Directors (the directors reflected in the Articles of Incorporation),
questioning the validity and truthfulness of the alleged stockholders meeting held on September 3, 2002.
- He asked the "real" Board to rectify what he perceived as erroneous entries in the GIS, and to allow him to inspect the corporate
books and records, which ignored Vitaliano's request.
- On September 27, 2004, Nathaniel, in the exercise of his power as FQB+7's president, appointed Antonio as the corporation's
attorney-in-fact. Pursuant thereto, Antonio attempted to take over the farm, but was allegedly prevented by Fidel and his men.
- The Complaint asked for an injunction against them and for the nullification of all their previous actions, and also sought damages
for the plaintiffs and a declaration of Vitaliano's right to inspect the corporate records.
- On October 27, 2004, the trial court issued the writ of preliminary injunction after Vitaliano filed an injunction bond.
- The respondents filed a motion for an extension of 10 days to file the pleadings which the trial court denied motion for being a
prohibited pleading under the Interim Rules of Procedure Governing Intra-corporate Controversies under Republic Act (R.A.) No.
8799.
- The respondents filed a Petition for Certiorari and Prohibition on the ground that Branch 24 of the Manila RTC has no jurisdiction,
theorizing that Vitaliano's real goal in filing the Complaint was to maintain custody of the corporate farm in Quezon Province, the
jurisdiction belonging to the Department of Agrarian Reform (DAR), not to the Manila RTC. Respondents also invoked that FQB+7’s
Certificate of Registration has been revoked, hence dissolved.
- CA dismissed the petition of Aguirre, as well as holding that the RTC committed grave abuse of discretion. CA held that Section
122 of the Corporation Code allows a dissolved corporation to continue as a body corporate for the limited purpose of liquidating
the corporate assets and distributing them to its creditors, stockholders, and others in interest. CA determined that Vitaliano's
Complaint, being geared towards the continuation of FQB+7, Inc.'s business, should be dismissed

ISSUE: WN the RTC has jurisdiction over an intra-corporate dispute involving a dissolved corporation.

99
CASE DIGEST | Corporation Law (2019) Atty. Villanueva-Castro ib3h

HELD:
- Yes. For one, the complaint is only to determine and vindicate an alleged stockholder’s right to return of his stockholdings and to
participate in the election of directors, and corporation’s right to remove usurpers and strangers from its affairs, which is not mooted
by the dissolution.
- A corporation’s board of directors is not rendered functus officio by its dissolution. Since Sec. 122 allows a corporation to continue
its existence for a limited purpose, then necessarily there must be a board that will continue acting for and behalf of the dissolved
corporation for that purpose.
- Sec. 122 authorizes the dissolved corporation’s board of directors to conduct its liquidation within three years, while jurisprudence
even recognized the board’s authority as trustee for person in interest beyond the three-year period.
- Furthermore, a party’s stockholdings in a corporation, whether existing or dissolved, is a property right which he may vindicate
against another party who has deprived him thereof, which right is not dissolved upon dissolution.

100
CASE DIGEST | Corporation Law (2019) Atty. Villanueva-Castro ib3h

SECURITIES AND REGULATIONS CODE


NB: All assigned cases not digested; only those left presumably to be asked in the final exam

SEC v. PRICE RICHARDSON CORP.


GR NO. 197032
July 26, 2017

Section 26. Fraudulent Transactions.—It shall be unlawful for any person, directly or indirectly, in connection
with the purchase or sale of any securities to: x x x
26.3. Engage in any act, transaction, practice or course of business which operates or would operate as a
fraud or deceit upon any person.

Section 28. Registration of Brokers, Dealers, Salesmen and Associated Persons.— 28.1. No person shall
engage in the business of buying or selling securities in the Philippines as a broker or dealer, or act as a
salesman, or an associated person of any broker or dealer unless registered as such with the Commission.

ABACUS SECURITIES CORPORATION v. AMPIL


GR NO. 160016
February 27, 2006

FACTS:
● Petitioner is engaged in business as a broker and dealer of securities of listed companies at the Philippine Stock Exchange Center.
● In April 1997, respondent opened a cash or regular account with [petitioner] for the purpose of buying and selling securities. Since
April 10, 1997, [respondent] actively traded his account, and as a result of such trading activities, he accumulated an outstanding
obligation in favor of [petitioner] in the principal sum of P6,617,036.22 as of April 30, 1997
● Despite lapse of the period within which to pay his account as well as sufficient time given by [petitioner] for [respondent] to
comply with his proposal to settle his account, the latter failed to do so. Petitioner then sold respondent’s securities to set off against
his unsettled obligations.
● Petitioner then sent a demand letter to settle his obligation; respondent requested for 69 days to raise funds to pay, which was
granted by the petitioner, but was respondent still failed or refused to pay.
● Respondent claims that he was induced to trade in a stock security with petitioner because the latter allowed offset settlements
wherein he is not obliged to pay the purchase price, but the deficiency after sale. He further claims that all his trades with [petitioner]
were not paid in full in cash at anytime after purchase or within the T+4 (4 days subsequent to trading), and none was cancelled,
neither request for extension of time was applied for.
● Had petitioner followed the provision under par. 8 of Exh. ‘A-1’ which stipulated the liquidation within the T+3 [3 days subsequent
to trading], his net deficit would only be P1,601,369.59.
● This is all while affirming that in accordance to Rule 25-1, par C of RSA Rules in which it mandates that if you do not pay for the
first order, “you cannot subsequently make any further order without depositing the cash price in full.” So, if this rule was applied,
he was limited only to the first transaction, in view of the acts of petitioner who failed to comply with the T+3, who not require
him to put up a deposit before it executed its subsequent orders.
● RTC Makati held that petitioner violated Sections 23 and 25 of the Revised Securities petitioner violated Sections 23 and 25 of the
Revised Securities Act and Rule 25-1 when it failed to: require the respondent to pay for his stock purchases within three (T+3) or
four days (T+4) from trading; and 2) request from the appropriate authority an extension of time for the payment of respondent’s
cash purchases. However, the RTC concluded that upon finding that respondent is equally at fault by incurring excessive credits
and seeing how his investments turned out before invoking RSA, that they are in pari delicto and therefore without recourse against
each other.
● CA upheld.

101
CASE DIGEST | Corporation Law (2019) Atty. Villanueva-Castro ib3h
ISSUE: Whether or not the Court of Appeal’s ruling that petitioner and respondent are in pari delicto which allegedly bars any
recovery

HELD:
- Yes. The provisions governing the above transactions are Secs. 23 and 25 of RSA and Rule 25-1 of the RSA Rules, to wit:
SEC. 23. Margin Requirements.
xxx xxx xxx
(b) It shall be unlawful for any member of an exchange or any broker or dealer, directly or indirectly, to extend or maintain credit or
arrange for the extension or maintenance of credit to or for any customer--
(1) On any security other than an exempted security, xxx
(2) Without collateral or on any collateral other than securities, xxx
xxx
“SEC. 25. Enforcement of margin requirements and restrictions on borrowings.—To prevent indirect violations of the margin
requirements under Section 23 hereof, the broker or dealer shall require the customer in nonmargin transactions to pay the price of the
security purchased for his account within such period as the Commission may prescribe, which shall in no case exceed three trading days;
otherwise, the broker shall sell the security purchased starting on the next trading day but not beyond ten trading days following the last
day for the customer to pay such purchase price, unless such sale cannot be effected within said period for justifiable reasons. The sale
shall be without prejudice to the right of the broker or dealer to recover any deficiency from the customer. x x x.” (emphasis supplied)

RSA RULE 25-1


Purchases and Sales in Cash Account

a) Purchases by a customer in a cash account shall be paid in full within three (3) business days after the trade date.
b) If full payment is not received within the required time period, the broker or dealer shall cancel or otherwise liquidate the transaction
xxx
c) If a transaction is cancelled or otherwise liquidated as a result of non-payment by the customer, prior to any subsequent purchase
during the next ninety (90) days, the customer shall be required to deposit sufficient funds in the account to cover each purchase
transaction prior to execution
xxx xxx
f) Written application for an extension of the period of time required for payment under paragraph (a) be made by the broker or dealer to
Philippine Stock Exchange, in case a member of Exchange, or in SEC for non-members.

- Section 23(b) above—the alleged violation of petitioner which provides the basis for respondent’s defense—makes it unlawful for
a broker to extend or maintain credit on any securities other than in conformity with the rules and regulations issued by Securities
and Exchange Commission (SEC).
- On the other hand, RSA Rule 25-1 prescribes in detail regulations governing cash accounts.

- A margin account x x x is an account in which the broker lends the customer cash with which to purchase securities. Unlike a cash
account, a margin account allows an investor to buy securities with money that he does not have, by borrowing the money from the
broker. The RSA limits margin borrowing to a maximum of 50% of the amount invested. (SEC Comment dated September 27, 2005)
- Restrictions on margin percentages are imposed “in order to achieve the objectives of the government with due regard for the
promotion of the economy and prevention of the use of excessive credit.

- The law places the burden of compliance with margin requirements primarily upon the brokers and dealers. Sections 23 and 25 and
Rule 25-1, otherwise known as the “mandatory close-out rule,clearly vest upon petitioner the obligation, not just the right, to cancel
or otherwise liquidate a customer’s order, if payment is not received within three days from the date of purchase. For transactions
subsequent to an unpaid order, the broker should require its customer to deposit funds into the account sufficient to cover each
purchase transaction prior to its execution.
- It will be noted that trading on credit (or “margin trading”) allows investors to buy more securities than their cash position would
their cash position would normally allow. Investors pay only a portion of the purchase price of the securities; their broker advances
for them the balance of the purchase price and keeps the securities as collateral for the advance or loan. The nature of the stock
brokerage business enables brokers, not the clients, to verify, at any time, the status of the client’s account Brokers, therefore, are in
the superior position to prevent the unlawful extension of credit. Concomitant there to, is their primary obligation to enforce the
margin requirements.
102
CASE DIGEST | Corporation Law (2019) Atty. Villanueva-Castro ib3h

- In the case at bar, these margin requirements are applicable only to transactions entered into by the present parties subsequent to
the initial trades of April 10 and 11, 1997. Thus, can still collect from respondent to the extent of the difference between the latter’s
outstanding obligation as of April 11, 1997 less the proceeds from the mandatory sell out of the shares pursuant to the RSA Rules.
- Petitioners’ fault arose only when it failed liquidate the transactions on the fourth day following the stock purchases, or on April
14 and 15, 1997; and 2) complete its liquidation no later than ten days thereafter, applying the proceeds thereof as payment for
respondent’s outstanding obligation
- It should be clear that Congress imposed the margin requirements to protect the general economy, not to give the customer a free
ride at the expense of the broker Not to require respondent to pay for his April 10 and 11 trades would put a premium on his
circumvention of the laws and would enable him to enrich himself unjustly at the expense of petitioner.
- By failing to ensure the payment of first purchase, and allowing him to make subsequent purchases, petitioner effectively converted
respondent’s cash account into a credit account.
- In the same vein, the Court find respondent equally guilty in entering into the transactions in violation of the RSA and RSA Rules.
He is an experienced and knowledgeable trader well-versed in the securities market and made his own investment decisions. He
knowingly speculated on the market, taking advantage of the ‘no-cash-out’ arrangement extended to him by petitioner.

- In the final analysis, both parties acted in violation of the law and did not come to court with clean hands with regard to transactions
subsequent to the initial trades made on April 10 and 11, 1997. Thus, the peculiar facts of the present case bar the applicat ion of the
pari delicto rule. Such is applied only to transactions entered into after the initial trades made on April 10 and 11, 1997.

SEC v. INTERPORT RESOURCES CORP., ET. AL.


G.R. NO. 135808
October 6, 2008

Section 53 of the Securities Regulations Code clearly provides that criminal complaints for violations of rules and regulations
enforced or administered by the SEC shall be referred to the Department of Justice (DOJ) for preliminary investigation, while the
SEC nevertheless retains limited investigatory powers. Additionally, the SEC may still impose the appropriate administrative
sanctions under Section 54 of the aforementioned law.

Sec. 30. Insider's duty to disclose when trading. - (a) It shall be unlawful for an insider to sell or buy a security of the issuer, if he
knows a fact of special significance with respect to the issuer or the security that is not generally available, unless (1) the insider
proves that the fact is generally available or (2) if the other party to the transaction (or his agent) is identified, (a) the insider proves
that the other party knows it, or (b) that other party in fact knows it from the insider or otherwise.

The provision explains in simple terms that the insider's misuse of nonpublic and undisclosed information is the gravamen of illegal
conduct. The intent of the law is the protection of investors against fraud, committed when an insider, using secret information, takes
advantage of an uninformed investor. Insiders are obligated to disclose material information to the other party or abstain from trading
the shares of his corporation. This duty to disclose or abstain is based on two factors: first, the existence of a relationship giving
access, directly or indirectly, to information intended to be available only for a corporate purpose and not for the personal benefit of
anyone; and second, the inherent unfairness involved when a party takes advantage of such information knowing it is unavailable to
those with whom he is dealing.

BAVIERA v. PAGLINAWAN
G.R. NO. 170602
February 8, 2007

FACTS
● Petitioner was the former head of HR Service Delivery and Industrial Relations of Standard Chartered Bank-Philippines.
● BSP imposed some conditions in the conduct of SCB’s business in this jurisdiction, but apparently did not comply with these.
Instead, as early as 1996, it acted as a stock broker, soliciting from local residents foreign securities called “GLOBAL THIRD
103
CASE DIGEST | Corporation Law (2019) Atty. Villanueva-Castro ib3h
PARTY MUTUAL FUNDS” (GTPMF), denominated in US dollars. These securities were not registered with the Securities
and Exchange Commission (SEC). These were then remitted outwardly to SCB-Hong Kong and SCB-Singapore.
● On July 18, 1997, Investment Capital Association of the Philippines (ICAP) filed with the SEC a complaint alleging that SCB
violated the Revised Securities Act, on selling of securities without prior registration with the SEC. SEC issued then a CDO
against SCB, holding that its services violated Sections 4(a) and 19 of RSA. The BSP also directed SCB not to include
investments in global mutual funds issued abroad in its trust investments portfolio without prior registration with the SEC.
Notwithstanding this directive, SCB continued to offer GTPMF securities in this country.
● This prompted petitioner to enter into an Investment Trust Agreement with SCB wherein he purchased US$8,000.00 worth of
securities upon the bank’s promise of 40% return on his investment and a guarantee that his money is safe. After six (6) months,
however, petitioner learned that the value of his investment went down to US$7,000.00. He tried to withdraw his invest ment
but was persuaded by Antonette de los Reyes of SCB to hold on to it for another six (6) months in view of the possibility that
the market would pickup. However, it further went down to only USD 3,000.00.
● Petitioner learned from Marivel Gonzales, head of the SCB Legal and Compliance Department, that the latter had been
prohibited by the BSP to sell GPTMF securities. Petitioner then filed with the BSP a letter-complaint demanding compensation
for his lost investment.
● Petitioner also filed with the DOJ a complaint for syndicated estafa, and thereafter for violation of the Securities Regulation
Code. DOJ dismissed the complaint for estafa; dismissed also for violation of SRC the latter for the reason that it should have
been filed with the SEC. CA dismissed the petition.

ISSUE: Whether DOJ committed grave abuse of discretion in dismissing petitioner’s complaint for violation of SRC

HELD:
- No. Sec. 53.1 of the Securities Regulation Code provides:
53.1. The Commission may, in its discretion, make such investigation as it deems necessary to determine whether any person has violated
or is about to violate any provision of this Code, any rule, regulation or order thereunder, or any rule of an Exchange, registered securities
association, clearing agency, other self regulatory organization, and may require or permit any person to file with it a statement in
writing, under oath or otherwise, as the Commission shall determine, as to all facts and circumstances concerning the matter to be
investigated. The Commission may publish information concerning any such violations and to investigate any fact, condition, practice or
matter which it may deem necessary or proper to aid in the enforcement of the provisions of this Code, in the prescribing of rules and
regulations thereunder, x x x Provided, further, That all criminal complaints for violations of this Code and the implementing rules and
regulations enforced or administered by the Commission shall be referred to the Department of Justice for preliminary investigation and
prosecution before the proper court. x x x
- A criminal complaint for violation of any law or rule administered by the SEC, in this case, SRC, must be first filed with the latter,
being a specialized dispute. If the Commission finds that there is probable cause, then it should refer the case to the DOJ.
- The Securities Regulation Code is a special law. Its enforcement is particularly vested in the SEC. Hence, all complaints for any
violation of the Code and its implementing rules and regulations should be filed with the SEC. Where the complaint is criminal in
nature, the SEC shall indorse the complaint to the DOJ for preliminary investigation and prosecution as provided in Section 53.1
earlier quoted.
- Since petitioner failed to comply with the foregoing procedural requirement, the DOJ did not gravely abuse its discretion in
dismissing his complaint.

CEMCO HOLDINGS v. NATIONAL LIFE INSURANCE


GR NO. 171815
August 7, 2007

Tender offer is a publicly announced intention by a person acting alone or in concert with other persons to acquire equity securities
of a public company. A public company is defined as a corporation which is listed on an exchange, or a corporation with assets
exceeding P50,000,000.00 and with 200 or more stockholders, at least 200 of them holding not less than 100 shares of such company.
Stated differently, a tender offer is an offer by the acquiring person to stockholders of a public company for them to tender their
shares therein on the terms specified in the offer. Tender offer is in place to protect minority shareholders against any scheme that
dilutes the share value of their investments. It gives the minority shareholders the chance to exit the company under reasonable terms,
giving them the opportunity to sell their shares at the same price as those of the majority shareholders.
104
CASE DIGEST | Corporation Law (2019) Atty. Villanueva-Castro ib3h

The legislative intent of Section 19 of the Code is to regulate activities relating to acquisition of control of the listed company and
for the purpose of protecting the minority stockholders of a listed corporation. Whatever may be the method by which control of a
public company is obtained, either through the direct purchase of its stocks or through an indirect means, mandatory tender offer
applies.

PHILIPPINE VETERANS BANK v. CALLANGAN


G.R. NO. 191995
August 3, 2011

FACTS
● Justina F. Callangan, the Director of the Corporation Finance Department of the Securities and Exchange Commission (SEC),
sent the Bank a letter, informing it that it qualifies as a “public company” under Section 17.2 of the Securities Regulation Code
(SRC) in relation with Rule 3(1)(m) of the Amended Implementing Rules and Regulations of the SRC. The Bank is thus required
to comply with the reportorial requirements set forth in Section 17.1 of the SRC.
● The bank responded that it should not be considered a “public company” because it is a private company whose shares of stock
are available only to a limited class or sector, i.e. World War II veterans.
● Callangan rejected the Bank’s explanation and assessed penalties for failure to comply with SRC reportorial requirements. SEC
en banc dismissed the Bank’s appeal. CA dismissed petition, the Court denied Bank’s petition, hence this MR.

ISSUE: Whether the Bank is a “public company” burdened with the reportorial requirements ordered by the SEC

HELD:
- Yes. Sec. 17.2 of SRC requires reportorial requirements to:
An issuer with assets of at least Fifty million pesos (P50,000,000.00) or such other amount as the Commission shall
prescribe, and having two hundred (200) or more holders each holding at least one hundred (100) shares of a class of its
equity securities x x x
- Furthermore, Rule 3(1)(m) defines a “public company” defines as “any corporation with a class of equity securities listed on an
Exchange OR assets in excess of Fifty Million Pesos (P50,000,000.00) and having two hundred (200) or more holders, at least two
hundred (200) of which are holding at least one hundred (100) shares of a class of its equity securities.”
- “Public company,” as contemplated by the SRC, is not limited to a company whose shares of stock are publicly listed; even
companies like the Bank, whose shares are offered only to a specific group of people, are considered a public company, provided
they meet the requirements enumerated above.
- The Bank does not dispute, that the Bank has assets exceeding P50,000,000.00 and has 395,998 shareholders. It is thus considered
a public company that must comply with the reportorial requirements set forth in Section 17.1 of the SRC.

SEC v. PROSPERITY.COM, INC.


G.R. NO. 164197
January 25, 2012

The Securities Regulation Code treats investment contracts as "securities" that have to be registered with the SEC before they can be
distributed and sold. An investment contract is a contract, transaction, or scheme where a person invests his money in a common
enterprise and is led to expect profits primarily from the efforts of others.

The United States Supreme Court, grappling with the problem, has on several occasions discussed the nature of investment contracts.
That court’s rulings, while not binding in the Philippines, enjoy some degree of persuasiveness insofar as they are logical and
consistent with the country’s best interests. The United States Supreme Court held in Securities and Exchange Commission v. W.J.
Howey Co. that, for an investment contract to exist, the following elements, referred to as the HOWEY TEST must concur: (1) a

105
CASE DIGEST | Corporation Law (2019) Atty. Villanueva-Castro ib3h
contract, transaction, or scheme; (2) an investment of money; (3) investment is made in a common enterprise; (4) expectation of
profits; and (5) profits arising primarily from the efforts of others. Thus, to sustain the SEC position in this case, PCI’s scheme or
contract with its buyers must have all these elements.

INTESTATE ESTATE OF TY v. CA
GR NO. 112872
April 19, 2001

It does not necessarily follow that when both parties of a dispute are stockholders of a corporation, the dispute is automatically
considered intra-corporate in nature and jurisdiction consequently falls with the SEC. PD 902-A did not confer upon the SEC absolute
jurisdiction. The better policy in determining which body has jurisdiction over this case would be to consider, not merely the status
of the parties involved, but likewise the nature of the question that is the subject of the controversy. When the nature of the controversy
involves matters that are purely civil in character, it is beyond the ambit of the limited jurisdiction of the SEC.

The determination whether a contract is simulated or not is an issue that could be resolved by applying pertinent provisions of the
Civil Code, particularly those relative to obligations and contracts. Disputes concerning the application of the Civil Code are properly
cognizable by courts of general jurisdiction. No special skill is necessary that would require the technical expertise of the SEC.

YUJUICO v. QUIAMBAO
GR NO. 168639
January 29, 2007

Upon the enactment of R.A. No. 8799, otherwise known as "The Securities Regulation Code" which took effect on August 8, 2000,
the jurisdiction of the SEC over intra-corporate controversies and other cases enumerated in Section 5 of P.D. No. 902-A has been
transferred to the courts of general jurisdiction, or the appropriate RTC. Section 5.2 of R.A. No. 8799 provides:
5.2. The Commission’s jurisdiction over all cases enumerated in Section 5 of Presidential Decree No. 902-A
is hereby transferred to the Courts of general jurisdiction or the appropriate Regional Trial Court,

Clearly, the RTC has the power to hear and decide the intra-corporate controversy of the parties herein. Concomitant to said power
is the authority to issue orders necessary or incidental to the carrying out of the powers expressly granted to it. Thus, the RTC may,
in appropriate cases, order the holding of a special meeting of stockholders or members of a corporation involving an intra-corporate
dispute under its supervision.

GSIS v. CA
GR NO. 183905
April 16, 2009

The conferment of original and exclusive jurisdiction on the regular courts over such controversies in the election of corporate
directors must be seen as intended to confine to one body the adjudication of all related claims and controversy arising from the
election of such directors. For that reason, the aforequoted Section 2, Rule 6 of the Interim Rules broadly defines the term "election
contest" as encompassing all plausible incidents arising from the election of corporate directors, including: (1) any controversy or
dispute involving title or claim to any elective office in a stock or nonstock corporation, (2) the validation of proxies, (3) the manner
and validity of elections and (4) the qualifications of candidates, including the proclamation of winners. If all matters anteceding the
holding of such election which affect its manner and conduct, such as the proxy solicitation process, are deemed within the original
and exclusive jurisdiction of the SEC, then the prospect of overlapping and competing jurisdictions between that body and the regular
courts becomes frighteningly real.

From the language of Section 5(c) of Presidential Decree No. 902-A, it is indubitable that controversies as to the qualification of
voting shares, or the validity of votes cast in favor of a candidate for election to the board of directors are properly cognizable and
adjudicable by the regular courts exercising original and exclusive jurisdiction over election cases.

106
CASE DIGEST | Corporation Law (2019) Atty. Villanueva-Castro ib3h
Under Section 20.1, the solicitation of proxies must be in accordance with rules and regulations issued by the SEC, such as AIRR-
SRC Rule 4. And by virtue of Section 53.1, the SEC has the discretion "to make such investigations as it deems necessary to determine
whether any person has violated" any rule issued by it, such as AIRR-SRC Rule 4. The investigatory power of the SEC established
by Section 53.1 is central to its regulatory authority, most crucial to the public interest especially as it may pertain to corporations
with publicly traded shares. Questions relating to the proper solicitation of proxies used in election are now cognizable by the regular
courts. However, the power of SEC to regulate proxies remains extant and could very well be exercised when stockholders vote on
matters other than the election of directors.

SEC v. PERFORMANCE FOREIGN EXCHANGE CORP.


GR NO. 154131
July 20, 2006

Sec. 64. Cease and Desist Order. – 64.1. The Commission, after proper investigation or verification, motu proprio, or upon verified
complaint by any aggrieved party, may issue a cease and desist order without the necessity of a prior hearing if in its judgment the
act or practice, unless restrained, will operate as a fraud on investors or is otherwise likely to cause grave or irreparable injury or
prejudice to the investing public.

Under the above provision, there are two essential requirements that must be complied with by the SEC before it may issue a cease
and desist order: First, it must conduct proper investigation or verification; and Second, there must be a finding that the act or practice,
unless restrained, will operate as a fraud on investors or is otherwise likely to cause grave or irreparable injury or prejudice to the
investing public. It bears stressing that the investigation and verification, to be proper, must be conducted by petitioner before, not
after, issuing the Cease and Desist Order.

107

Das könnte Ihnen auch gefallen